You are on page 1of 175

PREFACE

NTS, a project of its own kind is aimed at setting standards of excellence in


educational testing. NTS is a rapidly growing and credible entity of national
repute. Its context and potential coincides with the expressed need of the
Government for the establishment of a National Accreditation Council and
Testing Service.

Establishment of this set-up has been necessitated by the prevailing lack of


credibility and absence of standardized testing facilities in our national
Institutes. Institutions waste massive amount of work hours in preparing
and conducting diverse kind of entrance tests with least credible and
authentic consequences.

NTS has made a bid to restore some order to the growing chaos in the field
of testing. It has developed the capability to offer the testing services to all
national institutes providing education. Its content is aimed at identifying
the ability-oriented strengths among prospective candidates for IT education.
NTS is a multifaceted and comprehensive facility that can provide services at
an enormous scale in all dimensions of testing. Based on a vision aimed at
providing quality services in educational testing and assessment mechanisms
for various stakeholders, NTS is committed to promoting the overall quality
of education as a value-added adjunct.

It also extends Services for the employed & unemployed professionals to get
satisfactory jobs in their relevant fields through an assessment of their
knowledge base, skills, and professional expertise & abilities. NTS is a
unique and trend-setting venture that has shown great promise.

Student Guide 1
1 NTS – INTRODUCTION
NTS was formed in 2001 with the basic objective of establishing a credible testing
system to evaluate the knowledge level of students against a consistent and
standard yardstick. It was felt that although our universities and other educational
institutions are producing a large number of qualified professional and non-
professional manpower, there is no single standard for quality evaluation of this
educated human resource. Since there is no single syllabus right from basic to the
advanced levels of education, the normal performance evaluation procedures fail to
measure the quality as well as the extent of discrepancy between the existing and
the desirable quality for individual candidates.

Similarly, a big variation in syllabi and the apparent status of the educational
institutions have created a huge gap between the haves and have-nots of the
society. The students from big name institutions always have an added advantage
where mediocre students are preferred over excellent students from low status
institutes because of the brand name of the institutions they carry with them. NTS
addresses these important issues by providing testing procedures that evaluate the
knowledge and skills of individuals on a unified scale using standardized measures
that are independent of class differentiation.

In a short period of three years, NTS has made a significant contribution by


developing and implementing training and testing programs for individual students,
educational institutions and the potential employers of the country. The hallmarks of
the NTS system are its consistency, transparency, reliability and efficiency. Being a
non-profit organization, it takes pride in offering a cost-efficient system that could
meet the needs of individuals and organizations to their utmost satisfaction.

NTS continues to strive for excellence through research and development in its
testing system and related services. The goal of the organization is to keep
contributing towards improvement in education by building standards for educational
and professional testing. Its ultimate aim is to be recognized nationally and
internationally as a credible and reliable name in the field of testing.

1.1 NTS – BACKGROUND

Th e re is a n e ve r- in c re a s in g dem an d o f qu ality prof e s s io n als aro u n d th e glo be .


O u r u n ive rs it ie s a n d o th e r e duc atio n al in s ti tu t io n s are pro du c in g large
qu a n tit ie s o f prof e s s io n a ls , ye t th e re appe ars to be n o s in gle s tan dard f o r th e
pe rf o rm a nc e e va lu a tio n o f th e se prof e s s io n als an d th e ir qu ality as s u ran c e .
S in c e th e re is n ’t a s in gle s tan dard s yllabu s , th e pe rfo rm an c e e valu atio n
pro c e du re s f a il to m ea s u re th e qu ality as we ll as th e e xte n t o f dis c re pan c y
be tw ee n th e exis tin g a n d th e de s irable qu ality o f in div idu al c an didate . Th is
s itu a t io n is ra is in g a la c k of c o nf ide n c e amo n g th e c an didate s an d th e
e n tre pre n e u rs .
To a lle via te th is , th e re is a n im m e diate ne e d fo r in c o rpo ratin g an au to no mo u s
a n d a so ve re ign bo dy, wh ic h s ho u ld c ate r to th e n ee ds o f th e e n tre pre n e u r an d
h a rn e s s th e in div idu a l ta le n t. Th is bo dy s h o u ld be m ade re s po n s ible f o r
bridg in g th e ga p be tw ee n th e edu c atio n in du s tr y an d th e prac tic a l/ pro f e s s io n al
in du s try by pro vidin g a n e valu at io n an d te s tin g s tan dard. Th is No n
G o ve rn m e n t O rga n iza tio n ( NG O ) , u n de r th e n ame o f NTS “Nat io n al Te s tin g
S e rvic e ” f a c ili ta te s th e e va lu a t io n o f gradu ate o r u n de rgradu ate c an didate s fo r
a dm is s io n to a n y u n ive rs it y. NTS w ill n o t on ly le ad th e edu c atio n al in du s try to

Student Guide 2
a he a lth y co m pe tit io n bu t a ls o im pro ve th e s tan dards of th e e du c atio n al
in s ti tu t io n s in a ll f ie lds .

1.2. NTS – Objectives


W ith in th e o ve ra ll prem is e o f c o n tribu t in g to th e im pro ve me n t o f th e qu ality of
e du c a tio n in Pa kis ta n , th e im po rtan t obje c tive s o f NTS in c lu de :

• To im pro ve u po n th e sh o rtc o m in gs in o u r pre vailin g pe rf o rm an ce e valu atio n


s ys te m of c an dida te s th ro u gh es tabl is h in g a c re dible , te c h n ic all y
a ppro pria te a n d a c c u ra te s ys te m of pe rf o rm an ce e valu atio n fo r s tu de n ts o f
dif f e re n t e duc a tio n a l in s ti tu t io n s at vario u s le ve ls .

• To pro vide a s ta n da rdize d an d u n if o rm c rite r io n f o r se le c tin g c an didate s


s ee kin g a dm is s io n to va rio u s u n ive rs it ie s in th e c o un try an d abro ad an d
a ls o to pro vide a ba s is on w h ic h s tu de n ts , in dividu a ls an d in s titu tio n s c an
a s s e s s pe rf o rm a nc e fo r pre paratio n o f vario u s type s o f te s ts c o n du c te d by
dif f e re n t in te rn a tio n a l te s tin g age n c ie s .

• E n a blin g a s s e ss m en t a n d ran kin g n o t on ly in a partic u l ar s u bje c t on a


ge ne ra l co m pe te n c y le ve l bu t als o h e lp in ide n tif ic a tio n of are as o f
s tre n gth s a n d w e a kn es s e s in its s u b- c o n te n ts on a dis aggre gate d bas is .

• To pro vide a re lia ble a n d ef f ic ie n t s ys tem fo r e valu atin g th e kn o w le dge bas e


a n d s kills of c a n dida te s in c lu din g pro fe s s io n als - e mplo ye d an d u ne m plo ye d.

• E s ta blis h in g a c re dible a n d re liable me as u re of as s e s s in g th e s tu den ts ’


kn o w le dge in c om m on ly ta u gh t pro gram s f o r re lative ran kin g.

• To bridge th e ga p be tw ee n ac ade m ic pre paratio n an d th e prac tic al m arke t


n ee ds in a h igh ly dyn a m ic an d co m pe tit ive e n viro n me n t.

1.3 NTS – COMMITMENT

National Testing Service (NTS) is a non profit, autonomous organization established


with the basic objective of developing a credible, technically appropriate and
accurate system to evaluate the knowledge level of students of different educational
institutions at various levels. It thus assists in carrying out an estimated ranking of
students for admissions in various educational institutions and also provides potential
employers with a credible yardstick for evaluation and induction of quality human
resource. NTS operates on national level and is committed to:

1.3.1 Promoting Overall Quality of Education


B a s e d o n a vis io n a ime d a t pro vid in g qu ality S e rvic e s in e duc atio n al te s tin g
a n d a ss e s sm e n t me ch a n is m s fo r vario u s s take h o lde rs , NTS is co mm itte d to
pro mo tin g th e o ve ra ll qu a lity o f e du c atio n as a valu e - adde d adju n c t.

1.3.2 Performance Evaluation


NTS u n de rta ke s pro gra m s th a t are aim e d at e s tablis h in g a c re dible , te ch n ic al ly
a ppro pria te and a c c u ra te s ys tem of pe rfo rm an c e e valu atio n of
s tu de n ts / c a n dida te s of va rio u s e du c atio n al in s titu tio n s at dif f e re n t le ve ls .

1.3.3 Assessing Knowledge, Skills & Professional Abilities

Student Guide 3
I t a ls o e xte n ds se rvic e s f o r th e e m plo ye d an d u n em plo ye d pro fe s s io n als to ge t
s a tis f a c to ry j o bs in th e ir re le van t are a th ro u gh an ass e s sm e n t o f th e ir
kn o w le dge ba s e , s kil ls , a n d pro f e s s io n al abili tie s .

1.3.4 A Rapidly Growing & Credible Entity


NTS is a ra pid ly gro w in g a n d c re dible e n tity o f n atio n al re pu te . I ts c o n te xt an d
po te n tia l co in c ide s w ith th e e xpre s se d ne e d o f th e Go ve rn me n t fo r th e
e s ta blis h m e n t of a Na tio n a l Ac c re ditat io n C o u n c il an d Te s tin g Se rvic e .

1.3.5 Conducting Various Types of Tests


W h ile th e be gin n in gs we re made by c o n du c tin g Ge ne ral Te s ts th ro u gh an I T-
f oc u s e d th ru s t, NTS h a s e xpan de d its se rvic e s to o th e r dis c ipl in e s o f
e du c a tio n a l te s tin g. I t no w pro vide s se rvic e s f o r co n du c tin g Su bje c t Te s ts by
pro vidin g fa c ili tie s bo th fo r in div idu al c an didate s an d in s titu tio n s . Th e se te s ts
a re pre pa re d a n d a s se s s e d by e xpe rts an d pro f es s io n als f ro m re le van t f ie lds .

1.3.6 Linkages & Networks


NTS n ow h a s grow in g lin kage s an d as s o c iatio n s w ith edu c atio n al an d
pro f e ss io n a l o rga n iza t io n s & I n s titu tio n s , in c lu din g th o se en gage d in tale n t
s e a rc h a n d c a ree r & e du c a tio n al pro mo tio n ac tiv it ie s , at bo th n atio n al an d
in te rn at io n a l le ve ls .

1.4 NTS- BENEFITS


NTS a im s a t be in g be ne f ic ia l f o r bo th s tu den ts an d in s ti tu te s .

1.4.1 Students
NTS Te s ts e n a ble s tu de n ts to draw be n ef its in a n um be r o f w ays :

• I t pro vide s a s in gle s ta n dard f o r me as u rin g kn o w le dge bas e an d als o s kil l


le ve l eva lu a t io n o f c a n dida te s .
• C a n dida te s c a n h a ve a m o re tran s pare n t, c re dible an d s tan dardi ze d
a s s e s sm e n t of th e ir kn o w le dge.
• B es ide s be in g e co n om ic a l, NTS dec lare s th e te s t res u lts in ju s t a fe w days
tim e .
• NTS ’ s co re s a re a re lia ble so u rc e to m e as u re c an didate s ’ kno w le dge in
c om mo n ly ta u gh t a c a de m ic co u rs e s .
• Te s tin g th ro u gh NTS a u tom atic al ly le ads to s tan dardiza tio n at a n atio n al
le ve l, wh ic h e lim in a te s c h an ce s o f large dro po u ts du rin g th e c o u rs e of
e du c a tio n .

1.4.2 Institutions
B es ide s s tu de n ts , NTS is a ls o u se f u l f o r th e e du c atio n al in s titu te s an d s im ilar
e n titie s :

• I ns titu te s , w is h in g to ge t a dm is s io n te s ts pre pare d f o r th e ir c an didate s , an d


a ls o w a n tin g to ge t th e m s c o re d in a m o re tran s pare n t an d in a le s s tim e
c o n s um in g m a n ne r c a n re ly o n NTS .
• Th is te s tin g se rvic e is h igh ly re liab le , ef f ic ie n t an d s tan dardize d; so an y of
th e in s titu te s c an s h a re th e ir re s po n s ibil ity w ith NTS .
• Th e pre pa ra t io n a n d h o ldin g o f te s ts c o ns u me a lo t of tim e an d ef fo rt. Th e
in s ti tu te s c a n le s s e n th is bu rde n o f re s po n s ibi li ty of co n du c tin g te s ts by
s h a rin g it w ith NTS . Pre paratio n of re gis trat io n fo rm s, ro ll n um be r s lips ,
qu e s tio n pa pe rs , a n sw e r s he e ts , s atis f ac to ry arran ge me n ts f o r in vigi lat io n ,
pa pe r m a rkin g, re s u lt de c laratio n e tc . are arran ge d by NTS .

Student Guide 4
• Th e I ns titu te s c a n to ta lly re ly o n NTS f o r e ff ic ie n c y of S e rvic e s in c lu din g
tra n s pa re n c y o f re s u lts .

• Th e I n s titu te s a re pro vide d w ith a c re dible s ys te m of pe rf o rm an ce ran kin g.

1.5 NTS – SERVICES


NTS provides its services in vast areas:

1.5.1 National Aptitude/General Testing


NTS conducts General Tests so as to evaluate the critical thinking skills of
analytical reasoning and also performance assessment of candidates with
regard to quantitative & verbal ability. General Tests also called National
A p t i t u d e T e s t s , h e l p e s t a b l i s h i n g a r e l i a b l e a n d c r e d i b l e m e a s u r e o f ju d g i n g
the knowledge base of students in commonly taught programs.

1.5.2 Services - Measuring Knowledge Base and Skill Levels


T h e N T S S u b j e c t T e s t s a r e p r e p a r e d a n d de s i g n e d w i t h a v i e w t o a s s e s s i n g
the qualifications, knowledge base, competency and skill level of the
c a n d i d a t e s i n a s pe c i f i c a r e a o f s t u d y .

1.5.3 Services for Test Preparation


NTS provides complete guidelines with regard to Test Preparation. A guide
for users is available on our website as well as in the market in printed form.
M o de l Q u e s t i o n s a n d S a m p l e T e s t s a r e a l s o a v a i l a b l e o n t h e w e b s i t e f o r
helping the candidates to practice and be well prepared for the test.
Furthermore Reference Books are also recommended by NTS to be an
a d d i t i o n a l s o u r c e o f h e l p f o r t h e t e s t t a k e r s f o r c o m p u t e r S c i e n c e S u b je c t
Tests.

1.5.4 Administrative Services


NTS provides an administrative service for not only the candidates but also
t h e i n s t i t u t i o n s . A l l a dm i n i s t r a t i v e a r r a n g e m e n t s r e g a r d i n g r e g i s t r a t i o n , t e s t
center selection, test execution, result preparation and then its reporting etc,
are taken as a responsibility by NTS.

1.5.5 Admission Test Services for Institutions


NTS conducts Admission Tests for various institutes and institutions, all
s e r v i c e s l i k e t h e a r r a n g e m e n t s o f t e s t r e g i s t r a t i o n , p a p e r ge n e r a t i o n , t e s t
e x e c u t i o n , p a pe r m a r k i n g , r e s u l t d e c l a r a t i o n e t c . a r e p r o v i d e d b y N T S f o r
these entities.

1.5.6 Services for Online Testing


NTS also conducts Online Tests for the candidates’ efficient and credible
evaluation of their knowledge and skills. Arrangements like the availability of
l a b s , c o n t e n t d e ve l o p m e n t f o r t h e a s s e s s m e n t , o n l i n e p a p e r g e n e r a t i o n a n d
t e s t e x e c u t i o n a n d o n l i n e r e s u l t f o rm a t i o n i m m e d i a t e l y a f t e r t h e t e s t , a r e
made by NTS.

1.5.7 Establishing Performance Ranking


NTS also assists in carrying out an estimated ranking of students as well as
p r o f e s s i o n a l s . T h i s r a n k i n g i s n o t o n l y i n a p a r t i c u l a r s u b je c t o n a g e n e r a l
competency level but also helps in the identification of areas of strengths and
weaknesses in its sub-contents on a disaggregated basis.

Student Guide 5
1.5.8 Services for Score Reporting
F o r G e n e r a l T e s t s , N T S d e c l a r e s i t s r e s u l t s e le c t r o n i c a l l y w i t h i n s e v e n d a y s o f
the test being conducted, the results of each candidate are sent to their
homes through a fast and reliable courier service.

The result of Online Subject Tests is displayed online immediately after the
c a n d i d a t e c o m p l e t e s h i s / h e r t e s t . T h e s c o r e o f c a n d i d a t e s i s a l s o r e po r t e d i n
the form of a certificate at their respective addresses.

1.5.9 Services to Facilitate Employment


NTS extends its services for employed and unemployed candidates to get
s a t i s f a c t o r y j o b s i n t h e i r r e l e v a n t a r e a s t h r o u g h a ve r y e s t i m a t e d a n d c a r e f u l
j u d g m e n t o f t h e i r k n o w l e d ge b a s e a n d s k i l l s .

I t f a c i l i t a t e s a c c e s s t o j o b o p po r t u n i t i e s f o r v a r i o u s p r o f e s s i o n a l s a n d
c a n d i d a t e s bo t h w i t h i n t h e c o u n t r y a n d a b r o a d o n t h e b a s i s o f t h e i r s c o r e s i n
NTS Subject Tests.

1.5.10 Statistical Analysis and Reports


NTS by virtue of it’s flexible and user friendly solution can better help the
institutions and organizations to have statistical analysis and report for
administrative tasks and it will ultimately lead to true decision making and
managerial tasks.

1.5.11 Survey
N T S h a s v e r y r o b u s t a n d a c c u r a t e s y s t e m f o r p r o c e s s i n g f o rm s a n d s u r v e y s .
In addition to the educational and professional testing services NTS has the
survey facility for different organizations, government data collection and
other methods to get feedback from people.

1.6 NTS – PARTNERS

1.6.1 COMSATS Institute of Information Technology

COMSATS, through a notification in 1979, setup the COMSATS Institute of


I n f o rm a t i o n T e c h n o l o g y ( C I I T ) , w h i c h b e c a m e f u n c t i o n a l i n A p r i l 1 9 9 8 . C I I T
is a world-class teaching institute, creating knowledge base and advanced
technologies that are intended to shape the new direction of the country and
educate its future leaders for the 21st century.

The aim of the Institute is to train the country’s human resource in the field
o f I n f o rm a t i o n T e c h n o l o g y s o a s t o e q u i p t h e m w i t h e m e r g i n g I T t r e n d s . T h e
C I I T i s f u l l y e q u i p p e d t o c o pe w i t h t h e n e e d s o f a g r o w i n g a n d f a s t - p a c e d I T
industry, flexible enough to adapt to the challenges of the future. The CIIT
has a mission to deliver new ideas and products through research,
development and education in strategic partnership with IT industry and
organizations.

CIIT has a total of five campuses across the country, two in Islamabad, one
e a c h i n W a h , A b bo t t a b a d a n d L a h o r e .

Online Test

Student Guide 6
N T S i s c u r r e n t l y c o n d u c t i n g o n l i n e e x am ( 2 0 % ) f o r t h e s t u d e n t s a t C I I T
A b b o t t a b a d a n d i t w i l l l i k e l y t o b e a do p t e d i n a l l t h e c a m p u s e s i n n e a r f u t u r e .
S t u d e n t s w h i l e t a k i n g t h i s k i n d o f t e s t c a n h a v e a b e t t e r v i s i o n a n d f ro m
academic and teaching point of view NTS can prepare different statistical
a n a l y s i s a n d r e p o r t s t o i m p r o v e t h e q u a l i t y o f e du c a t i o n , p erformance and
knowledge.

A d m issio n Test s for CI I T


N a t i o n a l T e s t i n g S e r v i c e ( N T S ) h as be e n c o n d u c t i n g A d m i s s i o n T e s t s f o r C I I T
across all its campuses since July 2002. The General Tests are conducted for
admissions at both Graduate and Post-Graduate levels catering for candidates
from different educational backgrounds.

1.6.2 Virtual University

Virtual University (VU) is one of the priority projects of the Ministry of


Science and Technology (MoST), which participates as a key contributor in
t h e s o c i o - e c o n o m i c de v e l o p m e n t o f t h e c o u n t r y . V U i s b a s e d o n a n e x c e l l e n t
telecommunication infrastructure and provides educational facilities on a
nationwide basis.

N TS Co nd uc t s Ad m issio n Test s fo r VU
NTS conducted admission tests (NAT-IS) at various Private Virtual Centers
(PVCs) of VU in August ’02. To facilitate the conduct of these Tests, NTS
provided Services on a countrywide basis simultaneously in 135 centers
spread over 51 major cities of Pakistan.

O n b e h a l f o f t h e s a t i s f a c t o r y a n d s u c c e s s f u l s e r v i c e s p r o v i de d b y N T S , V i r t u a l
U n i ve r s i t y r e p o s e d i t s c o m p l e t e t r u s t i n t h e a u t h e n t i c i t y a n d c r e di b i l i t y o f
NTS’ initial testing for the university which they reconfirmed by their pledge
to continue availing the services of NTS for any further testing. We certainly
view this with great appreciation and encouragement.

1.6.3 Higher Education Commission (HEC)

The Higher Education Commission has been setup to facilitate the


d e v e l o p m e n t o f t h e u n i v e r s i t i e s o f P a k i s t a n t o be c o m e w o r l d - c l a s s c e n t e r s o f
e d u c a t i o n , r e s e a r c h a n d d e v e l o pm e n t . A critical additional challenge is to
vastly increase access to higher education in Pakistan so that our talented
p e o p l e a r e n o t d e n i e d a v e n u e s f o r pe r s o n a l a n d p r o f e s s i o n a l g r o w t h . The
H E C i n t e n d s t o p l a y i t s p a r t i n s pe a r h e a d i n g t h e b u i l d i n g o f a k n o w l e d g e -
based economy in Pakistan. People are the real assets of Pakistan however it
i s o n l y t h r o u g h e d u c a t i o n a n d r e s e a r c h t h a t o u r c o u n t r y c a n p r o f i t f ro m t h i s
great asset. A well-educated graduate is the building block of a knowledge-
based economy, and it is for this reason that the HEC is focusing on the issue
of “quality” of higher education.

Co nd uc t ing Sc ho la r ship Test s fo r H EC

In February 2003 NTS successfully conducted Subject tests for various areas
of study across Pakistan on behalf of the Higher Education Commission and
since then all the scholarship tests (Subject/General) offered by HEC are
carried out by NTS.

1.6.4 Pakistan Telecommunication Corporation Limited


(PTCL)

Student Guide 7
P a k i s t a n T e l e c o m m u n i c a t i o n C o r p o r a t i o n ( P T C ) s e t s a i l s f o r i t s vo y a g e o f
g l o r y i n D e c e m b e r 1 9 9 0 , t a k i n g o ve r o p e r a t i o n s a n d f u n c t i o n s f r o m P a k i s t a n
T e l e p h o n e a n d T e l e g r a p h De p a r t m e n t u n de r P a k i s t a n T e l e c o m m u n i c a t i o n
C o r p o r a t i o n A c t 1 9 9 1 . T h i s c o i n c i d e d w i t h t h e G o v e r n m e n t ' s c o m pe t i t i v e
policy, encouraging private sector participation and resulting in award of
l i c e n s e s f o r c e l l u l a r , c a r d - o pe r a t e d p a y p h o n e s , p a g i n g a n d, l a t e l y , d a t a
communication services.

S i n c e t h e n , P T C L h a s be e n w o r k i n g v i g o r o u s l y t o m e e t t h e d u a l c h a l l e n g e o f
t e l e c o m d e v e l o pm e n t a n d s o c i o - e c o n o m i c u p l i f t o f t h e c o u n t r y . T h i s i s
c h a r a c t e r i z e d b y a c l e a r a p p r e c i a t i o n o f o n go i n g t e l e c o m s c e n a r i o w h e r e i n
convergence of technologies continuously changes the shape of the sector. A
measure of this understanding is reflected by progressive measures such as
e s t a b l i s h m e n t o f t h e c o m p a n y ' s m o b i l e a n d I n te r n e t s u b s i d i a r i e s i n 1 9 9 8 .

The telecom sector of Pakistan has also entered into a new era. The monopoly
of PTCL is now over and the sector is moving towards liberalization. While
liberalization and competition are synonymous, the future scenario also poses
some challenges to the market dominance of PTCL. The Company is ready to
face this challenge and maintain its dominating position, while guarding its
r e v e n u e s t r e a m s i n t h e f a c e o f f o r t h c o m i n g c o m pe t i t i o n .

Recruitment Test for PTCL


Being an elite client, PTCL uses the services of National Testing Service [NTS]
f o r h i r i n g pe o p l e f o r t h e o r g a n i z a t i o n . A ro u n d t h o u s a n d s o f c a n d i d a t e s a p p l y
for jobs in PTCL through the testing services of NTS.

1.6.5 Bolan Medical College

Bolan Medical College as seat of medical education in the province of


Balochistan has a remarkable history.

Admission Test for BOLAN Medical College


N T S h a s b r o a d e n e d i t s s e r v i c e s a n d h a s a l s o c o n d u c t e d a dm i s s i o n t e s t f o r
Bolan Medical College Quetta, the conduct and outcome of which was to the
entire satisfaction of all candidates. The students could tally their score with
the key displayed afterwards.

1.6.6 Engineering University Khuzdar

N T S do e s n o t l i m i t i t s s e r v i c e s t o a s p e c i f i c e d u c a t i o n a l g r o u p b u t h as a l s o
p r o v i d e d i t s s e r v i c e s f o r t h e c o n d u c t o f A dm i s s i o n T e s t s a t E n g i n e e r i n g
University Khuzdar.

6.6.7 Federal Urdu University of Arts, Science and Technology,


Islamabad

T h e g o v e r n m e n t h a s e s t a b l i s h e d a F e de r a l U r d u U n i v e r s i t y o f A r t s , S c i e n c e s
and Technology Islamabad through an ordinance with Urdu to remain the
main language of instruction and teaching. The university will have its
p r i n c i p a l s e a t a t I s l a m a b a d a n d w i l l h a v e t h e F e de r a l G o v e r n m e n t U r d u
Science College, Karachi, and Federal Government Urdu Arts College, Karachi,
as constituent institutions. Under the rule, the university will have academic,
financial and administrative autonomy, including the power to employ
o f f i c e r s , t e a c h e r s a n d o t h e r e m p l o y e e s w h i l e t h e u n i v e r s i t y w i l l be c o n t r o l l e d
b y a s e n a t e , a s y n d i c a t e a n d a n a c a de m i c c o u n c i l .

Student Guide 8
Recruitment Test for Urdu University
The National Testing Service [NTS] provided its services to the University for
t h e h i r i n g p u r p o s e s o f t h e i r f a c u l t y m e m b e r s i n e i g h t s u b je c t s t h u s m a k i n g
the selection transparent.

6.6.8 Board of Intermediate and Secondary Education,


Abbottabad

Recruitment Test
Abbottabad Educational Board signed up the services of NTS for hiring
e m p l o y e e s . A C o m pu t e r S c i e n c e s u b j e c t t e s t w a s c o n d u c t e d f o r t e s t i n g t h e
academic skills of the candidates. The assessment was later used to
determine the merit for final selection.

6.6.9 Others
NTS now has growing linkages and associations with many educational and
p r o f e s s i o n a l o r g a n i z a t i o n s a n d i n s t i t u t i o n s , i n c l u d i n g t h o s e e n g a ge d i n t a l e n t
s e a r c h , c a r e e r a n d e d u c a t i o n a l p r o m o t i o n a l a c t i v i t i e s , a t bo t h n a t i o n a l a n d
international levels.

As a rapidly growing and credible entity of national repute, the context and
potential of NTS coincide with the expressed need of the Government for the
establishment of a National Accreditation Council and Testing Service.

1.7 NTS – Tests

N T S ge ne r ally co nducts tw o kinds o f te sts:

• General Tests
• Subject Tests

T he se are co nducte d bo th as co mpute r - aide d, pape r base d and o nline


te sts, as re quir e d.

General Tests

NTS co n du c ts dif fe re n t type s of Ge n e ral Te s ts for th e gradu ate an d


u n de rgra du a te s tu de n ts ;

T h e G e n e r a l T e s t c o m p r i s e s Q u a n t i t a t i v e , A n a l y t i c a l R e as o n i n g a n d V e r b a l
Sections for assessment.

Subject Tests

Subject Tests are designed to evaluate the educational skills in a specific


a r e a o f s t u d y . S o f a r N T S h a s c o n d u c t e d t h e S u b je c t T e s t i n d i f f e r e n t
subjects:

1. Physics
2. Electronics/Engineering
3. Biology
4. Chemistry
5. Pharmacy

Student Guide 9
6. Computer Science
7. Mathematics
8. Telecommunication
9. Biochemistry
10. Management Science
11. Health Sciences
12. Environmental Science
13. Earth Science
14. Statistics
15. Veterinary Science
16. Economics
17. Agricultural Science
18. Islamic Studies
19. Pakistan Studies
20. English

1.8 NTS – TEST CENTERS

NTS’ Head Office is in Islamabad, it conducts tests across Pakistan, for any
Institute or institution requesting our services, providing sufficient time is
provided for preparation.

1.9 SCHEDULE AND INFORMATION

There are several sources for obtaining information and schedules on the NTS
Tests.

i Advertisements through Printed Media


ii Advertisements through Electronic Media
iii NTS Allied Institutions
iv NTS and its Allied Institutions' Website

1.10 CONTACT NTS

For Tests and Test Center queries related to Score Reporting, and also for other
reasons like putting complaints or providing Feedback, you may:

Visit NT S at its w e bsite i.e . htt p :/ / w w w .nt s.o r g .p k

W r ite to N T S at
Sec r et a r y
N at io na l Test ing Ser v ic e (N TS)
Offic e # 10 – 11,
Fir st Flo or Pla za 2000
Plo t # 43, Ma r kaz I- 8, I slam a b a d .

Email Dir e cto r NT S at d ir ec t or nt s@c iit .net .p k


d ir ec t or nt s@nt s.o r g .p k

Student Guide 10
Email N T S at nt sc o or d @c iit .net .p k
sup p o r t @nt s.o r g .p k

TECHNICAL REVIEW

Student Guide 11
2. Technical Procedures and Operations
2.1 Content Collection

As NTS is an autonomous body following a set of SOPs, trying to maintain the National and
International state of the art technology and standards. Keeping this vision in mind, NTS team
would like to share the most important and central activity that is how the contents are being
gradually increased, updated, the template upon which questions are being collected and after
carefully collecting the questions how quality is controlled. Template creation and paper
generation for individual tests is flexible and fair enough to meet the requirements of different
clients and produces results is different scenarios reliably. NTS test engine and paper
processing mechanisms are matured and error prone and we are still trying to introduce new
features.
NTS is highly conscious about the sensitive issue of contents. To date, NTS has conducted
The Subject Test in several subjects and has formulated Subject Committees for
each of those subjects. The committee members consist of subject specialists from
various institutes and organizations, providing valuable input in building up an
authentic and reliable database.

Subject Committees
These committees are made for each subject i.e. Engineering, Biology, Physics etc.,
comprising of a Chairperson being a PhD doctor and at least four members who have
a Masters Degree. Making curricula, policies, identifying other subject experts for
the Review and Question Making Committee are all responsibilities of this working
group. Other committees formulated by the Subject Committee are:

i. Review Committees
ii. Question Making Committees

NTS operations and methodology for content collection and improved quality is very secure
and protected. We put the contents in encrypted/coded format into the central database along
with other precautions to make the system safe and fool proof. NTS has huge central database
in different areas of study. The method of question collection is of two types.

• Paper Base Content Collection


• Online Content Collection

2.1.1 Paper Based Content Collection

For paper based content collection a template has been designed with some fields and
attributes, and information required are associated with each question. Some are narrated

• Education Level of the Question


• Difficulty Level of the Question (Easy, Moderate, Difficult)
• Reference Information
As the question can be obtained from the book, own creativity (self-made), website etc., in
any case the additional information is required.
As an example in case a question is picked from the book the following information will be
linked with the question

• Title & edition of the referenced book


• Publisher of the referenced book
• Topic & subtopic the questions belong to

Student Guide 12
• Page No. of the Book.

Student Guide 13
The Sample Template for Content Collection

Field
Subject:
Section:
Branch/Topic
Sub-Topic
Type:  Conceptual  Practical  logical
Statement:

Difficulty Level:  Easy  Moderate  Hard


Max/Min Time
Correct Answer:
Possible Choice:
(Min 4, Max 5)

Author: 
Website:  Details:
Book: 

Student Guide 14
After collecting the paper based question it is carefully entered into the central repository by
skilled Data Entry Operators. Each Question passes through different stages of Quality Control
and undergoes the detailed review to ensure the Quality and finally it becomes the part of NTS
Question Bank.

2.1.2

Online Content Collection


As it is the age if IT and internet so NTS has also the online mechanism for building question
bank we have the user friendly web based front end to collect the questions in different
disciplines. The requirement is almost the same i.e. to ensure the quality of questions.

The Snapshot of the Front End for Online Content Collection is


Give Below

Student Guide 15
2.2 Review Process

2.2.1 Review by Reviewer


As soon as question is added by the data entry operator, it becomes available to the reviewer
for review. Reviewer takes care of the typing mistakes, spelling, and formatting (font face,
font size) of the question. Questions that are marked as reviewed by reviewer become non-
editable for this operator.

1.2 Technical Review

Student Guide 16
After review from reviewer, it is then viewed technically by a person/committee nominated by
the peer committee relevant to that subject for the verification of the educational level,
difficulty level, quality, correct answer, time, etc., regarding each question

2.2.3 Grammatical Review


The grammatical reviewer takes care of all the questions for grammatical and language point
of view.

Note: in both cases either paper based or online content collection, all the question passes
through the same review process.

2.3 Paper Generation Process


NTS information system is capable of generating fully-formatted paper by selecting questions
randomly and/or according to the template requirement.
The Following can be specified as an input while generating the Paper

 Education level of the paper


 Subjects/Topics/Subtopics to be included in the paper.
 No. of easy, moderate, difficult questions to be included from each subject
 Selection of the questions can be controlled by including or excluding topics/subtopics.
 Selection of a question can also be limited on the basis of “No. of times question used”

Note: By default, the system automatically selects 30% easy, 40% moderate and 30%
difficult questions to generate a paper. However this can be changed.

After the template is defined, as many sample papers can be generated as required having the
following in it;

 Each sample paper can be a distinct one.


 Each sample paper can be re-randomized to change the sequence of the question.

If the No. of questions selected from a topic or subtopic is not according to requirement, then
same template can be redefined on topic or subtopic level by limiting the No. of questions
from each topic or subtopic. The discussed process of paper generation is for both Paper based
and online test.
For online test the questions are picked randomly from the template constructed from the
question bank and the test engine manipulates the sequence of questions for every candidate

Test Engine
For online test NTS has developed a very robust, user friendly, stable and secured Test Engine
for online test. It has very distinguished features like on spot registration, shuffling the
questions randomly for each candidate, result display immediately after the completion of test,
tracking of user session, flexible enough to cope with the requirement of a particular exam,
continuous time monitoring, pass box for placing questions temporarily, pictures handling in
the question statement and question choice, can accommodate multiple true answers, manage
scenario based questions, and toggle freely between Question Box and Pass Box.

Student Guide 17
Glimpse of the web based online test application is given below User Registration
Process

Instructions for the Test

Student Guide 18
Question Box

Pass Box

Student Guide 19
Paper Checking and Result Preparation
NTS has very reliable, error free, efficient and transparent system for paper checking and
result preparation. It is fast enough and flexible to cope with the requirements of any client.
As a reference we are quoting the Bolan Medical College, Quetta “Entry Test” we meet the
requirement of delivering the result after few hours on the same day the test was conducted.
The result was handed over along with merit lists in their required format and the notable
feature was “display of answer key on the spot” form where any candidate could calculate his/
her result because the carbon copy of answer sheet was given to the candidates after they
finished their test. Despite all these facts there was not a single error in the result which
shows the competency, reliability, transparency of NTS Paper Checking and Result Preparation
System. We are continuously improving our System day by day by introducing new features
and customization. A team of skilled professionals are dedicatedly working on the task.

Student Guide 20
Sample of an Answer Sheet is shown.

Student Guide 21
PATTERNS AND
PRACTICE

Student Guide 22
2 Quantitative Ability
The Quantitative section measures your basic mathematical skills,
understanding of elementary mathematical concepts, and the ability to reason
quantitatively and solve problems in a quantitative setting. There is a balance
o f q u e s t i o n s r e q u i r i n g b a s i c k n o w l e d g e o f a r i t h m e t i c , a l g e b r a , ge o m e t r y , a n d
data analysis. These are essential content areas usually studied at the high
school level.

T h e q u e s t i o n s i n t h e q u a n t i t a t i v e s e c t i o n c a n a l s o be f r o m

• Dis c re te Q u a n tita t ive Q ue s tio n


• Q u a n tita ti ve C om pa ris o n Q u e s tio n
• Da ta I n te rpre ta tio n Q ue s tio n e tc.

The distribution in this guide is only to facilitate the candidates. This


distribution is not a part of test template, so, a test may contain all the
questions of one format or may have a random number of questions of
different formats.

T h i s c h a p t e r i s d i v i d e d i n t o 4 m a jo r s e c t i o n s . T h e f i r s t d i s c u s s e s t h e s y l l a b u s
/contents in each section of the test respectively and the remaining three
s e c t i o n s a d d r e s s t h e q u e s t i o n f o rm a t , g u i d e l i n e s t o a t t e m p t t h e q u e s t i o n s i n
each format and some example questions.

2.1 General Mathematics Review

2.1.1 Arithmetic

The following are some key points, which are phrased here to refresh your
knowledge of basic arithmetic principles.

Basic arithmetic

• F o r a n y nu m be r a , e xa c tly o ne of th e f o llo w in g is tru e :


o a is n e ga tive
o a is ze ro
o a is po s itive

• Th e on ly n um be r th a t is e qu al to its o ppos ite is 0 (e . g.


a = −a only if a = 0 )

• I f 0 is mu ltip lie d to a n y o the r nu m be r, it w ill m ake it ze ro ( a × 0 = 0 ).

• Pro du c t o r qu o tie n t o f two n u m be rs of th e s am e s ign are alw ays po s itive


a n d of a dif f e re n t s ign a re alw ays n e gative . E. g. if a po s itive n u m be r is
m u ltipl ie d to a ne ga tive n um be r th e re s u lt w ill be ne gative an d if a ne gative
n u m be r is divide d by a n o th e r n e gative n u m be r th e re s u lt w ill be po s itive .

S ee th e f o llo w in g ta ble s fo r a ll co m bin atio n s .

Student Guide 23
+ ÷ or × + = +

+ ÷ or × − = −

− ÷ or × + = −

− ÷ or × − = +
• Th e s um of tw o po s itive n um be rs is alw ays po s itive .

• Th e s um of tw o n e ga tive nu m be rs is alw ays ne gative .

• S u btra c tin g a n u m be r f rom an o th e r is th e s am e as addin g its o ppos ite


a − b = a + ( −b )
1
• Th e re c ipro c a l of a n u m be r a is a

• Th e pro du c t o f a n um be r a n d its re c ipro c al is alw ays on e


1
a× =1
a

• Div idin g by a n um be r is th e s ame as m u ltip lyin g by its re c ipro c al


1
a ÷b = a ×
b

• E ve ry in te ge r h a s a f in ite se t o f fac t o r s ( divis o rs ) an d an in f in ite s e t of


mul t i p l ie r s.

• I f a a n d b a re tw o in te ge rs , th e fo llo w in g fo u r te rm s are s yn o n ym s
o a is a di vi so r o f b
o a is a f a c t or of b
o b is a di vi si b l e b y a
o b is a mul t i pl e of a

Th e y a ll m ea n th a t wh e n a is divide d by b th e re is no re mai nd e r .

• Po s itive in te ge rs , o th e r th a n 1, h ave at le as t two po s itive f ac to rs .

• Po s itive in te ge rs , o th e r th a n 1, wh ic h h ave e xac tly two f ac to rs , are kn o w n


a s pr i me numb e r s.

• E ve ry in te ge r gre a te r th a n 1 th at is no t a prim e c an be w ritte n as a pro du c t


o f prim e s .
To f in d th e prim e f a c to riza tio n of an in te ge r, f in d an y tw o fac to rs o f th at
n u m be r, if bo th a re prim e s, yo u are do ne ; if no t, c o n tin u e f ac to riza tio n
u n til e a c h fa c to r is a prim e .
E . g. to f in d th e prim e fa c to riza tio n of 48, tw o f ac to rs are 8 an d 6. Bo th o f
th e m a re n o t prim e n um be rs , s o co n tin u e to f ac to r th em .
F a c to rs o f 8 a re 4 a n d 2, a n d of 4 are 2 an d 2 ( 2 × 2 × 2) .
F a c to rs o f 6 a re 3 a n d 2 ( 3 × 2) .
S o th e n um be r 48 c a n be w ritte n as 2 × 2 × 2 × 2 × 3.

Student Guide 24
• Th e L e a s t Co mm o n M u ltip le ( LC M ) of two in te ge rs a an d b is th e s m alle s t
in te ge r wh ic h is divis ib le by bo th a an d b, e . g. th e LC M o f 6 an d 9 is 18.

• Th e G re a te s t Co mm o n Divis o r (G C D) o f tw o in te ge rs a an d b is th e large s t
in te ge r wh ic h divide s bo th a an d b, e. g. th e G C D o f 6 an d 9 is 3 .

• Th e pro du c t o f G C D a n d L C M of two in te ge rs is equ al to th e pro du c ts of


n u m be rs its e lf . E. g. 6 × 9 = 54
3 × 18 = 54 ( w he re 3 is G C D an d 18 is LCM o f 6 an d 9) .

• E ve n nu m be rs a re a ll th e mu lt iple s of 2 e. g. (… , − 4, − 2, 0, 2, 4, … )

• O dd n um be rs a re a ll in te ge rs no t divis ib le by 2 ( … , −5, −3, −1, 1, 3, 5, … )

• I f tw o in te ge rs a re bo th eve n o r bo th o dd, th e ir s um an d dif fe re n c e are


e ve n.

• I f o n e in te ge r is e ve n a n d th e o th e r is o dd, th e ir s um an d dif fe re n c e are


o dd.

• Th e pro du c t o f tw o in te ge rs is e ve n u n le s s bo th o f th e m are o dd.

• W h e n a n e qu a tio n in vo lve s m o re th an o ne o pe ratio n , it is im po rtan t to c arry


th e m o u t in th e co rre c t o rde r. Th e co rre c t o rde r is P are n th e se s ,
E xpo ne n ts , Mu ltip li c a t io n a n d D ivis io n , A dditio n an d S u btrac tio n , o r ju s t
th e f irs t le tte rs P EMD A S to re me m be r th e pro pe r o rde r.

Ex p o nent s a nd Ro ot s

• R e pe a te d a dditio n o f th e s am e n u m be r is in dic ate d by mu ltip lic at io n :


17 + 17 + 17 + 17 + 17 = 5 × 17

• R e pe a te d m u ltipl ic a t io n of th e s ame n um be r is in dic ate d by an e xpo n e n t:


5
17 × 17 × 17 × 17 × 17 = 17

5
I n th e e xpre s s io n 17 , 17 is c alle d b ase an d 5 is th e e xp o ne nt.

1 n
• F o r a n y nu m be r b: b = b an d b = b × b × … × b, wh e re b is us e d n tim e s
a s f a c to r.

• F o r a n y nu m be rs b a n d c a n d po s itive in te ge rs m an d n :
o b m b n = b m +n
bm
o = b m −n
bn
o (b m ) n = b mn
o b m c m = (bc) m

n
• I f a is n e ga tive , a is po s itive if n is eve n , an d n e gative if n is o dd.

Student Guide 25
Th e re a re two n um be rs th a t s atis f y th e e qu atio n x = 9 : x = 3 an d x = −3 .
2

Th e po s itive on e , 3, is c a lle d th e ( prin c ipa l) s qu are ro o t o f 9 an d is de no te d
by s ym bo l 9 . C le a rly, e a ch pe rfe c t s qu are h as a s qu are ro o t:
0 =0 , 9 =3, 36 = 6 , 169 =13 , 225 = 25 e tc .

• F o r a n y po s iti ve n u m be r a th e re is a po s iti ve n u m be r b th at s atis f ie s th e


e qu a tio n a =b .

F o r a n y po s iti ve in te ge r, ( a ) = a× a =a.
2

• F o r a n y po s iti ve n u m be rs a an d b:
a a
o ab = a × b and
=
b b

a +b ≠ a+ b
o
as 5= 25 = 9 +16 ≠ 9 + 16 = 3 + 4 = 7

A lth o u gh it is a lw a ys tru e th at ( a ) = a ,
2
• a 2 = a is tru e on ly if a is
( −5) 2 = 25 = 5 ≠ −5
po s itive a s
n
• F o r a n y nu m be r a ,
a =a
n 2
.

• F o r a n y nu m be r a , b a n d c :

• a (b + c ) = ab + ac a (b − c ) = ab − ac

a n d if a ≠0

(b + c) b c (b − c) b c
• = + = −
a a a a a a

I neq ua lit ies

• F o r a n y nu m be r a a n d b, e xac tly on e o f th e fo llo w in g is tru e :


a >b or a =b or a <b.

• F o r a n y nu m be r a a n d b, a > b m ean s th at a − b is po s itive .

• F o r a n y nu m be r a a n d b, a < b m ean s th at a − b is n e gative .

• F o r a n y nu m be r a a n d b, a = b m ean s th at a − b is ze ro .

• Th e s ym bo l ≥ me a n s gre a te r th an o r equ al to an d th e s ym bo l ≤ m e ans


le s s th a n o r equ a l to. E . g. th e s tate m e n t x ≥ 5 me an s th at x c an be 5 o r an y
n u m be r gre a te r th a n 5.
Th e s ta te m e n t 2 < x < 5 is a n abbre viat io n of 2 < x an d x < 5 .

Student Guide 26
• A ddin g o r s u btra c tin g a n u m be r to an in e qu ality pre s e rve s it.

• If a <b , th e n a + c < b + c and a − c < b −c .

e . g. 5 < 6 ⇒ 5 +10 < 6 +10 and 5 −10 < 6 −10

• A ddin g in e qu a li tie s in s am e dire c tio n pre se rve s it:


If a < b and c <d , th e n a +c < b +d .

• M u ltip ly in g o r dividin g a n in e qu alit y by a po s iti ve n u m be r pre s e rve s it. I f


a < b and c is a po s iti ve n u m be r, th e n a ×c < b ×c an d
a b
<
c c.
• M u ltip ly in g o r dividin g a n in e qu alit y by a ne gative n um be r re ve rs e s it. I f
a < b and c is a ne ga tive n um be r, th e n a ×c > b ×c an d
a b
>
c c.
• I f s ide s o f a n in e qu a lity a re bo th po s itive an d bo th n e gative , takin g th e
re c ipro c a l re ve rs e s th e in e qu alit y.

• If 0 < x < 1 a n d a is po s itive , th e n xa < a .

• If 0 < x < 1 a n d m and n a re in te ge rs w ith m>n, th e n xm < xn < x .

• If 0 < x < 1, th e n x > x.

1 1
>x >1
• If 0 < x < 1 , then x and x

Pr o p er t ies o f Zer o

• 0 is th e o n ly n um be r th a t is ne ith e r n e gative n o r po s itive .

• 0 is s m a lle r th a n e ve ry po s iti ve n u m be r an d gre ate r th an eve ry n e gative


n u m be r.

• 0 is a n e ve n in te ge r.

• 0 is a m u ltiple o f e ve ry in te ge r.

• F o r e ve ry n u m be r a :
a + 0 = a and a − 0 = a .

• F o r e ve ry n u m be r a : a×0 = 0.

• F o r e ve ry po s itive in te ge r n : 0n = 0 .

Student Guide 27
a a
• F o r e ve ry n u m be r ( in c lu din g 0): a ÷ 0 and are un de f in e d s ym bo ls .
0

0
• F o r e ve ry n u m be r a ( o th e r th an 0) : 0÷a = = 0.
a
• 0 is th e o n ly n um be r th a t is e qu al to its o ppo s ite : 0 = −0 .

• I f th e pro du c t o f tw o o r m o re n um be rs is 0, at le as t o n e o f th e m is 0.

Properties of One
a
F o r a n y nu m be r a : a ×1 = a a n d = a.
1
• F o r a n y nu m be r n : 1n = 1 .

• 1 is th e divis o r o f e ve ry in te ge r.

• 1 is th e s m a lle s t po s itive in te ge r.

• 1 is a n o dd in te ge r.

• 1 is n o t a prim e .

Fractions and Decimals

• W h e n a w ho le is divide d in to n e qu al parts , e ac h part is c alle d on e n th of


th e w ho le , w ritte n 1. Fo r e xam ple , if a pizza is c u t ( divide d) in to 8 equ al
n
1
s lic e s , e a c h s lic e is on e e igh th ( ) o f th e pizza; a day is divide d in to 24
8
1
e qu a l h o u rs , s o a n h o u r is on e twe n ty- fo u rth ( ) o f a day an d an in c h is
24
1
o ne tw e lf th ( ) of a fo o t. I f on e wo rks f o r 8 h o u rs a day, h e wo rks e igh t
12
8
tw e n ty- fo u rth ( ) of a da y. I f a h o c ke y s tic k is 40 in c he s lo n g, it
24
40
m ea s u re s fo rty tw e lf th s ( ) of a f oo t.
12
1 1 8 40
• Th e n um be rs s u c h a s , , an d , in wh ic h on e in te ge r is w ritte n
8 24 24 12
o ve r th e s e co n d in te ge r, a re c alle d fr ac t i o ns. Th e c en te r lin e is c alle d th e
f ra c tio n ba r. Th e n um be r abo ve th e bar is c alle d th e nume r at or , an d th e
n u m be r be lo w th e ba r is c a lle d d e no mi nat or .

• Th e de no m in a to r o f a f ra c tio n c an ne ve r be 0.
1
• A f ra c tio n , su c h a s , in w h ic h th e de n om in ato r is gre ate r th an
24
n u me ra to r, is kn ow n a s a p r o pe r fr ac ti o n. I ts valu e is le s s th an on e.

Student Guide 28
40
• A f ra c tio n , su c h a s , in w h ic h th e de n om in ato r is le s s th an n um e rato r, is
12
kn o w n a s a n i mpr o p e r f r a c t i o n. I ts valu e is gre ate r th an on e .
12
• A f ra c tio n , su c h a s , in w h ic h th e de n om in ato r is e qu al to th e n u me rato r,
12
is a ls o kn o w n a s a n i mpr o p e r fr ac t i o n. B u t, I ts valu e is on e .
• E ve ry f ra c tio n c a n be expre s s e d in dec im al fo rm (o r as a wh o le nu m be r) by
divid in g th e n um be r by th e de n om in ato r.
3 3 8 48 100
= 0.3, = 0.75, = 1, = 3, = 12.5
10 4 8 16 8
• U n like th e e xa m ple s a bo ve , wh e n m os t f rac tio n s are co n ve rte d to de c im als ,
th e divis io n doe s no t te rm in ate , af te r 2 o r 3 o r 4 de c im al plac e s ; rath e r it
go es o n f o re ve r w ith so me s e t of digits re pe atin g it.
2 3 5 17
= 0.66666..., = 0.272727..., = 0.416666..., = 1.133333...
3 11 12 15

Student Guide 29
• To c om pa re tw o de c im a ls , fo llo w th e s e ru le s :

o W h ic h e ve r n um be r h as th e gre ate r n um be r to th e le f t o f th e dec im al


po in t is gre a te r: s in c e 11 > 9, 11. 0001 > 9. 8965 an d s in c e 1 > 0,
1. 234 > . 8. ( Re c a ll th at if a de c im al is w ritte n w ith o u t a n um be r on
le f t of de c im a l po in t, yo u m ay as s u me th at a 0 is th e re , so , . 8 =
0. 8) .
o I f th e n um be rs to th e le f t of th e de c im al po in t are equ al, pro ce e d as
f o llo w s :

• I f th e n um be rs do n o t h ave th e s ame n um be r o f digits to th e righ t


o f th e dec im a l po in t, add ze ro e s to th e e n d o f th e sh o rte r o ne to
m a ke th e m equ a l in le n gth .
• No w co m pa re th e n um be rs ign o rin g th e de c im al po in t.
• F o r e xam ple , to co m pare 1. 83 an d 1. 823, add a 0 to th e e n d of 1. 83
f o rm in g 1. 830. Now co m pare th em , th in k in g o f th em as wh o le
n u m be rs w ith o u t de c im al po in t: s in c e 1830 > 1823, th e n 1. 830
> 1. 823.

• Th e re a re two w a ys to c om pare f rac tio n s :


o C o n ve rt th em to de c im als by divid in g, an d u se th e m e th o d alre ady
2
de s c ribe d to co m pare th e s e de c im als . F o r e xam ple to c om pare
5
1 2 1
and , co n ve rt th e m to de c im als . = 0.4 an d = 0.25 . Now , as 0. 4
4 5 4
2 1
> 0. 25, > .
5 4
2 1
o C ro s s m u ltipl y th e f rac tio n s . Fo r e xam ple to co m pare an d ,
5 4
c ro s s mu ltip ly:
2 1 2 1
S in ce
5 4 2 × 4 > 1× 5 , th e n >
5 4.
• W h ile co m pa rin g th e f ra c tio n s , if th e y h ave s am e th e den o m in ato rs , th e
3 2
f ra c tio n w ith th e la rge r n u me rato r is gre ate r. F o r e xam ple > .
5 5
• If t he f r a c ti o ns ha ve t he same nume r at o r, t he fr ac t i o n wi t h t he
3 3
sma l l e r de no mi na t o r i s gr e at e r. Fo r e xamp le > .
5 10
• Tw o f ra c tio n s a re c a lle d eq ui val e nt fr ac t io ns if bo th o f th e m h ave s am e
de c im a l va lu e .
1 5
• Fo r e xa m ple , = a s bo th o f th e s e are e qu al to 0. 5.
2 10
• A no th e r w a y to c he c k th e equ ivale n c e o f tw o f rac tio n s is to c ro s s - mu ltip ly.
I f bo th o f th e pro du c ts a re s am e, th e f rac tio n s are equ ivale n t. Fo r E xam ple ,
2 6 2 × 15 = 6 × 5 ,
to c om pa re w ith , c ro s s - mu ltip ly. S in c e bo th o f th e
5 15
f ra c tio n s a re e qu iva le n t.

Student Guide 30
• E ve ry f ra c tio n c a n be re du ce d to low e s t te rm s by dividin g th e n u me rato r
a n d de no m in a to r by th e ir gre ate s t c om m on divis o r ( G C D) . I f th e G C D is 1,
10
th e f ra c tio n is a lre a dy in lo we s t te rm s . Fo r exam ple to re du ce , divide
15
bo th nu me ra to r a n d den o m in ato r by 5 (w h ic h is G C D o f 10 an d 15) . Th is
2
w ill re du c e th e f ra c tio n to .
3
• To mu ltip ly two f ra c tio n s , m u ltipl y th e ir n u me rato rs an d mu ltip ly th e ir
3 4 3 × 4 12
de no m in a to rs . F o r e xam ple × = = .
5 7 5 × 7 35
• To m u ltipl y a n u m be r to a f rac tio n , w rite th at n u m be r as a f rac tio n w ho s e
3 3 7 3 × 7 21
de no m in a to r is 1. Fo r e xam ple ×7 = × = = .
5 5 1 5 ×1 5
• W h e n a pro ble m re qu ire s yo u to f in d th e f rac tio n o f a n um be r, m u ltip ly th at
2
f ra c tio n w ith th e nu m be r. F o r e xam ple , to f in d two f if th ( ) o f 200,
5
80
2 2 200 400
× 200 = × = = 80
m u ltipl y: 5 5 1 5/ .
a b a b
• Th e re c ipro c a l of a f ra c tio n is ano th e r f rac tio n s in c e × =1
b a b a
• To divide on e f ra c tio n by th e o the r f rac tio n , m u ltip ly th e re c ipro c al of
2
22 11 22 7 2
divis o r w ith th e divide n d. Fo r e xam ple , ÷ = × = = 2.
7 7 7 11 1
• To a dd o r s u btra c t th e f rac tio n s w ith s ame de n om in ato r, add o r s u btrac t
n u me ra to rs and ke e p th e de n om in ato r. For e xam ple
4 1 5 4 1 3
+ = and − = .
9 9 9 9 9 9

Per c ent s

• Th e wo rd p e r ce nt me a n s hu n dre dth . W e us e th e s ym bo l % to e xpre s s th e


w o rd pe r c e nt. F o r exa m ple “15 pe rc e n t” m e ans “15 hu n dre dth s ” an d c an be
20
w ritte n w ith a % s ym bo l, a s a f rac tio n , o r as a de c im al. 20% = = 0.20 .
100
• To c on ve rt a de c im a l to a pe rc e n t, mo ve th e de c im al po in t tw o plac e s to th e
righ t, a ddin g 0s is ne c e s s a ry, an d add th e pe rc e n t s ym bo l ( % ) .
F o r e xam ple , 0. 375 = 37. 5% 0. 3 = 30% 1. 25 = 125% 10= 1000%

• To c on ve rt a f ra c tio n to a pe rc e n t, f irs t co n ve rt th at f rac tio n to de c im al,


th a n u s e th e m e tho d s ta te d abo ve to c o n ve rt it to a pe rc e n t.

• To c on ve rt a pe rc e n t to a de c im al, mo ve th e de c im al po in t tw o plac e s to th e
le f t a n d re mo ve th e % s ym bo l. A dd 0s if n e ce s s ary.
F o r e xam ple , 25% = 0. 25 1% = 0. 01 100% = 1

• Yo u s ho u ld be f am ilia r w ith th e fo llo w in g bas ic c o n ve rs io n s :

Student Guide 31
1 5 1 2
= = 0.50 = 50% = = 0.20 = 20%
2 10 5 10
1 3
= 0.25 = 25% = 0.75 = 75%
4 4

• F o r a n y po s iti ve in te ge r a : a % of 100 is a.

• F o r a n y po s iti ve n u m be rs a an d b : a % of b = b% of a

actual change
• Th e pe rc en t ch a n ge in th e qu an tity is ×100% . Fo r e xam ple :
original amount
I f th e pric e o f a la m p goe s f ro m Rs . 80 to Rs . 100, th e ac tu al in c re as e is
20 1
R s . 20, a n d th e pe rce n t in c re as e is × 100% = × 100% = 25% .
80 4
• If a < b , th e pe rc e n t in c re a s e in go in g f ro m a to b is alw ays gre ate r th an
pe rc e n t de c re a se in go in g f rom b to a .

• To in c re a s e a n u m be r by k % , m u ltipl y it by 1 + k % , an d to de c re as e a
n u m be r by k % , mu ltip ly it by 1 − k % . Fo r e xam ple , th e valu e of an
in ve s tm e n t o f R s. 20, 000 af te r 25% in c re as e is
20, 000 × (1 + 25%) = 20, 000 × (1.25) = 25, 000 .

• I f a n u m be r is th e re s u lt of in c re as in g an o th e r n um be r by k% , to f in d th e
o rigin a l n um be r divide it by 1 + k % , an d if a n um be r is th e re s u lt of
de c re a s in g a n o th e r n um be r by k % , to f in d th e o rigin al n um be r, divide it by
1− k% .
F o r e xam ple , Th e go ve rn me n t an n o u n ce d an 20% in c re as e in s alarie s . I f
a f te r th e in c re me n t, Th e s alary of a partic u la r em plo ye e is R s . 18, 000,
w h a t w a s th e o rigin a l s a la ry?
current salary 18, 000 18, 000
O rigin a l s a la ry ( in R s. ) = = = = 15, 000
1 + percent increase 1 + 20% 1.20

Ra t io s a nd Pr o p or t io ns

• A ra t io is a f ra c tio n th a t c om pare s tw o qu an tit ie s th at are m eas u re d in th e


s a me un its . Th e f irs t qu a n tity is th e n um e rato r an d th e s e co n d qu an tit y is
de no m in a to r. F o r e xam ple , if th e re are 16 bo ys an d 4 girls , we s ay th at th e
ra tio o f th e n um be r of bo ys to th e nu m be r of girls on th e te am is 16 to 4,
16
or . Th is is of te n w rit te n as 16: 4. S in c e a ratio is ju s t a f rac tio n , it c an
4
be re du ce d o r co n ve rte d to a de c im al o r a pe rc en t. Th e F o llo w in g are
dif f e re n t w a ys to e xpre s s th e s ame ratio :
16 4
16 to 4 , 16 : 4 , 0.25 , 25%
4 , 1,

Student Guide 32
• I f a s e t o f obj e c ts is divide d in to tw o gro u ps in th e ratio n a :b , th e n th e
a
f irs t gro u p co n ta in s of th e to tal o bjec ts an d s im ilar ly th e s e co n d gro u p
a+b
b
c o n ta in s of th e to ta l n um be r o f obje c ts . Th is ru le applie s to e xte n de d
a+b
ra tio s , a s w e ll. I f a s e t is divide d in to th re e gro u ps in th e ratio a :b:c , th e n
a
th e f irs t gro u p c o n ta in s o f th e to tal o bje c ts , an d so on .
a+b+c

• A pro po rtio n is a n equ a tio n th at s tate s th at tw o ratio s are e qu ivale n t. S inc e


4 10
ra tio s a re j u s t f ra c tio n s , a n y e qu atio n su c h as = in wh ic h eac h s ide is
6 15
a s in gle f ra c tio n is pro po rtio n . Th is pro po rtio n s tate s th at 4 re late s to 6 in
s a me ra tio a s 10 re la te s to 15.
a c ad = bc .
• F o r e a c h pro po rtio n of th e fo rm = , Th is pro pe rty c an be u se d
b d
to so lve pro po rtio n s f o r u n kn o wn s ( variable s ) . Fo r e xam ple : “I f 3 o ran ges
c o s t R s . 5, ho w m a n y o ra n ge s c an yo u bu y f o r R s. 100”. To so lve th is
pro ble m we h a ve to se t u p a pro po rtio n . I f th e n um be r of o ran ge s fo r
R s . 100 is x , th e n:
3 x 3 ×100
= ⇒ 3 ×100 = x × 5 ⇒ x = ⇒ x = 60
5 100 5
A v er a g es

• Th e a ve r a g e o f a s e t of n n u m be rs is th e s um o f th o s e n u m be rs divide d by
n.
sum of n numbers Sum
average = o r s im ply A=
n n
th e te c h n ic a l n am e f o r th e se kin d o f ave rage s is A rith m e tic Me an .

• I f yo u kn ow th e a ve ra ge o f n n um be rs , mu ltip ly th at ave rage w ith n to ge t


th e s um of n um be rs .

• I f a ll th e n um be rs in a se t are th e s ame , th en th at n um be r is th e ave rage .

• A s s um e th a t th e a ve ra ge o f a s e t of nu m be rs is A. I f a ne w n u m be r x is
a dde d to th a t s e t, th e n e w ave rage w ill be ;

o G r e a t er i f x i s g r e at er t han t he e xi st i ng ave r ag e
o S ma l l e r i f x i s smal l er t han t he e xi st i ng ave r ag e
o Unc ha ng e d i f x i s e q ual to t he e xi st i ng ave r ag e

• A rith m e tic se qu e n ce is a n o rde re d s e t o f n um be rs , s uc h th at, th e dif f e re n ce


be tw ee n tw o c o n se c u tive nu m be rs is th e s am e.

• I f th e re is a n a rith m e tic se qu e n ce o f n te rm s , th e n th e ave rage c alc u lat io n


c a n be m a de s im ple by u s in g th e s e ru le s .

Student Guide 33
o Th e a ve ra ge of th e te rm s in th at s e que n c e w ill be th e m iddle te rm ,
if n is o dd.
o I f n is e ve n, th e a ve rage w ill be th e ave rage o f tw o m iddle te rm s.

2.1.2 Algebra
Po lyno m ia ls

• A mo n om ia l is a n y nu m be r o r variab le o r pro du c t of n u m be rs an d variab le s .

F o r e xam ple
3, − 4, x, y, 3 x, − 2 xyz , 5 x 3 , 1.5 xy 2 , a 3b 2 are all mo n om ials .

• Th e n um be r th a t a ppe a rs in f ro n t of a variab le in a m on o m ial is c alle d th e

c oe f f ic ie n t. Th e co e ff ic ie n t of 5x 3 is 5. I f th e re is no n u m be r, th e co e ff ic ie n t
is e ith e r 1 o r –1, be c a u se x m e ans 1x an d − x m ean s −1x .

• A po lyn o m ia l is a mo no m ial o r th e s um o f tw o o r m o re mo no m ials . E ac h


m on om ia l th a t m a ke s u p th e po lyn o m ial is c alle d a te rm of th at po lyn o m ial.

• I f a po lyn o m ia l h a s o n ly o ne te rm it is a s im ple m on om ial, if it h as tw o


te rm s , it is kno w n a s bin o m ial an d if it h as th re e te rm s , it is c alle d
trin o m ia l.

• Tw o te rm s a re c a lle d like te rm s if th e y dif fe r o n ly in th e ir c oe f f ic ie n ts . 5x 3


and −2x 3 a re like te rm s , wh e re as , 5x 3 an d 5x 2 are no t.

• I f like te rm s a re in vo lve d in a po lyn om ial, th e y c an be co m bin e d, by addin g


th e ir co e ff ic ie n ts , to ma ke th at po lyn o m ial s im ple r. Th e po lyn o m ial
3x 2 + 4 x + 5 x − 2 x 2 − 7 is e qu ivale n t to th e po lyn o m ial x2 + 9 x − 7 .
• A ll la w s of a rith m e tic a re a ls o applic ab le to po lyn o m ials . M os t im po rtan t o f
th e m is PE M DA S .

• Po lyn o m ia ls c a n be a dde d, s u btrac te d, m u ltip lie d o r divide d.

• To a dd tw o po lyn o m ia ls , pu t a plu s s ign be tw e e n th e m , e ras e th e


pa re n th e s e s , a n d co m bin e like te rm s .

Exa mp l e :
W h a t is th e s u m o f 5 x 2 + 10 x − 7 an d 3x 2 − 4 x + 2 ?

S o l ut io n:
(5 x 2 + 10 x − 7) + (3 x 2 − 4 x + 2)
= 5 x 2 + 10 x − 7 + 3 x 2 − 4 x + 2
= 8x2 + 6 x − 5

Student Guide 34
• To s u btra c t tw o po lyn o m ia ls , re ve rs e th e s ign s of su btrah e n d, an d add tw o
po lyn o m ia ls a s do ne bef o re .

Exa mp l e :
S u btra c t 3x 2 − 4 x + 2 f ro m 5 x 2 + 10 x − 7

S o l ut io n:
(5 x 2 + 10 x − 7) − (3 x 2 − 4 x + 2)
= (5 x 2 + 10 x − 7) + (−3 x 2 + 4 x − 2)
= 5 x 2 + 10 x − 7 − 3 x 2 + 4 x − 2
= 2 x 2 + 14 x − 9
• To m u ltipl y mo no m ia ls , f irs t m u ltipl y th e ir c o ef f ic ie n ts , an d th e n mu ltip ly
th e ir va ria ble s by a ddin g th e e xpo ne n ts .

Exa mp l e :

W h a t is th e pro du c t o f
3x 2 yz f rom
−2x 2 y 2 ?

S o l ut io n:
(3x 2 yz )(−2 x 2 y 2 )
= (3 × −2)( x 2 × x 2 )( y × y 2 )( z )
= −6 x 4 y 3 z

• To m u ltipl y a m on o m ia l by a po lyn o m ial, ju s t m u ltip ly e ac h te rm of th e


po lyn o m ia l by th e m on om ia l.

Exa mp l e :

W h a t is th e pro du c t o f 3x f rom
3 x 2 − 6 xy 2 + 2 ?

S o l ut io n:
(3x )(3 x 2 − 6 xy 2 + 2)
= (3 x × 3 x 2 ) − (3 x × 6 xy 2 ) + (3 x × 2)
= 9 x 3 − 18 x 2 y 2 + 6 x

• To m u ltipl y two bin o m ia ls , m u ltip ly e ac h te rm of f irs t bin om ial by e ac h te rm


o f s e co n d bin o m ia l, th e n a dd th e re s u lts .

Exa mp l e :

W h a t is th e pro du c t o f
3x + y f rom
3 x 2 − 6 xy 2 ?

Student Guide 35
S o l ut io n:
(3x + y )(3x 2 − 6 xy 2 )
= (3 x × 3 x 2 ) + (3 x × (−6 xy 2 )) + ( y × 3 x 2 ) + ( y × (−6 xy 2 ))
= (9 x 2 ) + (−18 x 2 y 2 ) + (3 x 2 y ) + (−6 xy 3 )
= 9 x 2 − 18 x 2 y 2 + 3 x 2 y − 6 xy 3

• Th e th re e mo s t im po rta n t bin om ial pro du c ts are :

o
( x − y )( x + y ) = x 2 + xy − xy + y 2 = x 2 + y 2

o
( x + y )( x + y ) = x 2 + xy + xy + y 2 = x 2 + 2 xy + y 2

o
( x − y )( x − y ) = x 2 − xy − xy + y 2 = x 2 − 2 xy + y 2
M em o rizin g th e s e c an s ave a lo t of c alc u latio n tim e du rin g th e te s t.

• To divide a po lyn o m ia l by a mo no m ial, divide e ac h te rm o f th e po lyn o m ial


by th e mo n om ia l.

Exa mp l e :

W h a t is th e qu o tie n t if
32 x 2 y + 12 xy 3 z is divide d by
8xy ?

S o l ut io n:
32 x 2 y + 12 xy 3 z 32 x 2 y 12 xy 3 z 3
= + = 4 x + y 2 z (by reducing the terms)
8 xy 8 xy 8 xy 2

Student Guide 36
Solving Equations and Inequalities

• Th e ba s ic prin c ip le in so lvin g e qu atio n s an d in e qu alit ie s is th at yo u c an


m a n ipu la te th e m in a n y w ay as lo n g as yo u do t he same t hi ng to bo t h
si d e s . Fo r e xa m ple yo u m ay add a n u m be r to bo th s ide s , o r yo u m ay
divide o r mu ltip ly bo th s ide s w ith s ame n um be r e tc .
• B y u s in g th e fo llo w in g s ix- s te p me th o d, yo u c an s o lve m os t o f th e equ atio n s
a n d in e qu a litie s . Th e me th o d is explain e d w ith th e h e lp of an e xam ple .

Exa mp l e :
1 x?
if x + 3( x − 2) = 2( x + 1) + 1 , wh at is th e valu e o f
2

S o l ut io n:
Step Wha t t o do … Examp l e
1 Ge t rid o f f ra c tio n s an d M u ltipl y eac h s ide by 2 to ge t:
de c im a ls by mu ltip ly in g x + 6( x − 2) = 4( x + 1) + 2
bo th s ide s by th e L C D.
2 Ge t rid o f a ll pa re n th e se s x + 6 x − 12 = 4 x + 4 + 2
by so lvin g th e m .
3 Co m bin e like te rm s on 7 x − 12 = 4 x + 6
ea c h s ide .
4 B y a ddin g a n d s u btra c tin g 4x f ro m
Su btrac t e ac h s ide to ge t:
ge t a ll th e va ria ble s o n
on e s ide ( mo s tly le f t) . 3 x − 12 = 6
5 B y a ddin g o r s u btra c tin g Add 12 to e ac h s ide to ge t:
ge t a ll pla in n um be rs o n 3 x = 18
th e o th e r s ide .
6 Divide bo th s ide s by th e Divide bo th s ide s by 3 to ge t:
co e ff ic ie n t of th e va ria ble . x=6
(I f yo u a re de a lin g w ith
an in e qu a li ty and yo u
divide w ith a ne ga tive
n um be r, rem em be r to
re ve rs e th e in e qu a lity. )
• W h e n yo u h a ve to s o lve on e variab le an d th e e qu atio n / in e qu ali ty in vo lve
m o re th a n o ne va ria ble , tre at all o the r variab le s as plain n um be rs an d apply
th e s ix- s te p m e tho d.
Exa mp l e :
if a = 3b − c , wh a t is th e va lu e o f b in te rm s of a an d c?

S o l ut io n:
S te p W ha t t o d o … Examp l e
1 Th e re a re no f rac tio n s an d
de c im a ls .
2 Th e re a re no pare n th e se s .
3 Th e re a re no like te rms .
4 B y a ddin g a n d su btrac t in g ge t Re me m be r th e re is o n ly o ne
a ll th e va ria ble s o n o ne s ide .
variable b , wh ic h is on o ne
s ide on ly.
5 B y a ddin g o r su btrac t in g ge t Re me m be r we are
a ll pla in n um be rs on th e co n s ide rin g a an d c as
o th e r s ide .

Student Guide 37
plain n um be r. A dd c to
eac h s ide to ge t:
a + c = 3b
6 Div ide bo th s ide s by th e Divide bo th s ide s by 3 to
c oe f f ic ie n t o f th e variab le . ge t:
a+c
=b
3
• I t is n o t n e ce s s a ry to f o llo w th e s e s te ps in th e o rde r s pe c if ie d. S om e tim e s
it m a ke s th e pro ble m m u ch e as ie r, if yo u c h an ge th e o rde r of th es e s te ps.

Exa mp l e :
If x − 4 = 11 , wh a t is th e va lu e o f x - 8?

S o l ut io n:
G o in g imm e dia te ly to s te p 5, add 4 to e ac h s ide to ge t: x = 15 . Now
s u btra c t 8 f rom bo th s ide s to ge t: x −8 = 7 .

• Do in g th e s a me th in g on eac h s ide of an e qu atio n doe s no t me an do in g th e


s a me th in g to e a c h te rm o f th e e qu atio n . Th is is ve ry im po rtan t if yo u are
do in g divis io n s , o r de a lin g w ith expo n e n ts an d ro o ts .

Exa mp l e :
If a>0 and a 2 + b2 = c2 , w h at is th e valu e of a in te rm s o f b an d c.

S o l ut io n:
a 2 + b 2 = c 2 ⇒ a 2 = c 2 − b 2 . Now yo u c an ’t take a s qu are ro o t of eac h te rm
to ge t a = c − b . Yo u mu s t take th e s qu are roo t o f e ac h s ide :
a 2 = c 2 − b2 ⇒ a = c 2 − b2

• A no th e r type o f e qu a tio n is th at in w h ic h th e variable appe ars in e xpon e n t.


Th e s e e qu a tio n s a re ba s ic a ll y so lve d by in c e ptio n .
E xa m ple :
If 2 = 32 , wh a t is th e va lu e o f
x+ 3
3x+ 2 ?
S o lu tio n :
2 x +3 = 32 ⇒ 2 x +3 = 25 ⇒ x+3= 5 ⇒ x = 2.

No w a s x = 2, you c a n ge t
x = 2 ⇒ x + 2 = 4 ⇒ 3x + 2 = 34 = 81

• A s ys te m of equ a tio n s is a s e t o f tw o o r m o re e qu atio n s h avin g tw o o r m o re


va ria b le s . To so lve s u c h equ atio n s , yo u mu s t f in d th e valu e of eac h
va ria b le th a t w ill m a ke e a c h equ atio n tru e .

• To s o lve a s ys te m of equ a tio n s , add o r s u btrac t th em to ge t a th ird


e qu a tio n . I f th e re a re m o re th an tw o e qu atio n s yo u c an ju s t add th e m.

Exa mp l e :
If
x + y = 10 and
x − y = 10 w h at is th e valu e o f
y?

Student Guide 38
S o l ut io n:
x + y = 10
A dd two e qu a tio n s : x− y = 2
2x = 12 ⇒ x = 6
No w re pla c in g x w ith 6 in th e f irs t equ atio n :
6 + y = 10 ⇒ y=4

• I f yo u kn ow th e va lu e o f on e variable in a s ys tem o f tw o equ atio n s , yo u c an


u s e th is va lu e to ge t th e valu e o f th e o th e r variable . As it is do n e in th e
pre vio u s qu e s tio n .

Word problems
• To s o lve wo rd pro ble m s , f irs t tran s late th e pro ble m f ro m E n glis h to A lge bra.
W h ile tra n s la t in g, us e va riable s to re pre se n t u n kn o w ns . O n ce tran s late d, it
is e a s y to so lve th e m u s in g th e te c h n iqu e s yo u h ave le arn e d in pre vio u s
s e c tio n s .
• F o llo w in g E n glis h to A lge bra dic tio n ary w ill be h e lpf u l in tran s la tin g w o rd
pro ble m s to a lge bra ic expre s s io n s .

Mat he mat i c al
Eng l i sh wo r d s S y mb ol
me ani ng
I s , w a s, w ill be , h a d, h a s, w ill E qu als =
h a ve, is e qu a l to , is th e s a m e as
Plu s , mo re th a n , s u m , in c re as e d A dditio n +
by, a dde d to , e xce e ds , re ce ive d,
go t, o lde r th a n , f a rth e r th an ,
gre a te r th a n
M in u s , fe we r, le s s th an , S u btrac tio n −
dif fe re n c e , de c re a se d by,
s u btra c te d f rom , yo u n ge r th an ,
ga ve , lo s t
Tim e s , o f, pro du c t, mu lti pl ie d by M u ltip lic a tio n ×
a
Divide d by, quo tie n t, pe r, f o r Div is io n ÷ or
b
Mo re th a n , gre a te r th a n I ne qu ality >
A t le as t I ne qu ality ≥
Fe we r th a n , le s s th a n I ne qu ality <
A t m os t I ne qu ality ≤
W h a t, ho w ma n y, e tc. U n kn ow n qu an tity x
( So me variable )

Examples:
o The sum of 5 and some number is 13. 5 + x = 13
o Javed was two years younger than Saleem. J = S −2
o Bilal has at most Rs.10,000. B ≤ 10000
o The product of 2 and a number exceeds that
number by 5 (is 5 more than that number). 2N = N + 5

• I n wo rd pro ble m s , yo u mu s t be su re abo u t wh at you are an sw e rin g. Do n o t


a n s we r th e w ro n g que s tio n .

Student Guide 39
• I n pro ble m s in vo lvin g a ge s, rem em be r th at “ye ars ago ” me an s yo u ne e d to
s u btra c t, a n d “ye a rs f rom no w ” m ean s you n ee d to add.

• Dis ta n c e pro ble m s a ll de pe n d o n th re e variat io n s of th e s ame fo rm u la:

o distance = speed × time


distance
speed =
o time
distance
time =
o
speed

Exa mp l e :
H ow m u c h lo n ge r, in se c on ds , is re qu ire d to drive 1 m ile at 40 m ile s pe r
hour th a n at 60 m ile s pe r ho u r?

S o l ut io n:
Th e tim e to drive a t 40 m ile s pe r h o u r c an be c alc u late d as
1 1 3 30
time1 = h o u rs = × 60 3 m in u te s = × 60 se c o n ds = 90
40 40 2 2
s e co n ds

Th e tim e to drive a t 60 m ile s pe r h o u r c an be c alc u late d as

time2 1 1 1× 60 se c o n ds 60 s e co n ds
= h o u rs = × 60 m in u te s = =
60 60
difference = time1 − time2 = 90 − 60 = 30 se c o n ds .

2.1.3 Geometry

L ines a nd Ang les

• A n a ng l e is fo rme d a t th e in te rs e c tio n of two lin e s e gm e n ts , rays o r lin e s .


Th e po in t o f in te rs e c tio n is c alle d th e ve r t e x. A n gle s are m e as u re d in
de gre e s.

• A n gle s a re c la s s if ie d a c co rdin g to th e ir de gre e me as u res .


• A n a c ut e a n gle me a s u re s le s s th an 90°

• A r i g ht a n gle m e as u re s 90°

• A n o b t use a n gle me a s u re s m o re th an 90° bu t le s s th an 180°


• A s tra i gh t a n gle m ea s u re s 180°

• I f tw o o r m o re a n gle s co m bin e to ge th e r to fo rm a s traigh t an gle , th e su m o f


th e ir me a s u re s is 180° .

Student Guide 40
a° + b° + c° + d ° = 180°

• Th e s u m of a ll th e me a s u re s o f all th e an gle s aro u n d a po in t is 360°

a° + b° + c° + d ° + e° = 360°

• W h e n tw o lin e s in te rs e c t, f o u r an gle s are f o rm e d, tw o an gle s in e ac h pair of


o ppo s ite a n gle s a re c a lle d ve r t ic al ang l e s. V e rtic al an gle s , fo rm e d by th e
in te rs e c tio n of tw o lin e s , h ave equ al me as u re s.

a=c and b=d

• I f o n e o f th e a n gle s fo rme d by th e in te rs e c tio n o f tw o lin e s is a righ t an gle ,


th e n a ll fo u r a n gle s a re righ t an gle s . S uc h lin e s are c alle d pe r p e nd i c ul ar
lin e s

a = b = c = 90

• I n th e f igu re be lo w a lin e l divide s th e an gle in tw o equ al parts . Th is lin e is


s a id to bi se c t th e a n gle . Th e o the r lin e k bis e c ts ano th e r lin e in to tw o
e qu a l pa rts . Th is lin e is s a id to bis e c t a lin e .

• Tw o lin e s a re s a id to be p ar al le l , if th e y ne ve r in te rs e c t eac h o the r.


H ow e ve r, if a th ird lin e , c alle d a t r ansve r sal , in te rs e c ts a pair o f paral le l
lin e s , e igh t a n gle s a re f o rm e d. A n d th e re latio n s h ip am on g th e s e s an gle s is
s h ow n in th e fo llo w in g dia gram .

Student Guide 41
• - A ll f ou r ac u te an gle s are equ al
a=c=e= g
• - A ll f ou r o btu s e an gle s are e qu al
b=d = f =h
• - Th e sum of an y pair of ac u te an d
• 180° ,
o btu s e an gle is e. g.
a + d = 180°, d + e = 180°, b + g = 180° e tc .

Triangles
• I n a n y tria n gle , th e s u m of th e me as u res of th e th re e an gle s is 180° .

x + y + z = 180

• I n a n y tria n gle :
o Th e lo n ge s t s ide of trian gle is o ppo s ite th e large s t an gle .
o Th e s ho rte s t s ide is o ppo s ite th e s m alle s t an gle .
o S ide s w ith th e s a me le n gth are o ppo s ite th e an gle s w ith th e s am e
m ea s u re .

• Tria n g le s a re c la s s if ie d in to th re e dif f e re n t kin ds w ith re s pe c t to th e le n gth s


o f s ide s .
o S c a l e ne : in w h ic h all th re e s ide s are o f dif f e re n t le n gth s .
o Iso sc e l e s : in w h ic h two of th e s ide s o f trian gle are e qu al in
le n gth , th e th ird is dif f e re n t.
o Eq ui l a t e r a l : in w h ic h all th re e s ide s are e qu al in le n gth .

• Tria n g le s a re a ls o c la s s if ie d w ith re s pe c t to th e an gle s .


o A c ut e tr i a ng l e : in w h ic h all th re e an gle s are ac u te .
o O b t use tr i a ng l e : in w h ic h o ne an gle is o btu s e an d tw o are ac u te .
o Ri g ht t r i a ng le : Th is h as on e righ t an d two ac u te an gle s .

• I n a righ t tria n g le , th e o ppo s ite to th e righ t an gle is kn o w n as hy p o t e nuse


a n d is th e lo n ge s t s ide . Th e o the r tw o s ide s are c alle d le g s .

• I n a n y righ t tria n g le , th e s um of th e m eas u re s o f th e tw o ac u te an gle s is


90° .

• B y Pyth a go re a n Th e o rem , th e s u m of s qu are s o f th e le n gth s of le gs o f a


righ t tri a n gle is a lw a ys e qu al to th e s qu are o f le n gth o f h ypo te n u s e.

Student Guide 42
a 2 + b2 = c2
• I n a n y tria n gle , th e s u m o f an y tw o s ide s is alw ays gre ate r th an th e th ird
o ne . A n d th e dif f e re n ce of an y tw o s ide s is alw ays le s s th an th e th ird on e .

a +b > c an d a −b < c

• Th e pe r i me t er of a tria n gle is c alc u late d by addin g th e le n gth s o f all th e


s ide s o f th a t tria n gle .

perimeter = a + b + c
1
area = bh
• Th e a re a of a tri a n gle is c a lc u late d by th e fo rm u la: 2 w h e re b is
th e ba s e o f th e tria n gle a n d h is th e hei g ht of th e trian gle .
o A n y s ide o f tria n g le c an be take n as th e bas e.
o H e igh t is th e a lti tu de ( pe rpe n dic u lar) draw n to th e bas e f ro m its
o ppo s ite ve rte x.
o I n a righ t tria n g le an y le g co u ld be take n as th e base , th e o the r
w ill be th e a lt itu de .

Qua d r ila t er a l a nd o t her Po lyg o ns

• A po l y go n is a c lo se d ge om e tric f igu re , made u p o f lin e se gme n ts . Th e lin e


s e gm e n ts a re c a lle d si d e s an d th e en d po in ts o f lin e s are c alle d ve r t i c e s

Student Guide 43
( plu ra l o f ve r t e x). L in e s , in s ide th e po lygo n , draw n f ro m o ne ve rte x to th e
o th e r, a re c a lle d di a g o na l s .

• Th e s um of th e me a s u res o f th e n an gle s in a po lygo n w ith n s ide s is

a lw a ys
(n − 2) ×180° .

• I n a n y qu a drila te r a l, th e s um of th e m eas u re s of th e f o u r an gle s is 360° .

• A re g ul a r p o ly g o n is a po lygo n in wh ic h all o f th e s ide s are o f th e s ame


le n gth . I n a n y re gu la r po lygo n , th e me as u re of e ac h in te rio r an gle is
(n − 2) ×180° 360°
a n d th e m e as u re o f e ach e xte rio r an gle is .
n n
• A p a r a l le l o g r a m is a s pe c ial qu adrila te ral, in w h ic h bo th pairs of o ppo s ite
s ide s a re pa ra lle l. Th e Fo llo w in g are so me pro pe rtie s o f paral le lo gram .

o L e n gth s of o ppo s ite s ide s are e qu al. AB = CD an d AD = BC


o M e a s u re s o f o ppo s ite an gle s are e qu al. a = c an d b = d
o C o n se c u tive a n gle s add u p to 180° . a + b = 180° , b + c = 180° e tc .

o Th e tw o dia go n a ls bis e c t e ac h o th e r. AE = EC an d BE = ED
o A dia go n a l divide s th e paralle lo gram in to tw o trian g le s th at are
c o n gru e n t.

• A r e c t a ng le is a pa ra lle lo gr am in w h ic h all fo u r an gle s are righ t an gle s . I t


h a s a ll th e pro pe rtie s of a paralle lo gram . I n additio n it h as th e f o llo w in g
pro pe rtie s :
o Th e me a s u re of ea c h an gle in a rec tan gle is 90° .
o Th e dia go n a ls of a re c tan gle are e qu al in le n gth .

• A sq ua r e is a re c ta n gle th at h as th e f o llo w in g additio n a l pro pe rtie s :


o A s qu a re h a s a ll its s ide s e qu al in le n gth .
o I n a s qu a re , dia go n als are pe rpe n dic u lar to e ac h o th e r.

Student Guide 44
• To c a lc u la te th e a re a, th e fo llo w in g fo rm u las are re qu ire d:
o F o r a pa ra l le lo gra m , Area = bh , wh e re b is th e base an d h is th e
h e igh t.
o F o r a re c ta n gle , Area = lw , wh e re l is th e le n gth an d w is th e
w idth .
o F o r a s qu a re , Area = s 2 , wh e re s is th e s ide o f th e s qu are .

• Pe rim e te r f o r a n y po lygo n is th e s u m o f le n gth s , o f all its s ide s.

Student Guide 45
Circles

• A ci r c l e co n s is ts o f a ll th e po in ts th at are th e s am e dis tan c e f ro m o ne f ixe d


po in t c a lle d th e c e nt e r . Th at dis tan c e is c alle d th e r ad i us o f a c irc le . Th e
w o rd ra diu s is a ls o u s e d to re pre s e n t an y of th e lin e s e gm e n ts jo in in g th e
c e n te r a n d a po in t on th e c irc le . Th e plu ral of radiu s is r ad i i .

• A n y tria n g le , s u ch a s VCED in th e f igu re , f o rme d by co n n e c tin g th e e n d


po in ts of tw o ra dii, is a n is o s ce le s .

• A lin e se gme n t, s u c h a s ED in th e diagram abo ve , bo th o f w ho s e e n d


po in ts a re o n a c irc le is c a lle d a c ho r d .

• A c ho rd th a t pa s s e s th ro u gh th e ce n te r of th e c irc le is c alle d th e di ame t e r


o f th e c irc le . Th e le n gth o f th e diam e te r is alw ays do u ble th e radiu s o f th e
c irc le . Th e dia m e te r is th e lo n ge s t co rd th at c an be draw n in a c irc le .

• Th e to ta l le n gth a ro u n d a c irc le is kn ow n as th e ci r c umfe r e nce o f th e


c irc le .

• Th e ra tio of th e c irc u m f e re n ce to th e diam e te r is alw ays th e s am e f o r an y


c irc le . Th is ra tio is de no te d by th e s ym bo l π ( pro no u n c e d as pi) .
C
• π = ⇒ C = π d ⇒ C = 2π r w he re C is th e c irc u m f e re nc e , d is th e diam e te r
d
a n d r is th e ra diu s of th e c irc le .
• V a lu e of π is a ppro xim a te ly 3.14

• A n a rc c o n s is ts o f tw o po in ts in a c irc le an d all th e po in ts be tw ee n th em .
E . g. PQ is a n a rc in th e diagram .

• A n a n gle w ho s e ve rte x is a t th e ce n te r of th e c irc le is c alle d th e c e nt r al

a ng l e .
∠PCQ in th e dia gram abo ve is a c e nt r al ang l e .

• Th e de gree me a s u re of a c om ple te c irc le is 360° .

Student Guide 46
• Th e de gree me a s u re of a n arc is th e m eas u re of th e c e n tral an gle th at
in te rc e pts it. E . g. th e de gre e m eas u re of »
PQ is equ al to th e me as u re of
∠PCQ in th e dia gra m a bo ve .

x x
• If is th e de gre e me a s u re o f an arc , its le n gth c an be c alc u late d as C,
360
w he re C is th e c irc u m f e re n ce .
• π r2 .
Th e a re a of a c irc le c a n be c alc u late d as
• The area of a sector formed by the arc and two radii can be
x
calculated as A , where A is the area of a circle.
360

2.2 Discrete Quantitative Questions


These are standard multiple-choice questions. Most of such questions require you to do some
computations and you have to choose exactly one of the available choices based upon those
computations. This section will teach you the basic tactics to attempt such questions.

2.2.1 Question format


Each question will consist of a question statement and the choices labeled from A to E. The
number of choices may vary from 2 to 5, but exactly one choice will be correct for each
question.

2.2.2 How to attempt?

Following are some tactics, which will lead you to the correct answer.

• W h e ne ve r yo u kno w ho w to an s we r a qu e s tio n dire c tly, ju s t do it. Th e


ta c tic s s h o u ld be u s e d on ly wh e n yo u do n o t kno w th e e xac t s o lu tio n , an d
yo u j u s t wa n t to e lim in a te th e ch o ic e s .

• R em em be r th a t n o pro blem re qu ire s le n gth y o r dif f ic u lt co m pu tatio n s . I f


yo u f in d yo u rs e lf do in g a lo t o f co m ple x arith m e tic , th in k again . Yo u m ay
be go in g in th e w ro n g dire c tio n .

• W h e ne ve r th e re is a que s tio n w ith so me u n kn ow n s ( variable s ) , re plac e th e m


w ith th e a ppro pria te n u me ric valu e s fo r e ase o f c alc u lat io n .

• W h e n yo u ne e d to re pla c e variab le s w ith valu e s , ch o o se eas y- to - u s e


n u m be rs , e. g. th e n um be r 100 is appro pria te in mo s t pe rc e n t- re late d
pro ble m s a n d th e L C D ( le as t co mm o n den o m in ato r) is be s t su ite d in
qu e s tio n s th a t in vo lve f ra c tio n s .

• A pply “ba c k- s o lvin g” wh e n e ve r yo u kn ow w h at to do to an s we r th e qu e s tio n


bu t yo u w a n t to a vo id do in g alge bra. To u n de rs tan d th is tac tic re ad th e
f o llo w in g e xam ple :

O n Mo n da y, a s to reo w ne r re c e ive d a s h ipme n t of bo o ks . O n Tu e s day, s h e


s o ld h a lf o f th e m . O n W e dne s day af te r tw o mo re w e re so ld, s he h ad
e xa c tly 2/ 5 o f th e bo o ks le f t. Ho w m an y we re in th e s h ipm e n t?
( A) 10 ( B ) 20 ( C ) 30 ( D) 40 ( E ) 50

Student Guide 47
n ow by th is ta c tic :
A s s um e th a t (A ) is th e c o rre c t an sw e r, if so ; s he mu s t h ave 3 boo ks on
W e dne s da y. B u t 2/ 5 of 10 are 4, s o , ( A ) is in c o rre c t;
A s s um e th a t (B ) is th e c o rre c t an sw e r, if so ; s h e mu s t h ave 8 boo ks on
W e dne s da y. 2/ 5 o f 20 a re 8, so , ( B) is th e c o rre c t c ho ic e , an d as th e re m ay
be o n ly on e co rre c t c h o ic e , th e re is no n ee d to c he c k fo r re m ain in g c h o ic e s.
Th is ta c tic is ve ry h e lpf u l wh e n a n o rm al alge braic s o lu tio n fo r th e pro ble m
in vo lve s co m ple x o r le n gth y c alc u lat io n s .
• I f yo u a re n o t s u re h ow to an sw e r th e que s tio n , do no t le ave it u n an s we re d.
Try to e lim in a te a bs u rd ch o ic e s an d gu e s s f rom th e rem ain in g o ne s . M o s t o f
th e tim e s fo u r o f th e c ho ic e s are abs u rd an d yo u r an s we r is no lo n ge r a
gu e s s.

M a n y th in gs ma y h e lp yo u to re alize th at a partic u lar ch o ic e is abs u rd.


S om e o f th e m a re lis te d be low .

o Th e a n s w e r m u s t be po s itive bu t so m e of th e c h o ic e s are n e gative


s o e lim in a te a ll th e ne gative on e s .
o Th e a n s w e r m u s t be e ve n bu t s om e of th e ch o ic e s are o dd s o
e lim in a te a ll th e o dd c ho ic e s .
o Th e a n s w e r m u s t be le s s th e n 100, bu t so me o f th e c ho ic e s are
gre a te r th a n 100 (o r an y o th e r valu e ) so e lim in ate all c ho ic e s th at
a re o u t of ra n ge.
o Th e a n s w e r m u s t be a w ho le n um be r, bu t so me of th e c h o ic e s are
f ra c tio n s so e lim in ate all f rac tio n s .
o Th e s e a re s om e e xam ple s . Th e re may be nu m e rou s s itu at io n s
w he re yo u c an a pply th is tac tic an d f in d th e c o rre c t an s we r e ve n if
yo u do n o t kno w th e righ t way to s o lve th e pro ble m .

Example questions with solutions

Th e f o llo w in g a re s om e e xam ple s , w h ic h w ill h e lp yo u to mas te r th e se type s of


qu e s tio n s .

Example
I f 25% o f 220 equ a ls 5. 5% o f X , w h at is X ?
( A) 10 ( B ) 55 ( C ) 100 ( D) 110 ( E ) 1000

Solution:
S in c e 5. 5% o f X e qu a ls 25% o f 220, X is mu c h gre ate r th an 220. So , ch o ic e s
A , B , C , a n d D a re im me dia te ly e lim in ate d be c au se th e se are no t large r th an
220. A n d th e co rre c t a n s we r is c h o ic e E.

(Note: An important point here is that, even if you know how to solve a problem, if
you immediately see that four of the five choices are absurd, just pick the remaining
choice and move on.)
Example
S c ie n ce s tu de n ts c ho o se e xa c tly on e of th re e f ie lds ( i. e . me dic al sc ie n c e s ,
e n gin e e rin g s c ie n ce s a n d c om pu te r s c ie n c es ) . I f, in a co lle ge , th re e- f if th s of
th e s tu de n ts c ho o se m edic a l s c ie n c es , on e - fo rth of th e re m ain in g s tu de n ts take
c om pu te r s c ie n c e s , w h a t p e r ce nt of th e s tu den ts take en gin e e rin g sc ie n c e s ?
( A) 10 ( B ) 15 ( C ) 20 ( D) 25 ( E ) 30

Student Guide 48
Solution:
Th e le a s t c om mo n de no m in a to r o f 3/ 5 an d 1/ 4 is 20, s o as s u me th at th e re are
20 s tu de n ts in th a t c o lle ge . Th e n th e nu m be r of s tu den ts ch o os in g me dic al
s c ie n c e s is 12 ( 3/ 4 o f 20) . Of th e rem ain in g 8 s tu de n ts , 2 ( 1/ 4 o f 8) c ho o s e
c om pu te r s c ie n ce s . Th e rem a in in g 6 c ho o se e n gin e e rin g s c ie n ce s . A s 6 is 30%
o f 20, th e a n sw e r is E .

Example
I f a s c h oo l c a fe te ria n e e ds C c an s of s o u p eac h w ee k fo r eac h s tu de n t an d
th e re are S s tu de n ts , fo r ho w man y w ee ks w ill X c an s of so u p las t?

( A) C X/ S ( B ) X S/ C ( C ) S / CX ( D) X/ C S ( E ) CS X

Solution:
R e pla c e C , S a n d X w ith th re e e as y to u s e n u m be rs . Le t C = 2, S= 5 an d X = 20.
No w ea c h s tu de n t w ill ne e d 2 c ans pe r we e k an d th e re are 5 s tu de n ts , s o 10
c a n s a re ne e de d pe r we e k a n d 20 c an s w ill las t fo r 2 w ee ks . No w pu t th es e
va lu e s in c h o ic e s to f in d th e co rre c t on e .
Th e c h o ic e s A , B , C , D a n d E be c om e 8, 50, 1/ 8, 2 an d 200 re s pe c tive ly. So
th e c ho ic e D re pre s e n ts th e c o rre c t an s w e r.

2.3 Quantitative Comparison Questions


S om e of th e qu e s tio n s in th e Q u an tita tive se c tio n of th e te s t m ay be
qu a n tita ti ve co m pa ris o n que s tio n s . Th e Fo llo w in g te xt w ill e xplain yo u th e
f o rm a t a n d te c h n iqu e s u ne e d to atte m pt th e qu e s tio n s of th is f o rm at.

2.3.1Question format
S u c h qu e s tio n s co n s is t of tw o qu an tit ie s , o ne in c o lu m n A an d th e o the r in
c o lu m n B. You h a ve to co m pare th e tw o qu an tit ie s . Th e in f o rm atio n
c o n ce rn in g o ne o r bo th qu a n tit ie s is pre s e n te d be fo re th e m . O n ly th e fo llo w in g
f ou r ch o ic e s w ill be give n :

A. Th e qu a n tity in co lu m n A is gre ate r


B. Th e qu a n tity in co lu m n B is gre ate r
C. Th e tw o qu a n tit ie s in bo th co lu m n s are e qu al
D. Th e re la tio n s h ip c a n n o t be de te rm in e d f ro m th e in f o rm atio n give n

And as it is clear from the choices, only one will be correct at one time. Your job is
to choose one of them after careful comparison. The following text explains some
simple tactics to attempt such questions.

2.3.2How to attempt

W h e ne ve r yo u en c o un te r a qu an tit at ive co m paris o n que s tio n , th e f o llo w in g


gu ide lin e s w ill he lp you to f in d th e c o rre c t an s w e r qu ic kly.

• I f th e qu e s tio n in vo lve s s om e variable s , re plac e th em w ith appro pria te


n u m be rs . He re a re s om e gu ide lin e s in c h oo s in g an appro pria te n u m be r:
o Th e ve ry be s t n um be rs to us e are –1, 0 an d 1.
o O f te n f ra c tio n s be tw e e n 0 an d 1 are us e f u l (e . g. 1/ 2, 3/ 4 e tc . ) .
o O c c a s io n a lly, “la rge ” n um be rs s u c h as 10 o r 100 c an be u s e d.
o I f th e re is mo re th an on e variable , it is pe rm is s ible to re plac e e ac h
w ith th e s a me n um be r.
o Do n o t im po s e a n y u n - s pe c if ie d c o n ditio n s on nu m be rs . C ho o se
th e m ra n do m ly.

Student Guide 49
• E lim in a te th e c ho ic e s a n d c ho o se f rom th e re m ain in g on e s. Fo r e xam ple I f
yo u f ou n d th e qu a n titie s eve r equ al, th e co rre c t c h o ic e c ou ld n e ve r be A o r
B , s o , e lim in a te A a n d B.
• A qu a n ti ta t ive co m pa ris o n qu es tio n c an be tre ate d as an e qu atio n o r
in e qu a li ty. E ith e r:
Column A < Column B, or
Column A = Column B, or
Column A > Column B
S o, yo u c a n pe rf o rm s im ila r o pe ratio n o n bo th co lu m n s to s im plif y th e
pro ble m j u s t a s in equ a tio n s ( o r in e qu alit ie s ) .

Example:
m > 0 and m ≠ 1

m2 m3

• I n th is e xa m ple divide bo th th e qu an titie s by m 2. Th is w ill c h an ge


c o lu m n A to 1 a n d co lu m n B to m . Now th e co m paris o n is ve ry s im ple ,
a s w e kn ow th a t m is gre ate r th an 0 an d c an no t be 1. S o th e
re la t io n s h ip is no t de te rm in able u s in g th e c u rre n t in f o rm atio n . m c an be
bo th gre a te r th a n 1 o r be twe e n 0 an d le s s th an 1.

2.3.3 Example questions with Answers and


Explanations
Example 1:
A s tu de n t ea rn e d a 75 o n e ach of h e r f irs t th re e
m a th te s ts a n d an 80 on he r f o u rth an d f if th
te s ts .
A B

A ve ra ge a f te r 4 te s ts A ve rage af te r 5 te s ts

A. Th e qu a n tity in co lu m n A is gre ate r


B. Th e qu a n tity in co lu m n B is gre ate r
C. Th e tw o qu a n tit ie s in bo th co lu m n s are e qu al
D. Th e re la tio n s h ip c a n n o t be de te rm in e d f ro m th e in f o rm atio n give n
R em em be r yo u w a n t to kn ow wh ic h ave rage is h igh e r, n o t w h at th e ave rage s
a re . A f te r 4 te s ts , th e a ve ra ge is c le arly le s s th an 80, so an 80 on th e f if th
te s t h a d to ra is e th e a ve ra ge . S o th e an s w e r is c ho ic e ( B) .

Example 2:
A B

Th e tim e it take s to Th e tim e it take s to


drive 40 m ile s at 35 drive 35 m ile s at 40
m ph m ph
A. Th e qu a n tity in co lu m n A is gre ate r
B. Th e qu a n tity in co lu m n B is gre ate r
C. Th e tw o qu a n tit ie s in bo th co lu m n s are e qu al
D. Th e re la tio n s h ip c a n n o t be de te rm in e d f ro m th e in f o rm atio n give n

Student Guide 50
O n c e a ga in th e re is n o ne e d f o r c alc u latio n , as th e s pee d in c o lu m n B is h igh e r
th a n th a t in co lu m n A . I t is o bvio u s th at it w ill take le s s tim e to trave l s ho rte r
dis ta n c e a t a gre a te r s pe e d. S o th e valu e in co lu m n A is large r. Th e an s we r is
o ptio n ( A) .

Example 3:
A B

20 5
2 5
A. Th e qu a n tity in co lu m n A is gre ate r
B. Th e qu a n tity in co lu m n B is gre ate r
C. Th e tw o qu a n tit ie s in bo th co lu m n s are e qu al
D. Th e re la tio n s h ip c a n n o t be de te rm in e d f ro m th e in f o rm atio n give n
2
 20  20  5 
2
25
  = =5   = 5 =5
S qu a re ea c h co lu m n :  2  4
an d  5 . So bo th c o lu m n s are
e qu a l a n d th e a n sw e r is c ho ic e ( C ).
Example 4:
A B

13y 15y

To so lve th is que s tio n , s u btra c t


13y f ro m bo th c o lu mn s to ge t
13 y − 13 y = 0 fo r
c o lu m n A a n d
15 y − 13 y = 2 y f o r c o lu m n B. As th e re are no re s tric tio n s ,
2 y c an
be gre a te r th a n , le s s th a n o r e qu al to 0. S o th e co rre c t ch o ic e is ( D) .

2.4 Data Interpretation Questions


Th e s e qu e s tio n s a re ba s e d o n th e in f o rm atio n th at is pre s en te d in th e f o rm of a
gra ph , c h a rt o r ta ble . Mo s t of th e data is pre s e n te d graph ic al ly. Th e mo s t
c om mo n type s of gra ph s a re lin e graph s , bar graph s an d c irc le graph s . Th e
o bj ec tive of s u c h que s tio n s is to te s t yo u r abili ty to u n de rs tan d an d an alyze
s ta tis tic a l da ta .

2.4.1 Question Format


Da ta in te rpre ta t io n que s tio n s alw ays appe ar in se ts , yo u are pre s en te d w ith
s om e da ta in a n y f o rm a t ( c h art, graph o r table ) , an d yo u w ill th e n be as ke d
w ith so me que s tio n s a bo u t th a t data.

Th e fo llo w in g e xam ple e xpla in s th e fo rm at of s u c h qu e s tio n s .

Example:

Question 1:
W h a t is th e a ve ra ge s a le , in m illio n Rs . , f o r th e pe rio d 1994- 2000?
( A) 5. 5 ( B ) 6. 0 ( C ) 7. 0 ( D) 8. 0
( E ) 8. 5

Question 2:

Student Guide 51
For which year, the percentage increase in sales from the previous year is the greatest.
(A) 1995 (B) 1996 (C) 1999 (D) 2000
(E) 2001

2.4.2 How to attempt


• Do no t try to a n s we r s u c h que s tio n s im m e diate ly, f irs t o f all re ad th e
pre s e n te d da ta c a re f u lly. Yo u m us t be ve ry c le ar abo u t th e data an d its
m ea n in gs eve n be fo re re adin g th e f irs t qu es tio n .

• Do n o t co n f us e n u m be rs w ith pe rc e n ts . Th is c o n fu s io n is m os t like ly to
o c c u r wh e n da ta is pre s e n te d in pie graph s . F o r e xam ple in th e
f o llo w in g gra ph

12
Sales in million Rs.

10
8
6
4
2
0
1994 1995 1996 1997 1998 1999 2000 2001
Years

No w it w ou ld be a gre a t m is take he re to th in k th at s ale s o f “TV s & VC R s ” is


15% mo re th a n th e s a le s o f C om pu te rs in 2001 by X YZ C o rpo ratio n . To
15
×100 = 60%
kn o w th is yo u h a ve to c a lc u late it as 25 .
• Try to a vo id u n - ne c e s s ary c alc u lat io n s . Mo s t o f th e que s tio n s c o u ld
e a s ily be s o lve d by o bse rvatio n an d es tim at io n . Us e e s tim atio n to
e lim in a te th e ch o ic e s , if yo u are n o t able to f in d th e co rre c t an s w e r
w ith o u t c a lc u la tio n . F o r e xam ple to so lve “Q u e s tio n 1” pre s e n te d in th e

Student Guide 52
e xa m ple a t th e s ta rt of th is s e c tio n , if yo u are no t s u re of th e co rre c t
a n s we r, yo u c a n th e n try to c u t dow n th e nu m be r o f po s s ible c ho ic e s by
o bse rva t io n . Yo u a re be in g as ke d to te ll th e pe rce n tage in c re as e . W he re
a s , in ye a r 2000, th e s ale is de c re as in g in s te ad o f in c re as in g, s o , yo u
c a n imm e dia te ly e lim in ate ch o ic e ( D) in th at qu e s tio n .
• Yo u r a n s we rs m u s t be bas e d u po n th e in f o rm atio n pre s e n te d in th e
give n c h a rts a n d gra ph s . I f yo u r kno w le dge c o n tradic ts an y o f th e data
pre s e n te d, ign o re w h a t yo u kn o w an d s tic k to th e pre s e n te d data. Th e
pre s e n te d da ta s ho u ld be th e on ly base f o r yo u r c alc u lat io n s an d
e s tim a tio n s .
• A lw a ys u s e th e pro pe r un its , th e re m ay be s om e qu e s tio n s th at as k yo u
to c om pa re dif fe re n t data ite m s po s s ibly f ro m dif f e re n t data s e ts . B e
c a re f u l a bo u t th e un its u s e d to re pre s e n t th e data.
• B ec a u s e gra ph s a n d c h arts pre s en t data in a fo rm th at e n able s yo u to
re a dily se e th e re la tio n s h ips amo n g valu e s an d to make qu ic k
c om pa ris o n s , you sh o u ld alw ays try to vis u al ize yo u an s we r in th e s am e
f o rm a t a s th e o rigin a l data w as pre s e n te d.
• B e s u re th a t yo u r a n s we r is re aso n able . Fo r e xam ple , th e prof it c o u ld
n e ve r in c re a s e th e a c tu al s ale s , o r th e expe n s es c o u ld ne ve r be ne gative
e tc . W h ile a n sw e rin g th e qu e s tio n , f irs t o f all e lim in ate s u c h u n -
re a s o n a ble c ho ic e s , a n d th e n c ho o se f rom th e rem ain in g on e s .

2.5 Practice exercise


1 W h a t is th e a ve ra ge o f po s itive in te ge rs f rom 1 to 100 in c lu s ive ?

( A) 49
( B ) 49. 5
( C ) 50
( D) 50. 5
( E ) 51

2 If
x+ y = 6, y + z = 7, and x+ z =9, w h at is th e ave rage of x, y an d z?
11 11 22
(C) ( D) 11 (E) 22
( A) 3 (B ) 2 3

3 In th e dia gra m be lo w , lin e s l an d m are not paralle l.

I f A re pre se n ts th e a ve ra ge m eas u re o f all th e e igh t an gle s , w h at is th e valu e


of A?
( A) A = 45
( B ) 45 < A < 90
( C ) A = 90
( D) 90 < A < 180
( E ) A = 180

Student Guide 53
4 A s la m h a s 4 tim e s a s m an y boo ks as Salm an an d 5 tim e s as man y as J ave d.
If J a ve d h as mo re th a n 40 boo ks , w h at is th e le as t n u m be r o f bo o ks th at
A s la m co u ld h a ve ?

( A) 200 ( B ) 205 ( C ) 210 ( D) 220 ( E ) 24


5 A s la m is no w 3 tim e s a s o ld as J ave d, bu t 5 ye ars ago, h e w as 5 tim e s as
J a ve d w a s. Ho w o ld is A s la m no w ?

( A) 10 ( B ) 12 ( C ) 24 ( D) 30 ( E ) 36

6 Ifx% of y is 10, wh a t is y?
10
(A)
x
100
(B)
x
1000
(C)
x
x
(D)
100
x
(E)
10

Answer Key
1 D
2 A
3 A
4 D
5 D
6 C

Student Guide 54
3 Analytical Ability
3 . 1. 1 Q ue st i o n f o r ma t

E a c h a n a lytic a l re a s o n in g que s tio n is a lo gic al pu zzle , base d o n a give n s e t of


c o n ditio n s . L ike m a th em a tic al que s tio n s , th e s e qu e s tio n s h ave e xac tly o ne
c o rre c t a n s we r, w h ic h is w h a t yo u n ee d to se le c t.

A n a lytic a l re a s on in g que s tio n s are pre s e n te d in gro u ps o f f ou r o r f ive


qu e s tio n s . E a c h gro u p is bas e d o n a s h o rt pas s age fo llo w e d by a se t of
c o n ditio n s . O c c a s io n a lly, th e re are graph s an d table s in s te ad o f a pas s age. To
u n de rs ta n d th e ge ne ra l fo rm a t o f th e qu es tio n , c o n s ide r th e fo llo w in g e xam ple .

Q ue st i o n 1 - 4 :
A s pa rt o f th e ir s po rts ph ys ic a l, se ve n co lle ge ath le te s  F, G, H , I , J, K an d L
 a re be in g we igh e d. I n a n no u n c in g th e re s u lts o f th e ph ys ic al e xam s , th e
c o a c h h a s give n th e fo llo w in g in fo rm atio n .
i. No n e o f th e a th le te s is e xac tly th e s am e we igh t as an o th e r ath le te .
ii. K is h e a vie r th a n L, bu t ligh te r th an H.
iii. I is he a vie r th a n J
iv. B o th F a n d G a re he a vie r th an H .
1 E a c h of th e f o llo w in g c o u ld be tru e E X C E PT
A. F is th e he a vie s t.
B. G is th e he a vie s t.
C. I is th e he a vie s t.
D. M o re th a n th re e a th le te s are h e avie r th an K.
E. M o re th a n th re e a th le te s are ligh te r th an K.
2 W h ic h o f th e fo llo w in g, if tru e , w ou ld be s u ff ic ie n t to de te rm in e w h ic h
a th le te is th e ligh te s t?
A. I is th e he a vie s t
B. I is ligh te r th a n K
C. K is h e a vie r th a n J
D. J is h e a vie r th a n K
E. E xa c tly f ive s tu de n ts a re ligh te r th an F .
3 I f J is h e a vie r th a n F , ho w m an y dif fe re n t ran kin gs by w e igh t, o f th e ath le te s
a re po s s ible ?
A. 1 B. 2 C. 3 D. 4 E. 5
4 I f H is he a vie r th a n I , w h ic h o f th e fo llo w in g CA NNO T be tru e ?
A. I ’s we igh t is e qu a l to th e ave rage o f F ’s we igh t an d G ’s we igh t.
B. I ’s we igh t is e qu a l to th e ave rage o f K’s w e igh t an d L’s we igh t
C. J ’s we igh t is e qu a l to th e ave rage o f K’s w e igh t an d L’s we igh t
D. J is th e se c o n d ligh te s t.

Answers:

1. E 2. D 3. C 4. A

3.1.2 How to attempt


• S im plif y th e in f o rm a tio n by u s in g abbre via tio n s an d s ym bo ls . Th e f irs t
s te p is to s trip a w a y a ll of th e e xc e s s ve rbiage f ro m th e give n pas s age
a n d th en to a bbre via te th e re m ain in g ke y wo rds w ith s in gle le tte rs . F o r
e xa m ple , in th e que s tio n s tate m e n t “f ive mu s ic ian s  a bas s is t, a
dru m me r, a gu ita ris t, a pian is t, an d a tru m pe te r  are pe rf o rm in g in a
ta le n t sh o w ”, yo u s ho u ld im me diate ly abbre viate th e m B , D, G, P an d T.
Yo u c a n u s e a bbre via te d le tte rs to re pre s en t a w h o le se n te n c e als o . Yo u
s h o u ld u s e s ym bo ls to re pre s e n t c o n ditio n s . Yo u m ay de ve lo p yo u r o w n
s ym bo lic c o n ve n tio n s fo r th is . Th e obje c tive is to c o n ve rt th e pro ble m
in to n o ta tio n s , so th a t, it is e as ily u n de rs tan dable . Th e f o llo w in g is a
ba s ic se t o f s ym bo ls , w h ic h are mo s t co mm o n ly u s e d.
‘A ’ re pre se n ts th e s ta te m e n t “A kbar is go in g”.
‘B ’ re pre s en ts th e s ta te m e n t “B abu r is go in g”.
S y mb o
Me a ni ng Examp l e s
l
~ No t ~A A kbar is no t go in g. O r yo u c an s ay,
“it is n o t th e c as e th at A kbar is
go in g”.
∧ An d A ∧ B A kbar an d Babu r are go in g.

A ∧ ~B A kbar is go in g an d Babu r is no t
go in g.
∨ Or A∨ B A kbar o r B abu r is go in g.

A ∨ ~B A kbar is go in g o r Babu r is no t go in g.
→ I f , th e n A →B I f A kbar is go in g th e n Babu r is go in g.

(A ∧ B) I f A kbar an d Babu r are go in g, th e n


→S Sale e m is go in g.

↔ I f a n d on ly if A ↔B B abu r is go in g, if an d o n ly if A kbar is
go in g.
• B ef o re le a rn in g th e ta c tic s to atte m pt an an alytic a l re as o n in g que s tio n ,
yo u mu s t be fa m ilia r w ith s om e bas ic lo gic f ac ts , w h ic h are e xplain e d in
th e fo llo w in g te xt. Co n s ide r A an d B are tw o s tate me n ts .
o A is tru e me a n s ~ A is f als e .
o ~ A is tru e m e an s A is f als e .
o (A ∧ B ) is tru e me an s bo th A an d B are tru e .
o ( A ∧ B ) is tru e me an s e ith e r A o r B o r bo th are f als e .
o ( A ∨ B ) is tru e me an s e ith e r A o r B o r bo th are tru e .
o ( A ∨ B ) is fa ls e m ean s bo th A an d B are fals e .
o ~ (A ∧ B) is e qu ivale n t to (~ A ∨ ~ B) .
o ~ (A ∨ B) is e qu ivale n t to (~ A ∧ ~ B) .
o If (A → B ) is tru e th e n
I f A is tru e B is als o tru e .
I f A is f a ls e B may be tru e o r f als e .
o If (A→ B ) is fa ls e th e n A is tru e an d B is f als e .
o ( A → B ) is equ iva le n t to ( ~B → ~A )
o ( A ↔ B) is tru e m ean s :
I f A is tru e B is tru e .
I f A is f a ls e B is f als e.
o (A ↔ B) is f a ls e me an s:
I f A is tru e B is f als e .
I f A is f a ls e B is tru e .
o (A ↔ B) is e qu ivale n t to [( A → B) ∧ (B → A )] .
• Yo u mu s t be fa m ilia r w ith th e mo s t co mm o n type s of an alytic a l
re a s o n in g qu e s tio n s . Th e f o llo w in g f ou r type s o c c u r mo re f re qu e n tly
th a n th e o th e rs , a n d w he n yo u s ee th e m, yo u s h o u ld im me diate ly kno w
w h a t yo u ne e d to do to an s we r th e m .
o W hi c h o f t he fo l l o wi ng c o ul d b e t r ue ? I f o n ly o ne of th e
a n s we r ch o ic e s co u ld be tru e , th en e ac h o f th e o the r fo u r c ho ic e s
m u s t be f a ls e ; th at is , e ac h o ne mu s t vio la te at le as t on e of th e
give n c o n ditio n s .

o W hi c h o f t he f o ll o wi ng must b e t r ue? S in ce o n ly on e o f th e
a n s we r c ho ic e s mu s t be tru e , th e n fo r eac h o f th e c ho ic e s , e ith e r
it is f a ls e o r it is po s s ibly ( bu t no t de f in ite ly) tru e . Yo u h ave to
c h oo s e o n ly th a t c h o ic e w h ic h is def in i te ly tru e .

o W hi c h o f t he f o ll o wi ng c anno t b e t r ue? S in c e o n ly on e of th e
a n s we r ch o ic e s c ann o t be tru e , th e n e ach o f th e o the r c h o ic e s
c o u ld be tru e . Th e c o rre c t ans w e r is th e o n ly ch o ic e , w h ic h
vio la te s a t le a s t on e o f th e give n c on dit io n s o r is o th e rw is e
in c o n s is te n t w ith w h at yo u kn o w m u s t be tru e .
o Ho w ma ny p o ssi b i li t i e s ar e t he r e? Th is que s tio n as ks , “Ho w
m a n y dif fe re n t w ays are th e re to s atis f y all of th e give n
c o n ditio n s ?” He re , yo u mu s t s ys te m atic al ly c o un t o r lis t all o f th e
po s s ibil it ie s th a t do n o t vio late an y of th e c o n ditio n s .

• I de n tif y th e ke y w o rds th at s e rve to lim i t th e s itu atio n . C e rtain w o rds


a re c ritic a l to yo u r u n de rs tan din g o f th e s itu atio n . B e su re to
in c o rpo ra te yo u r s ym bo ls . S om e f re que n tly u s e d ke y w o rds are lis te d
be lo w :

A f te r A ll A lw ays A t le as t A t mo s t
B ef o re But C an be C an n o t be C o n se c u tive
Dif f e re n t Dire c t ly E ac h No fe we r th an No mo re th an
O n ly Po s s ible E n tire E ve ry E xac tly
E xc e pt F ew e r F irs t If I f an d o n ly if
I mm edia te ly I mpos s ible Las t Le as t Most
M u s t be S am e S om e Th e Le as t Th e Mo s t
U n le s s S ma lle s t G re ate s t No n e

No te th a t ce rta in ke y w o rds h ave on ly o ne fu n c tio n , to ru le o u t a


po te n tia l a m bigu ity.

• E lim in a t in g th e c ho ic e s is alw ays a goo d s trate gy. W h ile e lim in atin g th e


c h o ic e s, f irs t of a ll, e lim in ate th os e w h ic h are ru le d o u t by in div idu al
c o n ditio n s : Th e n wo rk th ro u gh th e rem ain in g c h o ic e s.

• S tu dy c o n ditio n s , n o t me re ly f o r wh at th e y s tate bu t als o fo r w h at th e y


im ply. C e rta in a n a lytic a l re as o n in g qu es tio n s re se m ble th e in f e re n ce
qu e s tio n s yo u f in d in th e re adin g co m pre he n s io n se c tio n . To ans w e r
th e m co rre c tl y, yo u mu s t u n de rs tan d no t on ly w h at th e co n ditio n s s tate
e xplic i tl y, bu t a ls o wh a t th e y im ply.

• O f te n th e ke y to a n sw e rin g an alytic a l re aso n in g qu es tio n s is to o rgan ize


th e give n in f o rm a tio n in a lis t o r table .
• O n so me a n a lytic a l re a s o n in g qu es tio n s , an e xce lle n t w ay to de al w ith
th e in f o rm a tio n is to draw a s im ple diagram , pic tu re , o r map. Th is is
pa rtic u la r ly he lpf u l w h en yo u are de alin g w ith th e ph ys ic al o r tem po ral
o rde r of th in gs . I t is m uc h eas ie r to te ll wh e th e r pe rs o n A c an be
s e a te d o ppos ite pe rs o n B if yo u h ave s ke tc h e d a diagram o f th e table ; it
is e a s ie r to kno w w he th e r pe rso n C is o lde r o r yo un ge r th an pe rs o n D if
yo u h a ve e n te re d a ll o f th e give n in fo rm atio n o n a tim e lin e ; an d it is
e a s ie r to de te rm in e wh e th e r tow n E is e as t o f to wn W if yo u h ave draw n
a s im ple m a p.

3.1.3 Example questions with Answers and


Explanations

Questions 1-5:
S ix a c to rs - - - - B o b, C a ro l, Da ve E d, F ran k, an d G rac e au dit io n f o r a part in an
o ff - B ro a dw a y pla y. Th e a u dit io n s w ill take plac e o ve r f ou r co n s e c u tive days ,
s ta rt in g o n a Th u rs da y. E ac h ac to r w ill h ave on e au ditio n ; th e days on wh ic h
th e dif f e ren t a c to rs w ill a u ditio n mu s t c o nf o rm to th e fo llo w in g co n ditio n s .

i. A t le a s t o ne a u ditio n w ill take plac e e ac h day.


ii. No mo re th a n two au ditio n s w ill take plac e o n an y day.
iii. No mo re th a n th re e au dit io n s w ill take plac e o n an y tw o
c o n se c u tive da ys .
iv. B o b’s a u dit io n m u s t take plac e on S atu rday.
v. C a ro l’s a u ditio n m u s t take plac e on th e s am e day as an o th e r
a u dit io n .
vi. F ra n k’s a u ditio n s m u s t take plac e o n th e day be f o re G rac e ’s
a u dit io n .
vii. Da ve ’s a u dit io n m u s t take plac e on a day af te r E d’s au dit io n .

1 I f on ly o n e a u ditio n ta ke s plac e o n Th u rs day w h ic h ac to r co u ld h ave th at


a u dit io n ?
( A) B o b ( B ) C a ro l ( C ) Dave ( D) F ran k ( E ) G rac e

2 I f B o b’s a n d F ra n k’s a u dit io n s are o n th e s am e day, w h ic h o f th e fo llo w in g


m u s t be tru e
( A) Da ve ’s a u dit io n w ill ta ke plac e o n Th u rs day
( B ) Da ve ’s a u dit io n w ill ta ke plac e o n F riday
( C ) G ra c e ’s a u ditio n w ill ta ke plac e on Th u rs day
( D) C a ro l’s a u ditio n w ill ta ke plac e on Su n day
( E ) E d’s a u ditio n w ill ta ke plac e o n S u n day

3 I f th e dire c to r de c ide s to ho ld tw o au ditio n s o n Th u rs day an d tw o o n


S u n da y, h o w m a n y a c to rs wo u ld be e ligible to au ditio n on F riday?
( A) 1 (B) 2 (C) 3 ( D) 4 (E) 5

4 I f E d a n d G ra c e h a ve th e ir au dit io n s on th e s am e day w h ic h of th e f o llo w in g


m u s t be tru e ?
( A) E d’s a u ditio n w ill ta ke plac e o n Th u rs day.
( B ) F ra n k’s a u dit io n w ill ta ke plac e o n F riday.
( C ) C a ro l’s a u ditio n w ill ta ke plac e on Satu rday.
( D) G ra c e ’s a u ditio n w ill ta ke plac e on S atu rday.
( E ) C a ro l’s a u ditio n w ill ta ke plac e on Su n day.

5 I f E d’s a u ditio n is o n S atu rday, wh ic h o f th e fo llo w in g ac to rs c an no t


a u dit io n o n th e s a me da y a s an y o th e r ac to r?
( A) B o b ( B ) C a ro l (C) Ed ( D) F ran k ( E ) G rac e

Questions 6-10:
Du rin g th e f irs t h a lf of th e ye ar, f rom J an u ary th ro u gh J u ne , th e c h airpe rs o n o f
th e m a th em a tic s de pa rtm e n t w ill be o n s abbatic a l. Th e de an o f th e co lle ge h as
a s ke d e ac h o f th e s ix pro fe s s o rs in th e de partm e n t -- - A rke s , Bo ro f s ky, Ch an g,
De n tu re , H o bbe s , a n d Le e - -- to se rve as ac tin g c h airpe rs o n du rin g on e o f th o s e
m on th s . Th e ma th e m a tic ia n s c an de c ide th e o rde r in wh ic h th e y w ill s e rve ,
s u bj e c t on ly to th e f o llo w in g c rite r ia es tabl is h e d by th e de an .
i. C h a n g w ill se rve a s c h airpe rs o n in F e bru ary.
ii. A rke s w ill se rve a s c h airpe rs o n bef o re H o bbe s doe s .
iii. B o rof s ky a n d De xte r w ill s e rve as ch airpe rs o n s in co n s e cu tive
m on th s .

6 W h ic h o f th e fo llo w in g pro fe s s o rs co u ld s e rve as c h airpe rs o n in J anu ary?

(A) B o ro df s ky (B) Chang ( C ) De xte r ( D) H o bbe s ( E ) Le e

7 I n ho w ma n y w a ys c an th e s c he du le be m ade u p if Le e h as to s e rve as
c h a irpe rs o n in Ma y?
( A) 1
(B) 2
(C) 3
( D) 4
(E) 6

8 I f L e e s e rve s in Apri l, a ll o f th e fo llo w in g co u ld be tru e E XC E PT


( A) A rke s se rve s in J a n u a ry
( B ) H o bbe s se rve s in m a rc h
( C ) B o rof s ky s e rve s in ma y
( D) B o rof s ky s e rve s in J u ne
( E ) H o bbe s se rve s in J u n e
9 I f B o rof s ky s e rve s in M a y, wh at is th e late s t m on th in w h ic h A rke s c o u ld s e rve ?
( A) J a n u a ry
( B ) F e bru a ry
( C ) M a rc h
( D) A pril
(E) June

1 0 W h ic h o f th e fo llo w in g CA NNO T be tru e ?


( A) A rke s a n d L e e s e rve in co n s e c u tive m on th s .
( B ) L e e a n d Ho bbe s se rve in c o ns e c u tive m on th s .
( C ) H o bbe s a n d De xte r se rve in co n se c u tive mo n th s .
( D) A rke s a n d C h a n g se rve in c o n se c u tive mo n th s .
( E ) B o rof s ky a n d C h a n g se rve in c o ns e c u tive m on th s .

Solutions 1-5:
F irs t e xpre s s ea c h of th e c o n ditio n s s ym bo lic al ly:
B , C , D, E , F, a n d G: 1 a u ditio n eac h
Da ys : Th u , F ri, S a t, Su n
E a c h da y: 1 o r 2 a u ditio n s
2 c o n se c u tive da ys : 2 o r 3 au ditio n s
B =S a t Cx F <G E<D
1 A vio la te s th e c o n ditio n th at B o b’s au ditio n w ill take plac e on Satu rday
( B= S a t) . B vio la te s th e c o n ditio n th at C aro l’s au dit io n c an n o t be th e on ly
a u dit io n on a pa rtic u la r da y ( C  x) . C h o ic e s C an d E are im po s s ible . S in ce
Da ve ’s a u dit io n m u s t ta ke plac e on a day af te r E d’s au dit io n ( E < D) an d G rac e ’s
a u dit io n m u s t ta ke pla c e on a day af te r F ran k’s au ditio n (F < G ) n e ith e r c an
ta ke pla c e o n Th u rs da y. On ly c ho ic e D doe s no t vio late an y o f th e give n
c o n ditio n s , s o th is is th e co rre c t an sw e r.

2 Th e c o n ditio n th a t B o b’s a n d F ran k’s au ditio n s are on th e s ame day c om ple te ly


de te rm in e s th e s c h e du le . Th e y mu s t take plac e o n S atu rday (B = S at) . To
a vo id h a vin g m o re th a n th re e au ditio n s o n two co n s e c u tive days , th e re c an be
o n ly o ne a u ditio n on F rida y an d on e o n Su n day, wh ic h m e ans th e re w ill be tw o
o n Th u rs da y. S in ce F ra n k mu s t h ave to pre ce de G race ( F< G ) , G rac e ’s au ditio n
w ill ta ke pla c e on Su n da y. S in c e E d m u s t pre c e de Dave , E d’s au ditio n w ill take
pla c e o n Th u rs da y a n d Da ve ’s au ditio n on F riday. F in ally, C aro l’s au ditio n w ill
be th e s e co n d a u dit io n on Th u rs day. Th e f in al s c he du le is “C an d E on
Th u rs da y, D on F rida y, B a n d F o n S atu rday an d G on S un day”. O n ly c h o ic e B
is c o n s is te n t w ith th is s c h e du le , s o “B ” is th e c o rre c t c h o ic e .
3 S in c e o n ly o n e a u ditio n c a n take plac e on F riday, it c an n o t be C aro l’s ( C  x) ;
a n d, of co u rs e , it c a n n o t be B ob’s ( B = Sat) . An y o f th e o the r f ou r ac to rs
c o u ld a u ditio n on F rida y a s in dic ate d in th e f o llo w in g s c h e du le s :
E / F o n Th u , D o n F ri, B o n S at, C/ G on Su n
C / F o n Th u , E on F ri, B on Sat, D/ G on Su n
C / E on Th u , E o n F ri, B on Sat, D/ G on S un
E / F o n Th u , G on F ri, B o n Sat, C / D on Su n
S o th e co rre c t c ho ic e is D.

4 Th e on ly s c he du le th a t fu lf ils th e c on dit io n s is “F o n Th u , E / G o n F ri, B on S at,


a n d C / D on Su n ”. On ly c ho ic e E is co n s is te n t w ith th is s ch e du le .

5 S in c e E d a n d B o b’s a u ditio n s are bo th takin g plac e on S atu rday, e lim in ate


c h o ic e s A a n d C . S in c e C aro le m us t au ditio n on th e s ame day as ano th e r
a c to r, e lim in a te B. F in a lly, s in c e Dave ’s au ditio n m u s t take plac e o n Su n day
( E < D) , F ra n k’s a u ditio n m u s t take plac e o n Th u rs day an d G rac e ’s au dit io n on
F rida y ( F < G) . E lim in a te c h o ic e D. Th e co m ple te s c he du le is : “C / F o n Th u , G
o n F ri, B/ F on Sa t, a n d D on S un . ”
Solutions 6-10:
L e t A , B , C , D, H , L re pre s en ts prof e s so r n ame s .
C = Fe bru a ry, A <H , B << D a n d D< <B
6 O n ly ch o ic e E is th e re , wh ic h do e s n o t vio late an y of th e c o n ditio n s , s o is th e
c o rre c t c h o ic e .

7 W ith C s e rvin g in F ebru a ry a n d L in M ay, th e o n ly co n s ec u tive m on th s availab le


f o r B a n d D a re M a rc h a n d April. Th e n s in c e A mu s t se rve is be fo re H in J u ne .
Th e re a re two po s s ible s c he du le s , de pe n din g on th e o rde r of B an d D, s o th e
c o rre c t c h o ic e is B.

8 I f L s e rve s in April, th e co n s e c u tive m on th s availa ble fo r B an d D are M ay an d


J u n e; s o c h o ic e s C a n d D co u ld be tru e . S in c e A mu s t s e rve be fo re H , c h o ic e s
A a n d B mu s t be tru e , o n ly c h o ic e E c an no t be tru e .

9 S in c e A mu s t se rve be fo re H do e s, A c an n o t s e rve in J u ne . C an A se rve in


A pril? No, be c a u s e th e n, D w ou ld s e rve in J un e ( B< < D o r D<< B ) , an d again A
w ou ld n o t pre c e de H. Th e late s t th at A co u ld s e rve in M arc h , wh ic h c ou ld
occur in th e f in a l o rde r: L, C , A , D, B an d H .

10 Th e on ly pro fe s s o rs th a t c a n se rve in J an u ary are A an d L, so , on e of th em


m u s t se rve in J a n u a ry, a n d n e ith e r s e rve s in Fe bru ary. S o c h o ic e A c an n o t be
tru e .

.a Logical Reasoning
E a c h lo gic a l re a so n in g que s tio n re qu ire s yo u to an alyze an argu me n t pre s e n te d in
a s ho rt pa s s a ge . Of te n yo u a re as ke d e ith e r to f in d a c o n c lu s io n th at is a lo gic al
c o n se qu e n ce of th e pa s s a ge , o r to c ho o s e a s tate m e n t th at, if tru e , s tre n gth e n o r
w e a ke n s th e a rgu me n t.

3.2.1Question format
L o gic a l re a so n in g que s tio n s a re bas e d u po n a pas s age c alle d argu m e n t. Yo u h ave
to a n a lyze th e a rgu me n t pre se n te d in th e pas s age . Th e pas s age is f o llo we d by a
qu e s tio n . O c c a s io n a lly, th e re is m o re th an o ne que s tio n re late d be th e s am e
pa s s a ge . No ma tte r w h a t th e n u m be r is , th e qu e s tio n s alw ays aim at yo u r abili ty
to u n de rs ta n d th e co n c lu s io n re ac h e d by th e au th o r o f th e pas s age, an d to give
a rgu m e n t a n d co n tra a rgu m e n ts . Lo gic al re as o n in g qu e s tio n s are a lo t like re adin g
c om pre h e n s io n qu e s tio n s in a ve rbal se c tio n .

F o r ea c h lo gic a l re a s on in g que s tio n , th e argu me n t is fo llo w e d by a m u lti c h o ic e


qu e s tio n . Th e c h o ic e s a re s im ple s tate m en ts . Mo s tly th e qu es tio n s tate me n t
be gin s w ith th e ph ra s e “w h ic h of th e f o llo w in g s tate m e n ts ”. He re are a f ew
e xa m ple s :
• W h ic h o f th e fo llo w in g s tate me n ts is an as s u m ptio n on w h ic h th e c o n c lu s io n
o f th is a rgu me n t is ba se d?

• W h ic h o f th e fo llo w in g s tate me n ts ide n tif ie s a f law in th e re as o n in g o f th is


a rgu m e n t?

• W h ic h o f th e fo llo w in g s tate me n ts c an be m os t re aso n ably in fe rre d, f ro m


th e s ta te m e n ts in th e give n pas s age ?

• W h ic h o f th e fo llo w in g s tate me n ts , if tru e , wo u ld m os t se rio u s ly, we ake n


th e a rgu m en t of fe re d?

• W h ic h of th e f o llo w in g s tate me n ts , if tru e , w o u ld s tre n gth e n th e c o nc lu s io n


in th e pre c e din g a rgu m e n t?

• W h ic h of th e f o llo w in g s tate me n ts wo u ld be th e mo s t im po rtan t to kn o w to


e va lu a te th e a rgu m e n t give n in th e pre ce din g paragraph ? E ve ry lo gic al
re a s o n in g qu e s tio n do e s no t f it th is m o ld, bu t yo u s ho u ld try.
3.2.2 How to attempt

• W h ile a tte m ptin g lo gic a l re as o n in g qu e s tio n s , yo u s h o u ld re ad th e qu es tio n


s ta te m e n t be fo re re a din g th e argu me n t. Th e n yo u s ho u ld an alyze th e
a rgu m e n t pre s en te d in th e pas s age . Yo u m u s t kn ow w h at as pec t of th e
a rgu m e n t yo u a re to co n c e n trate o n , an d f oc u s on it. B y th is , yo u w ill no t
be un n e ce s s a ril y w a s tin g yo u r tim e .
• Yo u mu s t be a ble to s po t th e que s tio n type by re adin g th e que s tio n
s ta te m e n t. I f yo u do th is , you w ill be be tte r able to appro ac h th e argu m e n t
in h a n d. Th e f o llo w in g s ix c ate go rie s are th o s e w h ic h m os t c om mo n ly
o c c u r:

1 Assumption: Q ue s tio n s th at te s t yo u r abili ty to re co gn ize th e pre m is e s on


w h ic h a n a rgu me n t is ba s e d, of te n take th e f o llo w in g f o rm s :
o Th e c o n c lu s io n abo ve de pen ds on w h ic h o f th e f o llo w in g
a s s u m ptio n s ?
o Th e a u th o r o f th e pas s age abo ve m ake s w h ic h of th e fo llo w in g
a s s u m ptio n s ?
o I n th e pa s s a ge abo ve , th e au th o r ass u me s wh ic h of th e f o llo w in g
s ta te m e n t to be tru e ?

2 Inference: Q u e s tio n s , w h ic h te s t yo u r abil ity to go be yo n d th e au th o r’s


e xplic i t s ta te me n ts a n d se e w h at th es e s tate m e n ts im ply, m ay be w o rde d like
th e s e .
o I t c a n be in fe rre d f rom th e pas s age abo ve th at th e au th o r
be lie ve s th a t …
o W h ic h o f th e fo llo w in g is im plie d by th e pas s age abo ve ?
o F ro m th e in f o rm atio n abo ve , wh ic h of th e f o llo w in g is th e mo s t
re a s o n a ble in f e ren c e ?

3 Conclusion: Q u e s tio n s th at te s t yo u r abil it y to de te rm in e w h at c laim c an


lo gic a l ly be ma de o n th e ba s is of e vide n c e in th e pas s age abo ve ?
o I f th e s ta te m e n ts abo ve are tru e , wh ic h o f th e fo llo w in g in a
c o n c lu s io n th a t c an be pro pe rly draw n ?
o Th e s ta te m e n ts in th e pas s age , if tru e , be s t s u ppo rts w h ic h of th e
f o llo w in g c o n c lu s io n s ?

4 Central Point: Q u e s tio n s th at te s t yo u r abil it y to u n de rs tan d th e th ru s t


o f a n a rgu me n t.
o Th e s ta te m e n t s ite d abo ve c o n ve ys w h ic h o f th e fo llo w in g
pro po s itio n s ?
o Th e a u th o r o f th e pas s age abo ve argu e s th at…
o W h ic h o f th e fo llo w in g e xpre s se s th e po in t th e au th o r o f th e
pa s s a ge a bo ve m ake s ?

5 Support: Q ue s tio n s th at te s t yo u r abili ty to re co gn ize wh e th e r an


a s s e rtio n s u ppo rts o r u n de rm in e s an argu m en t.
o W h ic h of th e f o llo w in g, if tru e , be s t su ppo rts th e au th o r’s
c o n c lu s io n ?
o W h ic h o f th e fo llo w in g, if tru e , m os t w e ake ns th e au th o r’s
c o n c lu s io n ?

6 Argument Evaluation: Questions that test your ability to judge


an argument.
o W h ic h o f th e fo llo w in g ide n tif ie s a f law in th e s pe ake r’s
re a s o n in g?
o W h ic h o f th e fo llo w in g wo u ld be mo s t im po rtan t to kn o w w h e n
e va lu a tin g th e ac c u rac y o f th e argu m en t abo ve ?

• Do no t try to s kim th e pas s age , re ad eac h argu me n t c aref u lly. I t is no t


e no u gh to h a ve a ge ne ral ide a abo u t th e argu m e n t; yo u m u s t be able to
a n a lyze it ve ry c a ref u lly.

• Yo u m u s t f in d th e co n c lu s io n of th e argu m e n t, w h ic h th e au th o r c laim s to
h a ve re a c he d. Th a t m os t c om mo n s itu atio n s are as fo llo w s :
o Th e co n c lu s io n is th e las t s en te n c e of th e pas s age , o f te n s tart in g
by w o rds su c h as so , th e ref o re , th u s , he n c e , c o n se qu e n tly e tc.
o Th e co n c lu s io n is th e f irs t s en te n c e of th e pas s age fo llo w e d by
th e s u ppo rtin g e vide n ce .
o O c c a s io n a lly, th e co n c lu s io n is n o t pre s e n t in th e pas s age ; in th is
c a s e , th e que s tio n as ks yo u to ide n tif y th e c o nc lu s io n .

• Pa y pa rtic u la r a tte n tio n to s ign al w o rds su c h as acc o rdin gly, f o r th is


re a s o n , h e n ce , a lth o u gh , bu t, e xc e pt, in co n tras t, n e ve rth e le s s , u n like e tc .

• E lim in a t in g th e c ho ic e s is alw ays th e be s t s trate gy if yo u do no t kn o w wh at


th e c o rre c t a n s we r is . Th is pro c e s s w ill e lim in ate so me o bvio u s w ro n g
c h o ic e s. An d yo u w ill be able to make an e du c ate d gu e s s f ro m th e
re m a in in g on e s.

• E ve ry a rgu me n t is ba se d u po n c e rtain as s u m ptio n s made by th e au th o r. I f


a n a rgu m en t’s ba s ic pre m is e s are so u n d, th e argu m e n t is s tre n gth e n e d. I f
a n a rgu m e n t’s ba s ic pre m is e s are f law e d, th e argu m e n t is w eake n e d. I n
s u ppo rt qu es tio n s , wh e re yo u h ave to de c ide abo u t we ake n in g o r
s tre n gth e n in g th e que s tio n , pin po in t w h at th e argu m e n t as s um e s . Th e n
c om pa re th a t a s s um ptio n w ith th e an s w e r c ho ic e s . I f th e qu e s tio n as ks yo u
to f in d th e ch o ic e , wh ic h m os t s tre n gth e n s th e argu m en t, lo o k f o r th e
c h o ic e th a t is m os t in ke e pin g w ith th e argu m e n t’s bas ic as s u m ptio n . I f th e
qu e s tio n a s ks yo u to c ho o s e th e c ho ic e th at mo s t w e aken s th e argu me n t,
lo o k fo r th e a n s we r th a t c as ts th e mo s t do u bt on th at as s u m ptio n .

• S om e lo gic a l re a s o n in g qu es tio n s are e s se n tia lly m in i an alytic a l re as o n in g


qu e s tio n s , so , be f am ili a r w ith all o f th e im po rtan t lo gic al f ac ts an d apply
w he n e ve r ne e de d. E xa m ple qu e s tio n s w ith A n sw e rs an d E xplan atio n s

3.2.3Example Questions with Answers and Explanations

Questions 1-2:
Th e m ic ro w a ve o ve n h a s be c om e a s tan dard applian c e in m an y kitc h e n s , m ain ly
be c a u se it of fe rs a f a s t w a y of c o o kin g f oo d. Ye t, so me h om eo w ne rs be lie ve th at
th e o ve n s a re s till n o t c om ple te ly s afe . M ic ro w aves , th e re fo re , sh o u ld n o t be a
s ta n da rd a pplia n c e un ti l th e y h a ve bee n c are f u lly re se arc h e d an d te s te d.

1 W h ic h o f th e fo llo w in g, if tru e , w ou ld mo s t we ake n th e c o n c lu s io n of th e


pa s s a ge a bo ve ?

( A) H om eo w ne rs , o f te n pu rc h as e ite m s de s pite kn o w in g th e y may be u ns af e .


( B ) Th o s e ho m eo w ne rs in do u bt abou t m ic ro w ave s afe ty o u gh t n o t to pu rc h ase
m ic ro w a ve s .
( C ) R es e a rc h a n d te s tin g o f h om e applian c e s s e ldom re ve als s afe ty h azards .
( D) M ic ro w a ve s a re no t a s dan ge ro us as s te am iro n s , wh ic h are u s e d in almo s t
e ve ry h om e.
( E ) H om eo w ne rs o f te n pu rc h as e ite m s th at th e y do n o t n ee d.

2 W h ic h o ne o f th e fo llo w in g, if tru e , w ou ld mo s t s tre n gth e n th e c o n c lu s io n of


th e pa s s a ge a bo ve ?

( A) H om eo w ne rs o f te n do u bt th e adve rtis e d s af e ty o f all n ew applian c e s .


( B ) S pe e d of fo o d pre pa ra t io n is n o t th e on ly co n c e rn of to day’s ho me o w ne r.
( C ) M o de rn h om e ow n e rs h a ve m o re f ree tim e th an e ve r be fo re .
( D) F oo d pre pa ra t io n h a s be co me alm o s t a s c ie n ce , w ith m o re c om plic a te d an d
in vo lve d re c ipe s .
( E ) M a n y m ic ro w a ve o ve n s h ave be e n fo u n d to le ak radio ac t ive e le me n ts .

3 Ye a rs a go , a po ll co n c lu de d th at th e re are m o re te le vis io n s th an th e re are


ba th tu bs in A me ric a n h om e s . No do u bt th at f ac t rem ain s to day, e s pec ial ly in
ligh t of th e gro w in g po pu la ri ty of ho me c om pu te rs . Now , in additio n to ow n in g
te le vis io n s f o r e n te rta in m e n t, mo re an d m o re fam ilie s are pu rc h as in g TV
m on ito rs fo r u s e w ith a pe rs o n al c om pu te r. W e c an s afe ly gue s s th at th e re are
s til l m a n y mo re pe o ple s ta rin g at a pic tu re tu be th an s in gin g in th e s ho we r.
W h ic h o f th e fo llo w in g s ta te me n ts c an be in f e rre d f rom th is pas s age ?
( )A Pe rs o n a l c om pu te rs pro bably c o s t le s s th an in s tall in g a s ho we r o r bath tu b.
( ) B Pe o ple c a n w a s h th em s e lve s w ith o u t a tu b o r s h ow e r, bu t th e y c an n o t
w a tc h te le vis io n u n le s s th e y o wn a te le vis io n se t.
( ) C TV mo n ito rs w ill wo rk w ith pe rs o n al c om pu te rs in plac e of re gu lar c om pu te r
m on ito rs .
( ) D A s m a n y co m pu te rs a re so ld to day as te le vis io n s e ts a f ew ye ars ago.
( ) E M o re te le vis io n m on ito rs are no w u s e d w ith pe rso n al co m pu te rs th an are
u s e d to w a tc h co mm e rc ia l te le vis io n bro adc as ts .

4 S om e sc ie n tis ts h a ve pro po se d th at, ove r two h u n dre d m illio n ye ars ago , o ne


gia n t la n d m a s s , ra th e r th a n vario u s c o n tin e n ts an d is lan ds , co ve rs on e th ird
o f th e e a rth . L o n g be fo re th e re was an y h um an lif e , an d o ve r vas t pe rio ds o f
tim e , is la n ds a n d co n tin e n ts drif te d apart. Au s tral ia w as th e f irs t to se parate ,
w h ile S ou th A me ric a a n d A f ric a w e re late in s plitt in g apart. S om e is lan ds , of
c o u rs e, we re fo rm e d by vo lc an o e s an d w e re ne ve r part of th e gre at lan d m as s.

A ll th e f o llo w in g w ou ld s u ppo rt th e au th o r’s c laim E XC E PT

()A M a n y of th e pla n ts o f th e So u th Am e ric an rain f o re s ts are m arke dly s im ilar


to th o s e o f A f ric a n ra in fo re s ts .
()B A u s tra lia h a s m o re a n im als th at are no t fo u n d in an y o th e r c o n tin e n t th an
h a ve se ve ra l of th e m u c h large r c o n tin e n ts .
( ) C V o lc a n ic is la n ds like H aw aii h ave ec o s ys te ms ve ry dif f e re n t f ro m th o se o f
c o n tin e n ta l la n ds w ith th e s ame ave rage tem pe ratu re .
( ) D Th e pla n ts of s im ila r co n dit io n s in S ou th A me ric a h ave le s s in c om mo n w ith
th o s e o f A u s tra lia th a n w ith th o s e o f A s ia, Af ric a o r E u ro pe.
( ) E Th e prim i tive la n gu a ge s o f Au s trali a are un like th os e o f Af ric a, wh ic h
re s em ble s th o s e o f S ou th A me ric a.
5 E ve ry Sa tu rda y, Am ir h a s pizza f o r lu n c h an d th e n goe s to th e mo vie s .
I f th e s ta te m e n t a bo ve is tru e , w h ic h o f th e fo llo w in g s tate m e n ts m u s t als o be
tru e ?
1 I f it is no t S a tu rda y, th an A m ir is n o t h avin g pizza fo r lu n c h an d is no t
go in g to th e mo vie s .
2 I f A m ir h a s pizza fo r lu n c h an d th e n goe s to th e m o vie s , it is S atu rday.
3 I f A m ir h a s pizza fo r lu n c h , bu t do e s no t go to th e m ovie s , it is no t a
S a tu rda y.
( )A 1 o n ly
( ) B 2 o n ly
( ) C 3 o n ly
( ) D 1 a n d 2 on ly
( ) E 2 a n d 3 on ly

6 A n tif re e ze low e rs th e me ltin g po in t o f an y liqu i d to w h ic h it is adde d so th at


th e liqu id w ill no t f ree ze in c o ld we ath e r. I t is co mm o n ly u s e d to m ain tain th e
c oo lin g s ys te m in a u tom o bile radia to rs . O f c o u rs e, th e we ath e r m ay be c om e
s o co ld th a t e ve n a n tif re e ze is no t e ff e c tive , bu t s u c h a s e ve re c lim atic
c o n ditio n ra re ly oc c u rs in w e ll- trave le d plac e s .

W h ic h o f th e fo llo w in g c a n be de du c e d f ro m th e pas s age ?


( )A W e ll- tra ve le d pla c e s h a ve me an s of tran s po rtat io n o th e r th an au to mo bile s .
( ) B A n tif re e ze doe s n o t low e r th e me ltin g po in t o f ce rtain liqu ids in extre m e
c o n ditio n s .
( ) C S e ve re c lim a tic c o n ditio n s rare ly o c c u r.
( ) D I t is n o t of te n th a t m a n y trave le rs wh o u se an tif re e ze h ave th e ir c oo lin g
s ys te m f re e ze.
( ) E A n tif re e ze ra is e s th e m e ltin g po in t o f s om e liqu ids .

Solutions:
1 Th e c o n c lu s io n o f th e pa s s age is th at, be c au s e of s afe ty c o n ce rn s , m o re
re s e a rc h a n d te s tin g ou gh t to be do ne be fo re m ic ro w ave s be co me s tan dard
h o u se h o ld a pplia n c e s . I f , ho w e ve r, re s e arc h an d te s tin g are in e ff e c tive me an s
o f dis c e rn in g s a fe ty pro ble m s ( as c ho ic e C s ays ), th en re se arc h an d te s tin g
w ou ld be irre le va n t. Th is c rit ic is m s e rio u s ly w e ake n s th e co n c lu s io n . So
c h o ic e C is th e co rre c t a n sw e r.

2 I f m a n y m ic ro w a ve o ven s h ave be e n f o un d to le ak radio ac ti ve e le me n ts ( as


c h o ic e E s a ys ) , th e n th e c o n c lu s io n  th at m ic ro w ave s s ho u ld no t be s tan dard
a pplia n c e s u n til th e y a re mo re c are f u lly re s e arc he d an d te s te d is f u rth e r
s tre n gth e n e d be c a u s e mo re s afe ty c o nc e rn s n e e d to be addre s s e d. So , c ho ic e
E is th e co rre c t a n sw e r.

3 Th o u gh C ho ic e s A a n d B m ay we ll be tru e , th e y c an no t be in fe rre d f ro m th e
in f o rm a tio n in th e pa s s a ge . B u t c ho ic e C c an be in f e rre d s in c e, “m o re an d
m o re f am il ie s a re pu rc h a s in g TV m on ito rs fo r u s e w ith a pe rs o n al c om pu te r. ”
TV m on ito rs mu s t wo rk w ith th es e c om pu te rs , o the rw is e , peo ple wo u ld no t bu y
th e m fo r th a t pu rpo s e. Ch o ic e s D an d E m ay o r m ay n o t tru e , bu t th e y are no t
in f e re n ce s f ro m th e pa s s a ge , s im ply additio n al in f o rm atio n . S o, th e co rre c t
c h o ic e is C.

4 I f A u s tra li a wa s th e f irs t c o n tin e n t to s e parate , it wo u ld f o llo w th at its f lo ra


a n d fa u n a w o u ld de ve lo p in is o latio n o ve r a lo n ge r pe rio d of tim e . S im ilarl y,
w e m a y e xpe c t th e pla n ts an d an im al of So u th Am e ric a an d Af ric a th at
s e pa ra te d la te r, to be mo re alike . C ho ic e s A, B, an d D su ppo rt th e se ide as .
Th e se pa ra te l y de ve lo pe d is lan ds are dif f e re n t at is als o in ac c o rd w ith th e
pa s s a ge . H ow e ve r th e la n gu age s of all th e co n tin e n ts w ou ld h ave de ve lo pe d in
is o la t io n , s in ce ma n did n o t e vo lve u n til af te r th e bre ak- u p o f th e lan dm as s,
a n d it is s u rpris in g th a t A f ric an an d So u th Am e ric an lan gu age s are s im ilar.
H um a n like n e s s o r dif f e re n ce s are irre le van t to th e c laim s of th e pas s age . S o
c h o ic e E is th e co rre c t a n sw e r.

5 Th is lo gic a l re a s on in g qu e s tio n is ve ry eas y as so o n as yo u e xpre s s th e give n


s ta te m e n t s ym bo lic a l ly. “I f it is S atu rday, th e n A m ir h as Pizza an d go e s to
M o vie s ” tra n s la te s a s S → (P ∧ M ) . Th is is equ ivale n t to ~ ( P ∧ M ) →~ S ,
w h ic h is e qu iva le n t to (~ P ∨ ~ M ) →~ S . S o if e ith e r P o r M is fals e , th e n S is
f a ls e . Th e re fo re , 3 is tru e , ne ith e r 1 no r 2 are tru e . So , th e c o rre c t c h o ic e is
C.
6 C h o ic e D is th e c o rre c t a n s we r. S in ce s e ve re c lim atic c o n ditio n s rare ly oc c u r
in w e ll- tra ve le d pla c e s , it is no t ne c e ss ari ly tru e th at “I t is n o t of te n th at man y
tra ve le rs w ho u s e a n tif re e ze h ave th e ir c oo lin g s ys te m s f re e ze . ” C ho ic e A
m en tio n s o th e r m ea n s of tran s po rtat io n , w h ic h is n o t addre s se d in th e
pa s s a ge . C ho ic e B ref e rs to “ce rtain ” liqu ids .
4 Verbal Ability
Th e pu rpo s e o f th e Ve rba l Te s t is to e valu ate an d an alyze you r E n glis h
c om pre h e n s io n a n d u n de rs ta n din g to w ards th e lan gu age . Th e que s tio n s w ill be
ba s ic a l ly a s ke d to j u dge th e se n te n c e co m ple tio n , an alo gy an d c ritic a l re adin g
s kil ls . Th e qu e s tio n s of dif f e ren t type s i. e. abo u t se n te n c e co m ple tio n an d
a n a lo gy te s tin g w ill be a s ke d ran dom ly. Th e qu e s tio n s abo u t th e c rit ic al re adin g
h ow e ve r w ill be a s ke d se pa ra te l y.

.a About the Verbal Questions


A s a lre a dy dis c u s s e d, th is s ec tio n w ill co n s is t of th e f o llo w in g type s o f qu e s tio n s
i. e . s e n te nc e c om ple tio n , a n a lo gy an d th e c rit ic al re adin g. Th e de tail abo u t eac h
s e c tio n is a s be lo w ;

4.1.1 Sentence Completion


Th e qu es tio n s th a t c om e un de r th is c ate go ry are pro vide d w ith vario u s ch o ic e s .
Yo u a re a s ke d to co m ple te th e se n te n ce s by f ill in g in th e blan ks w ith th e m os t
s u ita ble c h o ic e.
Th e qu e s tio n s fo r s e n te n ce co m ple tio n c an be re late d to an y o f th e o the r are as o f
s tu dy i. e. s c ie n ce , lite ra tu re , h is to ry, geo graph y e tc bu t th e s u bje c t matte r w ou ld
n o t h in de r yo u r la n gu a ge a bili tie s . Yo u are as ke d to c om ple te th e se n te n c e w ith
th e u se o f co rre c t gra m m a r o r vo c abu lary.

Th e s e qu es tio n s try to de te rm in e yo u r abili ty to re c o gn ize th e c o rre c t se n te n c e


s tru c tu re , righ t gra m m a r a n d h o w yo u make th e c o rre c t c h o ic e of vo c abu lary.

Techniques for Sentence Completion

F o r th e s e n te n ce co m ple tio n a f ew c ho ic e s are give n th at c o u ld be s e le c te d fo r


c om ple tin g th e se n te n c e s . O n ly on e c ho ic e is c o rre c t ou t of th e s e ve ral c ho ic e s .
Yo u h a ve to c om ple te th e se n te n c e by s e le c tin g th e c o rre c t c h o ic e ac c o rdin g to
th e gra m m a r o r vo c a bu la ry. Fo r m akin g th e righ t c ho ic e yo u c an be ne f it f ro m th e
f o llo w in g te c h n iqu e s ;

• A f te r yo u re a d th e in c om ple te s en te n c e do no t loo k at th e c h o ic e s. Try to


th in k a bo u t th e c o rre c t an sw e r yo u rs e lf . I f yo u th in k th at yo u h ave
c om ple te d th e s en te n c e an d f o un d th e co rre c t c ho ic e yo u c an c on s u lt yo u r
lis t of c h o ic e s . I f th e an sw e r yo u th o u gh t matc h e s on e o f th e c ho ic e s
m en tio n e d in th e lis t th a t is m os t pro bably th e righ t c ho ic e to be marke d.
I f it doe s n o t ma tc h w ith th e c h o ic e yo u c an lo o k f o r a s yn o n ym
re pla c e m e n t. Th is ta c tic is ve ry h e lpf u l in f in din g th e righ t an sw e r, it
pre ve n ts yo u f ro m c o n fu s in g yo u rs e lf w ith th e w ro n g ch o ic e s .

• Do n o t se le c t th e c ho ic e h as tily. E ve n if yo u are s atis f ie d w ith yo u r c h o ic e


try to s u bs titu te it w ith th e o the r c ho ic e s s o th at yo u are m o re s atis f ie d
w ith yo u r de c is io n . So me tim e s th e o th e r c ho ic e f its m o re appro pria te ly to
th e se n te n c e .

• W h e n yo u a re a s ke d to co m ple te a se n te n c e, w h ic h h as tw o s pace s to be
f ille d in , try to pu t th e f irs t w o rd of e ve ry c h o ic e in th e f irs t blan k. No te
do w n th e c ho ic e th a t you f in d be s t. No w f o r th e s e co n d blan k try eve ry
s e co n d c ho ic e o f a ll c ho ic e s . No te th e c ho ic e th at yo u th in k is m os t
a ppro pria te . C he c k if th e two s e le c te d c h o ic e s are m atc h in g o n e o f th e
give n pa ir o f ch o ic e s . I f it doe s th e n s e le c t it as yo u r c o rre c t c ho ic e , if no t
th e n co n s ide r th is pa ir a s a w ro n g c ho ic e an d try w ith th e o th e r c h o ic e s.
• I f yo u f in d dif f ic u lt y in makin g s e n se ou t of c e rtain w o rds an d yo u are no t
ve ry f am ili a r w ith th e m yo u c an try to make a gue s s w ith re fe re n c e to th e
c o n te xt of th e s e n te n ce . Try to bre ak th e wo rd in to vario u s parts an d
a n a lyze its m e an in g e. g. if yo u do n o t kn ow th e me an in g of th e wo rd
“c ivi li za t io n ” bre a k it in to tw o i.e . ‘c ivil ize ’ an d ‘atio n ’ no w yo u may kn o w
th e m e a n in g o f c ivil ize a n d th ro u gh th e te rm ‘atio n ’ yo u c an m ake o u t th at
th e w o rd is a n o u n of c ivi li ze . I f yo u f in d th e wo rd u nf am ilia r w ith pre f ixe s
a n d su f f ixe s divide th e wo rd in to its parts e. g. pre re c o rdin g. Th is wo rd
c o n s is ts of bo th pre f ix a n d s u f f ix. Yo u c an bre ak th e w o rd like pre - re c o rd-
in g. He re yo u kn o w th a t pre m ean s be fo re , re c o rd m e ans to s to re an d - in g
is a te rm of co n tin u o u s te n s e. So yo u c an f in d th is bre ak u p of wo rds qu ite
h e lpf u l in m a kin g o u t th e righ t s en s e . I f no n e of th e te c h n iqu e wo rks try
m a kin g a gu e s s w ith ref e re n ce to th e c o n te xt.

• W h e n lo n g a n d co m ple x s e n te nc e s co n f u se yo u th e n try to bre ak th at


s e n te n ce in to s m a lle r m o re s e n te n ce s by re ph ras in g it. Af te r yo u divide it
c om pa re w ith th e o rigin a l se n te n c e to avo id an y m is in te rpre ta tio n . I f yo u
a re s a tis f ie d re a d th e s m alle r s e n te n ce s to ge t th e ide a m o re c le arly.

Example Questions

1. M u lta n ________ ___ a ve ry h o t c lim ate .


A. Has
B . H a ve
C . H a s be e n
D. W ith
2. O n e of th e le a s t e ff e c tive ways of s o rtin g in f o rm atio n is le arn in g
______ ___ it.
A . R epe a t
B . R e pe a tin g
C . To re pe a t
D. h ow re pe a t

3. S a lm a n f in is h e d________ __ tw o o f h is pu blis h e d co m pos it io n s be fo re h is


tw e lf th birth d a y.
A. W ritte n
B. W rit in g
C . To w rite
D. W ro te

4. S of ia ______ ____ co lle c t s tam ps , bu t n o w s h e h as o th e r in te re s ts .


A. U se d to
B. W a s u se d to
C . U s e d to be
D. Us in g to

5. A f te r pa s s in g th ro u gh a gre at trau m a o f he r hu s ban d’s de ath , s he


______ ____ h a rd to a c h ie ve me n tal re laxatio n .
A. S tru ggle d
B. S tru gglin g
C . S tru ggle
D. To s tru ggle

6. I n pa rtn e rs h ip w ith Pa kis tan , S ou th Ko re a ________ ____o n M o to r way.


A. He lpe d w o rke d
B. H e lpin g w o rk
C . H e lpe d w o rkin g
D. To h e lp w o rkin g

7. W e w ill wa it if yo u ______ ____ go .


A. W a n te d to
B. W a n t
C. W a n t to
D. W a n tin g to

8. I f I h a d mo re tim e I ________ ____ c h e c ke d m y pape r.


A. Wo u ld h a ve
B. Wo u ld
C . Wo u ld h a d
D. W il l h a ve

9. I th o u gh t th a t he _______ ____ c om in g to day.


A. H a s bee n
B. I s
C. Was
D. H a s

10. Th a t pro fe s s o r en j o ys te ac h in g an d _________ __.


A. W ritin g
B. W ritte n
C . To w rite
D. W rite

11. J u s t _______ ___ th e f ile s o n my table .


A. Let
B . L e a ve s
C . S ta y
D. L e a ve

12. Th a n k yo u f o r ______ ____ me yo u r boo k.


A. B o rro w in g
B. L e n din g
C. Bo rro we d
D. H a d le n t

13. ______ ____ __ dis c o ve ry of in s u lin , it was not po s s ible to tre at


dia be te s .
A. Prio r
B. Be fo re to th e
C. Prio r to th e
D. To prio r th e

14. Dis t ribu te th e h a n do u ts _________ __ th e c an didate s .


A. Be tw ee n
B. Am on g
C. To
D. I n

15. O n ly _________ w e re pre s e n t at th e se m in ar.


A. a f ew peo ple
B. a li tt le pe o ple
C. a fe w peo ple s
D. th e lit tle peo ple

Answer Key

1. A 9. C
2. C 10. A
3. B 11. D
4. A 12. B
5. A 13. C
6. C 14. B
7. C 15. A
8. A

4.1.2Analogy Questions

A n a lo gy me a n s s im ila ri ty in e xa m ple s o r de s c ribin g like n e s s be tw ee n tw o o r mo re


w o rds . Th e s e qu e s tio n s a s k th e re ade r to an alyze th e re latio n s h ip be tw e en tw o
w o rds in a pa ir a n d lo o k fo r a n o th e r s im ilar o r equ ivale n t pair o f w o rds . Yo u are
pro vide d w ith f ive o the r pa irs o f w o rds. Yo u are e xpec te d to m atc h th e o rigin a l
pa ir, w h ic h is give n in th e que s tio n w ith on e of th e pairs in th e give n c h o ic e s on
th e ba s e s o f s im ila r re la t io n s h ips be tw ee n th e m . Th is e xe rc is e o r s u c h qu e s tio n s
try to de te rm in e yo u r ba s ic u n de rs tan din g to w ards vo c abu lary an d yo u r abil it y to
re c o gn ize th e re la tio n s h ip be tw ee n wo rds . S om e que s tio n s may als o as k yo u to
s e le c t a s u ita ble a n to n ym f o r a give n w o rd.

Techniques for Analogy Questions


F o r th e a n a lo gy que s tio n s yo u c an f o llo w th e gu ide lin e s me n tio n e d be lo w ;

• Do n o t re a d th e c ho ic e s be f o re you h ave an alyze d th e re latio n s h ip be tw e e n


th e pa ir of wo rds , yo u rse lf . Try to u n de rs tan d th e w o rds m o re
a ppro pria te ly a n d th in k o n w h ic h bas is th e re latio n s h ip be twe e n th e w o rds
is fo rm e d. Af te r you re a c h a co n c lu s io n re ad th e give n c ho ic e s af te rw ards
to ge t a pro pe r m a tc h w ith an o th e r pair h avin g th e s ame re latio n s h ip.

• W h e n yo u f in d you rs e lf s tu c k w ith a wo rd of dif f ic u lt vo c abu lary, do no t fe e l


c o nf u s e d. Try to un de rs tan d its m ean in g ref e re n ce to th e c o n te xt o r if it is
s om e wh a t f am il ia r try to re m em be r w he re an d wh e n yo u h e ard th e w o rd
be fo re . I t c a n be a gre a t h e lp.

• S om e tim e s yo u f in d th a t th e re is m o re th an on e pair th at f its w e ll to th e


qu e s tio n a n d is a ppro pria te f o r th e c h o ic e, give th e o rigin a l pair a lit tle
m o re th o u gh t s o th a t yo u c an f u rth e r s tu dy th e re latio n s h ip be tw ee n th e
w o rds a n d n a rro w it dow n to a m o re dis tin c t on e . Af te r yo u h ave bee n
s u c c es s f u l in f in din g a c lo se r re latio n s h ip you c an no w s c ru tin i ze th e tw o
o th e r pa irs th a t c o nf u s e d yo u e arlie r. R e pe atin g th e s am e pro c e du re w ith
th e s e w o rds wo u ld pro ve u se f u l.
• Do n o t ge t c a u gh t u p by th e tric ks o f th e te s t m ake rs . S om e time s th e
qu e s tio n s a re pro vide d w ith ve ry tric ky an d do dgin g c ho ic e s th at m is gu ide
gre a tly. Try to th in k o f e ve ry c ho ic e m o re s pe c if ic al ly an d n arro w ly.

• I f yo u a re f am il ia r w ith th e parts of s pe ec h an d th e ir n atu re , it c an be


be ne f ic ia l in ma kin g a mo re se n s ible c h o ic e. Re me m be r if th e w o rds in th e
o rigin a l pa ir a re a n o un an d an adje c tive , th e co rre c t c h o ic e yo u make
s h o u ld a ls o co n ta in th e wo rds in th e s ame gramm atic al o rde r. O th e rw is e ,
yo u r ch o ic e is w ro n g. So , if yo u are c o nf u s e d w ith two pairs an d c an n o t
c h oo s e th e c o rre c t ch o ic e yo u c an e as ily lo o k at th e ir gramm atic al o rde r
a n d give pre f e ren c e to th e o ne , wh ic h m atc h e s th e o rigin al o n e.

• Exclude the choice from your consideration that you think is


incorrect, e.g. the choices that do not have the same grammatical
unit as of the original pair cannot match the original pair in anyway.
Spend more time on considering the more possible choices.

• You should know about the various kinds of analogies that are more
frequently asked. Some of the common analogy types are as follows;

i. Synonyms
Some words are linked together in a pair which means the same or
has a similar dictionary definition.e.g Pretty- Beautiful

ii. Describing Qualities


Some pairs have some words in which one word describes the other
word. Heavy- Rain

iii. Class and Member


Some pairs have words which are based on class and member basis
e.g. Electronics-Radio

iv. Antonyms
Some pairs consist of the words that are opposite to each other
e.g.
Love- Hate

v. Describing Intensity
Some pairs consist of the words in which one describes the
intensity of the other e.g.
Smile-Laughter

vi. Function
In some pairs a word describes the function of the other word e.g.
Pen-Write

vii. Manners
Some words in a speech describe the manners and behavior e.g.
Polite-Speech

viii. Worker- Workplace


Some pairs in a word describe the profession and its workplace e.g.
Doctor-Clinic

Example Questions

1. HEIGHT: MOUNTAIN

(A) Depth : Trench


(B) Shade : Tree
(C) Weight : Age
(D) Speed : Highway
(E) Mineral : Mine

2. OBLIVIOUS : AWARENESS

(A) Comatose : Consciousness


(B) Serene : Composure
(C) Erudite : Knowledge
(D) Adroit : Skill
(F) Invigorate : Energy

3. BELLWETHER : BAROMETER

(A) Proselyte : Spark plug


(B) Panhandler : Kill
(C) Embezzler : Abduct
(D) Cynosure : Magnet
(F) Morass : Catalyst

4. ACT : ACTION

(A) Therapy : Thermometer


(B) Oblivion : Obvious
(C) Liturgy : Literature
(D) Image : Imagine
(E) Bowl : Bowdlerize

5. BIBULOUS : DRINK

(A) Rapacious : Clothing


(B) Gluttonous : Food
(C) Altruistic : Money
(D) Vegetarian : Meat
(E) Controversy : Reconcile

6. SONG : RECITAL

(A) Author : Bibliography


(B) Episode : Series
(C) Coach : Team
(D) Dancer : Agile
(E) Poetry : Prose

7. HOUSE : BIG

(A) Home : Live


(B) School : Daily
(C) Water : Cold
(D) Clothes : Socks

8. ANIMAL : MONKEY

(A) Zebra : Giraffe


(B) Stationery: Pencil
(C) Book : Cap
(D) Tree : Wood

9. HEAVEY : LIGHT

(A) Fat : Thin


(B) Stupid : Idiot
(C) Rough : Surface
(D) Beautiful : Diary

Choose the lettered word or phrase that is most nearly opposite in


meaning to the word in capital letters.
10. DETER

(A) Twist
(B) Intimidate
(C) Encourage
(D) Straighten
(E) Supply

11. INDIGENOUS

(A) Modern
(B) Alien
(C) Ornamental
(D) Natural
(E) Conceal

12. THERAPEUTIC

(A) Curative
(B) Injurious
(C) Medicinal
(D) Practical
(E) Slander

13. QUIZZICAL

(A) Amused
(B) Unequivocal
(C) Contorted
(D) Dissimilar
(E) Lax

14. ANCILLARY

(A) Dependent
(B) Concomitant
(C) Appendix
(D) Primary
(E) Buffet

15. VIRTUOSO

(A) Wicked
(B) Dazzling
(C) Mediocre
(D) Honorable
(E) Bawdy

Answer Key

1. A 4. D 7. C 10. C 13. B
2. A 5. B 8. B 11. B 14. D
3. A 6. B 9. A 12. B 15. E
4.1.3 Critical Reading Questions

Q u e s tio n s re la te d to c rit ic a l re a din g try to ju dge yo u r re adin g s kills an d h ow yo u


u n de rs ta n d a n d in te rpre t wh a t yo u re ad. Th e pape r in c lu de s a fe w pas s age s th at
a s k a n s we rin g qu e s tio n s re la te d to th e pas s age .

Techniques for Critical Reading Exercises

There are a few techniques related to the Critical Reading Questions that
prove to be a good guideline for solving such questions.

• Do not read the questions before reading the whole passage. Try to
skim through the whole passage and then read the questions to look
for a more specific answer. Read the passage quickly with
understanding but do not panic. Try to analyze what the whole
passage is about and what the author really intends to convey.
While reading mark the lines where you think the passage carries the
most important points. These strategies would definitely help you
find the answers.

• When you find yourself stuck with a question, do not waste your time
on it and go ahead for the next questions. Sometimes, answering
other questions guide you about the earlier question. But, if you still
do not find the answer mark it for doing in the end more calmly,
having enough time to think.

• Try to familiarize yourself with the types of critical reading


questions. Once you know the nature of such questions, you will be
able to find the answers more quickly even when you are reading the
passage. The examples of some commonly asked questions are as
follows:

o Central Idea
Mostly, questions are asked to explain the central idea or main
theme of the whole passage, which analyzes how you skim
through it. Sometimes, the opening and closing lines can give you
a better clue about answering such questions properly.

o Specific Details
Sometimes to analyze your scanning abilities you are asked to
answer some specific details about the passage. Such questions
are about ‘when’, ‘where’, ‘which’ and ‘who’. You can get the
answers of this kind of questions from the area of the passage
which you marked in the first reading, where you think the most
important and informational remarks of the author lies.

o Making Inferences
Most of the questions ask you to infer from the passages, making
your opinion about what is said in the paragraph, implying
meaning and making your own point of view. These questions try
to assess your judgment; you must be clear in your mind about
what the author is referring to and then make your own opinion
according to your understanding and comprehension. Read and
think about all the choices and analyze each of it logically
according to your comprehension rather than the author’s point of
view.

o Meaning in Context
Some selected words from the passage are pointed out to explain
them with reference to the context to check your reading
comprehension. Sometimes the word that describes something in
a dictionary portrays it the other way when it appears in the
context. The test tries to judge your ability to make sense of the
word in the context.

o Author’s Approach
Some questions ask you to explain the mood in which the author
is writing whether it is sarcastic, humorous, witty, sad etc. When
you are asked questions like these you can look for certain
expressions, words, phrases or exclamations, which describe the
tone, mood or style of the author. The feelings of the writer are
mostly exhibited through choice of words. While answering these
questions read the message carefully observing particularly the
use of words.

o Title Selection
Some passages ask for selecting a title that best suits the
passage. Remember that the chosen title should not be narrowly
or broadly selected. Try to avoid choosing those titles that
describes only one or two paragraphs but the one, which is
applicable to the whole passage and portrays it best.
Example Questions
Passage I:
We are profoundly ignorant about the origins of language and have to
content ourselves with more or less plausible speculations. We do not even
know for certain when language arose, but it seems likely that it goes back
to the earliest history of man, perhaps half a million years. We have no
direct evidence, but it seems probable that speech arose at the same time
as tool making and the earliest forms of specifically human cooperation. In
the great Ice Ages of the Pleistocene period, our earliest human ancestors
established the Old Stone Age culture; they made flint tools and later tools
of bone, ivory, and antler; they made fire and cooked their food; they
hunted big game, often by methods that called for considerable cooperation
and coordination. As their material culture gradually improved, they
became artists and made carvings and engravings on bones and pebbles,
and wonderful paintings of animals on the walls of caves. It is difficult to
believe that the makers of these Paleolithic cultures lacked the power of
speech. It is a long step Admittedly, from the earliest flint weapons to the
splendid art of the late Old Stone Age: the first crude flints date back
perhaps to 500,000 B.C., while the finest achievements of Old Stone Age
man are later than 100,000 B.C.; and, in this period, we can envisage a
corresponding development of language, from the most primitive and
limited language of the earliest human groups to a fully developed
language in the flowering time of Old Stone Age culture.
How did language arise in the first place? There are many theories about
this, based on various types of indirect evidence, such as the language of
children, the language of primitive societies, the kinds of changes that
have taken place in languages in the course of recorded history, the
behavior of higher animals like chimpanzees, and the behavior of people
suffering from speech defects. These types of evidence may provide us
with useful pointers, but they all suffer from limitations, and must be
treated with caution. When we consider the language of children, we have
to remember that their situations are quite different from that of our
earliest human ancestors, because the child is growing up in an
environment where there is already a fully developed language, and is
surrounded by adults who use that language and are teaching it to him.
For example, it has been shown that the earliest words used by children
are mainly the names of things and people (“Doll,” “Spoon,” “Mummy”):
but, this does not prove that the earliest words of primitive man were also
the names of things and people. When the child learns the name of an
object, he may then use it to express his wishes or demands: “Doll!: often
means “Give me my doll!” Or “I’ve dropped my doll: pick it up for me!”;
the child is using language to get things done, and it is almost an accident
of adult teaching that the words used to formulate the child’s demands are
mainly nouns, instead of words like “Bring!”’ “Pick up!”; and so on.

1 The main idea of this excerpt is


A. to provide evidence of the origin of language.
B. to present the need for language.
C. to discuss how early man communicated.
D. to present the culture of early man.
E. to narrate the story of English.
F.
2 Theories of the origin of language include all of the following
EXCEPT
A. Changes occurring through the years.
B. The need to communicate.
C. Language of children.
D. The first man’s extensive vocabulary.
E. Communication among primitive men.

3 The purpose of the discussion of the word, “Doll,” is intended to


A. Trace the evolution of a noun.
B. Support the fact that naming things is most important.
C. Indicate how adults teach language to children.
D. Show the evolution of many meanings for one word.
E. Evince man’s multiple uses of single words

4 The implication of the author regarding the early elements of


language is that
A. There were specific real steps followed to develop our language.
B. Care must be exercised when exhuming what we consider the roots
of language.
C. We owe a debt of gratitude to the chimpanzee contribution.
D. Adults created language in order to instruct their children.
E. Language was fully developed by primitive man.
5 If we accept that primitive man existed for a very long period of
time without language, then we may assume that
A. Language is not necessary to man’s existence.
B. Language developed with the developing culture of primitives.
C. Primitives existed in total isolation from one another.
D. Children brought about a need for language.
E. Mankind was not intended to communicate.

6 After a reading of this article, one might infer that


A. Society creates problems with language.
B. Language is for adults to instruct children.
C. Society uses language to improve itself.
D. With the evolution of language came wisdom.
E. Language brings power.

Answer Key

1. A 2. D 3.C 4. B 5. B 6. E
COMMON QUERIES
5 About the General Test
Q What is the General Test generally about?

The NTS General Test is generally divided into three sections i.e. The
Verbal Section, Quantitative Section and the section of Analytical
Reasoning. The duration of these tests is 120 minutes which you can
yourself manage for each section, as there is no specific time distribution
for any.

The following time table informs of the number of questions in each section
as well as the marks given for that section:

No of
Test Sections Marks
Questions
Verbal 25 25
Analytical Reasoning 35 35
Quantitative 40 40

Q What Kind of Questions are asked?

The nature of the test is objective type, each section is based on Multiple
Choice Questions (MCQs), and every question will have at least two choices
or five choices at the most.

Q What are the Contents of the Test?

The General Test is generally divided into three sections i.e. The Verbal,
Quantitative and Analytical Reasoning. All the questions are based on
Multiple-Choice Question format.

The Verbal section of the test is based on completing sentences with the
appropriate words, comprehension exercises to assess the basic knowledge
about the vocabulary and grammar of the language. Questions basically
about sentence completion, analogy, finding antonyms, and critical reading
are asked.

In the Quantitative Section the NTS tries to measure your basic


mathematical skills and check your understanding of the elementary level
mathematics. The MCQs in this section are based on areas like Arithmetic,
Algebra and Geometry, data analysis etc.

In the Analytical Reasoning Section various objective type questions are


asked to analyze the capability of every student with regard to their
problem solving aptitude in their daily life activities and also the academic
interests. Questions asked are basically on Analytical Reasoning and
Logical Reasoning.

Q From Where Can I Get Help to Prepare for the Test?


NTS does not refer to one specific book for consultation during the test
preparation. However, the candidates can go through this Student Guide,
published by NTS, which is easily available in the market. It contains
sample question papers as well as examples of the types of questions you
may find in each segment. You can also practice by solving the sample
tests and the other questions found in this guide and the NTS website. It
would help you in becoming familiar with the nature of the test. Try
practicing the tests by keeping the allotted time in mind. Give yourself an
average of one minute to complete each question. It would help you pace
yourself through out the test. Do not leave any question unsolved. For
further measures you can consult materials easily available at any good
bookstore for TOEFL, SAT, GRE etc.

Q What are the General Guidelines to Attempt the Test?

This Student Guide by the NTS is the right source to be consulted. The
sample question papers for any of the three areas of study are a great help
for you. The test is composed on the same test pattern and the questions
are selected on the same level. You may also use additional books for your
preparation. If you answer 75% of the questions correctly, you will receive
an excellent score.

Q Can I Get My Registration Cancelled?

If you cannot take the test and want to cancel your registration, you
should inform the NTS Office in writing by sending an application or email
at least one week before the test date.
Q How Can I Get My Test Schedule or Test Centre Changed?

Once you have been allotted a Test Center and the time of the test, no
changes can be made possible to that schedule. However, you can ask NTS
by email and NTS may try to help you, if it finds it administratively
feasible.

Q Can I Appear for the Test More Than One Time?

Yes, if you want to improve your score and ranking, you can reappear in
the test whenever the new schedule is announced. However you have to
register again with full fee payment. You also have to wait when the next
schedule of the test is announced.

Q From Where Can I Get the Registration Forms for the General
Test (NAT)?

For the paper based tests, you can get a free of charge NTS Registration
Form, enclosed in its brochure, which is made available in the NTS’ allied
bank branches, the Front Office or Reception of the NTS or the allied
institutes of NTS. However, please ensure that you get these before the
last date of registration.

Q How Can I Register for NTS’ General Test?


After receiving the form, fill it appropriately. Submit it at the NTS Allied
Institute while submitting the NTS Registration Forms NTS Test Fee is
charged by the NTS Bank Branches/ Allied Institutes according to the
announced specifications.

Fill the form according to the instructions given at the back of it. Submit it
by paying the Test Fee in any branch of NTS allied
bank/institutes/organizations/NTS by email or post. You get a Test Entry
Coupon from the bank/institute, which shows you your Roll Number on it.

Q How Shall I Fill the Forms?

A few things must be clear to you before you fill in the Form. Any
incomplete or ambiguous information in the form may cause delay in the
registration process and you may not be able to take the test. For filling in
the forms correctly it is necessary to;

1. Use black ink and BLOCK LETTERS to fill in the form.


2. Write the appropriate test title, for which you want to register i.e.
General Test

3. Be careful while spelling your name.


4. Write the mailing address correctly where you can be contacted in
future.
5. Ensure that the Test Fee is submitted in the bank/Institute through
‘Challan Form’ attached with the Registration Form. After
submitting the fee, one copy of the ‘Challan’ Form must be attached
to this Registration Form and you will need to keep the other for
yourself. Do not forget to fill in the bank ‘Challan’ details in the
appropriate columns. If the ‘Challan’ Form is not attached with your
form, the Registration Form will not be accepted and no further
processing will be made on your request for registration.
6. Also indicate your choice of city, where you want your test center to
be. Your group and Test Center information will be intimated to you
well before the test day.
7. Specify the institute that you want your test score to be forwarded
to. You can get the list of Allied Institutes from the NTS’ website.
8. If handed over to the same bank where you have submitted the fee.
The bank will give you one part of this form as a Test Entry Coupon.
You have to show this coupon at the time of test. Without this, you
may not be allowed to enter the Test Center.

NOTE: These instructions are also given on the back of the Registration Form. The
General Test for some allied institutes/organizations may not need NTS Registration
Form.

Q How Can I Pay the Registration Fee for the Paper-based


Testing?

You can submit the Registration Fee through a Bank Voucher in an NTS
Authorized Bank along with your Registration Form. However the fee and
fee deposition procedure is subject to change from time to time. The exact
particulars are communicated on the announcement of the test schedule.
On occasions the Test Fee can also be submitted at the institutes where the
test is being conducted.

Q How to Submit the Form for the Paper-based Testing?

After you have completed the form according to the instructions, get it
submitted at the same branch of the bank/institute/organization from
where you collected your registration form. Do not forget to collect a
stamped and photographed Test Entry Coupon from the bank, which
ensures your identification on the Test Day. After receiving the forms, the
NTS Head office checks the essential requirements for NTS registration i.e.

i Three copies of the recent photographs


ii Candidate's name clearly written in the block letters
iii Candidate's father’s clearly written name in block letters.
iv Candidates postal address clearly written in block letters
v Phone number clearly written etc.

If any essential requirement is missing from the Registration Form, it is


clearly marked as rejected.

Note: The Registration Forms are at times differently printed for different
institutes/organizations. The instructions for those forms are available at
the back of the Registration Form.

Q How does NTS Confirm My Registration for the Paper-based


Testing?

After submitting the Registration Form at the bank/institute/organization,


you get your Test Entry Coupon, which indicates your Roll Number and also
serves as your Identification Slip. This slip confirms your registration.

In case of any confusion or problems with the registration process you may
contact NTS.

Q May I Register on the Day of Examination for the General


Test?

The registration of candidates on the day of the NTS Examination cannot be


entertained. The candidate should have got oneset registered before the
deadline, announced by NTS for the close of the registration process.

Q How Can I Get Help in Case of Any Problem During the


Registration Process?

If you have any confusion about the registration process you can clarify
this by calling the NTS Office or by sending an email.

Q How Can I Get the Test Schedule for the Test?

If you have any confusion about the Test Schedule you can obtain details
from the news media as well as NTS website.
Q How Can I Get My Test Entry Coupon for the Paper-based
Testing?

You are given your Test Entry Coupon when you submit your form. It
serves as your Identification Slip and also informs you about the location
of the center, Roll Number and other details.

Q What do I have to do with the Test Entry Coupon?

Take the Test Entry Coupon with you on the Test Day. At the Test Center
you are asked to show the Test Entry Coupon that was given to you after
you submitted your form there. The staff at the Test Center identifies you
with the photograph attached on the Coupon. Remember to attach a
photograph, which is not more than six months old. If you fail to show
your Test Entry Coupon to the administration on the test day or if your
photograph does not match your appearance, you may not be allowed to
enter the Test Center.

Q What Should I Take With Me to the Test Center for the Paper-
based Testing?

You are required to bring your own sharpened pencils and eraser. Besides
this, the candidate is also asked to show his/her Test Entry Coupon.

Q What Things Should I not bring with me in the Test Center for
Paper-based Testing?

Calculators, cell phones, books, booklets, any kind of paper or cheating


material are strictly prohibited inside of the Test Center. If any of these
items are found to be in your possession during the test you will be
disqualified from the examination.

Q What are the Rules and Regulations that are applied to me


inside the Test Center for the Paper-based Testing?

The test is taken on the day and at the time scheduled. You are asked to
observe punctuality.
1. You are assigned a seat by the invigilation staff.
2. Testing aids are not permitted to be taken along to the Test Center.
Items like calculators, pagers, cell phones, headphones, and any
paper or writing material, dictionaries etc are prohibited in the test
center.
3. You are not allowed to eat, drink, or use tobacco during the test.
4. If you do not show the Test Entry Coupon and your photograph does
not match, you will not be permitted to take the test.
5. No discussion or communication with your fellow candidates is
allowed during the test session.
6. You are not allowed to leave the test center without the permission
of the supervisor.
7. Under no circumstances can the test questions or any part of a test
be removed, reproduced or disclosed verbally, electronically, or
physically or by any other means to any person or entity.
8. Test centers do not have large waiting areas. Friends or relatives
who accompany you to the test center are not permitted to wait in
the near vicinity of the test center or contact you while you are
taking the test.
9. If you need to leave your seat at any time during the test, raise
your hand and ask the invigilator. Timing of the section does not
stop.
10. Repeated unscheduled breaks will be documented and reported to
NTS.
11. NTS reserves the right to videotape any or all of the testing
sessions.
12. Dress in such a way that you can adapt to any room temperature.
13. You cannot communicate with anyone (other than the test center
staff) while the test session is in progress or even during the
breaks.
14. You are not permitted to leave the test center vicinity during the
test session or breaks.
15. If at any time during the test you believe that you have a problem
with your test, or need the Invigilation Staff for any reason, raise
your hand to notify the Invigilation Staff.

Q On What Basis Can I be disqualified from the Test?

If you are found indulged in any of the following activities, you will be
disqualified from the test:

1. Arriving late to the test center.


2. Taking the test for someone else.
3. Failing to provide the Test Entry Coupon.
4. Using any objectionable material during the test.
5. Trying to create a disturbance inside the test center.
6. Showing ill behavior towards the invigilation staff.
7. Trying to communicate with fellow candidates while the test is in
progress.
8. Trying to tear pages from the question paper.
9. Showing indiscipline i.e. eating, smoking etc.
10. Leaving the test center without permission.

Q How Shall I Mark the Answer Sheet?

1. Fill in the particulars carefully like your Name, Father’s Name,


Enrolment Number and the Test Name i.e. General Test and other
particulars as mentioned on the answer sheet.
2. Read the directions carefully before you begin your test.
3. The invigilator announces the procedure for marking the answer
sheets. Listen carefully to the instructions. When you mark the
answer sheets, make sure that the space is completely filled in. You
can change the answer, by erasing the earlier choice neatly.
4. If a question is found with no answer or it has more than one
marked choice, it will not be considered for marking.
5. Completely fill in the corresponding circles on the answer sheet
using a lead pencil. Do not use ink, ball-pen or any other marking
material to mark the oval.
6. Filling more than one oval will render your answer incorrect.
7. Please fill in the oval completely and make no other marks on it.
Incorrect

Correct

Q What are the Other Test Taking Strategies for the General
Test?

1. The test is divided into three sections i.e. Verbal, Analytical


Reasoning and Quantitative. The directions given at the beginning
of every section will tell you the total number of questions and the
recommended time.
2. You are provided with a few blank pages in the test paper. You can
use them for your rough work.
3. Some formulas and helping tables are given at the end of the
question paper. You can take help from those for solving the
questions.
4. Keep a very careful track of time.
5. Do not panic while solving your paper. Give each section a specific
time period and try to complete every section on time.
6. If you find difficulty with some questions, do not waste your time by
thinking about them for a long time. Try finishing the easier
questions first and you can go back and finish the difficult questions
later.
7. Do not leave any question unanswered, if you are confused or have
no idea about a certain question, try making a guess for the right
answer.
8. Do not make any extra marks on the answer sheet. Computer might
register these and consider them as another marked choice.
Q If I am disqualified from the Test, Can I apply for it at a Later
Time?

If you are dismissed from the test session, you will be given a second
chance and will be allowed to reappear in the next testing session.

Q How can I Ask for Result Reporting?

You can ask for your result by requesting it from NTS, through a phone
call, fax or by sending an e-mail. You can also visit the web site of NTS to
check your results. The results will be sent to you by post on request. A
proper result would also be prepared for the institutes and the candidates
can check up their performances by contacting their respective institute.

Q How is My NTS Test Scored?

The papers are marked according to the international marking standards.


Each question carries one mark. Every question which is correctly
answered will be given one mark unless the computer finds an error with
your marking.

Q Is There Any Negative Marking While Scoring?

If the answer is wrong you will lose one mark but there is no negative
marking for an incorrect answer.
Q If I have already appeared in the Test, How Will the First
Score Affect My New Score?

If the candidate has appeared more than two times in an NTS test, his/her
highest score would be considered to be valid.

Q Is There Any Chance to Improve My Score?

If you are not satisfied with your test score and you want to improve your
grade you will be allowed to apply and appear in the test again.

Q What Should I do, if I Register and Then Can Not Take the
Test?

If you do not appear to take the test you will be given another chance to
take the test on the next scheduled test, but no refunds can be made.

Q Who Shall I Contact in Case of any Confusion?

If you have any confusions about the testing service, registration, test
execution process and the announcement of results etc you can call, mail
or fax the NTS Head office for the details.

Queries for Online General Test

Q. In What Ways is NTS General Test Different from Other NTS Tests?

The NTS General Tests are different from its other tests in many ways. Unlike the NTS
Subject tests, the General Tests aims to assess the verbal, quantitative and analytical
abilities that have been attained over a period of time and that are not necessarily
related to any specific field of study.

Q. How Can I Register for NTS-General Tests?

For the NTS, General Test Registration, you have to fill the NTS’ Online Registration
Form on the NTS’ website. Before you get registered online you have to pay the
registration fee as per the specified procedure for that test. You can fill the form online
at our present website and submit it. You will get e-mail from NTS after the submission
of the form, which indicates that your form has been received by NTS. NTS will confirm
your registration by sending you an email once the bank confirms the receipt of your
fee. This form is available when the NTS has announced its online test schedule.

Q. What happens when I submit my Online Registration Form?

After you submit your Online Registration Form you receive an email from NTS which
confirms the receipt of your registration request. NTS takes 3-5 day’s time to check up
the valid Bank Challan No from the bank. When NTS confirms the deposition of your fee
from the bank, it sends you the Admission Ticket which gives you your User ID, location
of the Test Center, Test Time etc.

Q. When do I receive confirmation of my final Registration Status?


After submitting your Online Registration Form, you receive the confirmation of your
Test Registration and some other information about the test in the form of an Admission
Ticket within 3-5 days, through an email by NTS. If your registration is not confirmed
during the given time you can Contact Us for inquiring about the reasons.

Q. How Can I Pay the Registration Fee?

For the NTS General Test, the Test Fee is to be deposited in the prescribed Bank
Branches or a draft to be sent to NTS or the allied institute as prescribed. The reference
of your Bank Deposit Slip Number (that you received from the bank after depositing the
fee) is necessary to be endorsed on your Online Registration Form for getting your
registration confirmed.

Q. Will NTS Confirm my Registration?

Yes, after receiving a completely and properly filled NTS Online Registration form, NTS
confirms your registration by sending you an email as soon as the bank confirms the
receipt of your registration.

Q. May I Register on the Day of Online Test?

No, for the NTS Online General Test, you have to get registered by the due date,
intimated on the NTS’ website or advertised through various newspapers prior to the
test.

Q. Where Shall I Contact if I Do Not Get my Registration Confirmed?

If NTS does not confirm your registration within a week’s time after you submit your
form, you may contact support@nts.org.pk by giving the reference of your Bank Deposit
Receipt Number and your other personal particulars.

Q. Can I Get My Registration Cancelled?

If you cannot take the test and want to cancel your registration, you should inform the
NTS Office in writing by sending an application or email at least one week before the
test date.

Q. How Can I Get the Test Schedule?

The Test Schedule that includes the Test Date, Test Time and the Venue of the test, is
sent to you by NTS through an email.

Q. How Can I Get My Test Schedule or Test Centre Changed?

Once you have been allotted a Test Center and the time of the test, no changes might
be possible to that schedule. However, you can ask NTS by email and NTS may try to
help you, if it finds feasible administratively.

Q. Can I Appear for the Test More Than One Time?

Yes, if you want to improve your score and ranking, you can reappear in the test
whenever the new schedule is announced. However you have to register again with full
fee payment. You also have to wait when the next schedule of the test is announced.
Q. Is There Any Chance to Improve My Score?

If you want to improve your score and ranking, you are allowed to apply and appear in
the test again. However, you have to register again, once the schedule is announced.

Q. What Computer Skills are Necessary for NTS Online General Tests?

For the NTS computer-based online General Tests, minimal computer skills are required.
The skills that are required include, using a mouse, entering and confirming a response,
changing a response, accessing the help function, and moving to the next question
although a help function is available throughout the test. These Test Instructions will
also be given to you before the test is given.

Q. How Often is the NTS Online General Test Conducted?

A series of computer-based General Tests is planned. NTS announces the


Schedule from time to time on its website www.nts.org.pk and through newspaper
advertisements.

Q. When Should I Arrive at the Test Center?

Try to arrive at the testing center at least 45 minutes before your scheduled Test Time
so that no delays are made during the check-in procedures and your identification.

Q. What Should I Take With Me to the Test Center?

Bring your ID card with you for your identification. No other items e.g. stationary,
books, calculators are required nor allowed to be taken inside the Test Centre.

Q. How Long Will I be at the Test Center?

Although the actual test takes 3 hours, you should be prepared to spend up to 4 hours
at the center. You will spend extra time on taking Test Instructions, getting seated, and
other administrative arrangements, etc.

Q. What are the important Things to be careful about for Online General Tests?

You should be particularly careful about a few things while taking the Online General
Test:

• Online testing does not allow you to come back and reattempt the question that you
have already marked.

• If you face any difficulty in logging on to your system or face any other related
problem you can ask for help from any lab assistant or invigilator.

Q. How to take NTS Online Test?

You will go through the following screen shots to learn how to take NTS Online test.

Step: 1
This is the first screen named as Candidate Login Screen. You will enter your Candidate ID and
Password provided to you by NTS
Step: 2
You will see Test Instructions’ Screen after you successfully login. Please read the instructions
carefully to avoid any confusion during the test. After reading the instructions, press Start Test
button on center bottom of the page.
Step: 3
After you click on the Start Test button your test starts and a page appears which shows your first
question of the test. Each question has various choices, if you know the answer you can select the
correct choice for your answer and press Next Question button. You can also add this question
in the Pass Box to answer it at some another stage. You will also find some additional information
about your test on this page. This information includes:

• Total Number of Questions in the Test


• Total Number of Questions Answered by you
• Total Number of Questions in the Pass box to be attempted later
Step: 4
If you place a question in Pass Box you will notice that the Questions in Pass Box field will
increase by one.
Step: 5
If you want to answer the questions in Pass Box, simply click on Questions in Pass Box link at
the top. It will take you on the following screen. Now click Answer this Question button for the
question you want to answer.
Step: 6
You have selected this question from Pass Box now select its answer and proceed for next
question.
Step: 7
You have attempted all questions and now this last screen will show your detailed result.

Wish You Good Luck with your Test.


National Testing Service

Q. How is NTS Online Test different from paper-based Test?

For the Paper-based Test,

• A pencil, eraser and a sharpener is required to attempt the paper-based test.


• You can make the changes in the answer that you have already marked.
• The announcement of results is delayed than the online tests.
• You have to fill in a separate answer sheet to mark your answers.

Where as for the Online Test,

• No stationery items are required to attempt online test.


• If you want to attempt a question at the end, you can place it in the Pass Box for
attempting later.
• You can not reattempt a question that you have already answered.
• As soon as you finish the test, the result is displayed on the screen.
• In case of a system failure during the test, you will have to log-in again and the
test will start from the same question where you had left. No information will be
lost while the system was unavailable.
Q. How Can I Ask for Result Reporting?

Your result is reported to you right after you finish your test. You are given the
certificate fifteen days after the conduct of the test of the last batch. If you still do not
get the result you can Contact Us. You can also visit the website of NTS to check your
result. The final result is sent to you by email.

Q. Is There Any Negative Marking While Scoring?

There is no negative marking for wrong answers. However the negative marking may be
activated if it is required by the allied institute or organization.

Q. What are the Rules and Regulations that apply to me in the Test Center
when taking the Online General Test?

Rules and Regulations:

• If you do not appear with the Identity Card (NIC) on the Test Center, you will not be
allowed to take the test.

• The test will be given on the day and at the scheduled time. You are asked to observe
punctuality. Arriving late at the center may disqualify you from taking the test.

• You are not allowed to bring any testing aids inside the test center. Nothing, except
the original Identity Card is required to be taken along.

• You are not allowed to smoke, eat or drink inside the test center.

• No discussion or any form of communication with the fellow candidates is allowed


during the testing session.

• You will also not be allowed to leave the test center without the permission of the
supervisor.

• Test centers do not have large waiting areas. Friends or relatives who accompany you
to the test center will not be permitted to wait in the test center or contact you while
you are taking the test.

• You will be required to sign the attendance sheet before and after the test session and
any time you leave or enter the premises where the test is being conducted.

• If you need to leave your seat at any time during the test (which shall only be allowed
in case of serious illness), raise your hand and ask the invigilator.

• Repeated unscheduled breaks will be documented and reported to NTS. NTS reserves
the right to videotape all or any of Testing Sessions and use it to determine any
misconduct, etc.

• If at any time during the test you believe that you have a problem with your test, or
need the Invigilation Staff for any reason, raise your hand to notify the Invigilation Staff.
NOTE: The rest of the queries regarding the test format, contents and other
procedures have almost the same answers as of the paper based tests, given
above.
DRILL TESTS
General

Note: The sample papers do not include quantitatively the same number
of questions as there would be in the actual papers. They are merely
meant to provide conceptual guidance to the users or prospective
candidates.
Drill Test I
I Choose the correct answer for each
question and shade the
Quantitative No. Of corresponding CIRCLE on the
10
Section Questions answer sheet

1. The number of degrees through which the hour hand of a clock moves in 2 hours and
12 minutes is

A. 66
B. 72
C. 732
D. 723
E. None of these

A B C D E

2. A cylindrical container has a diameter of 14 inches and a height of 6 inches. Since


one gallon equals 231 cubic inches, the capacity of the tank is approximately

A. 2-2/7 gallons
B. 4 gallons
C. 1-1/7 gallons
D. 2-2/7 gallons
E. None of these

A B C D E

3. A train running between two towns arrives at its destination 10 minutes late when it
goes 40 miles per hour and 16 minutes late when it goes 30 miles per hour. The
distance between the two towns is

A. 720 miles
B. 12 miles
C. 8-6/7 miles
D. 12-7/7 miles
E. None of these
A B C D E

4. If the base of a rectangle is increased by 30% and the altitude is decreased by 20%
the area is increased by

A. 25%
B. 10%
C. 5%
D. 1%
E. 4%
A B C D E
5. If the sum of the edges of a cube is 48 inches, the volume of the cube is

A. 512 inches
B. 96 cubic inches
C. 64 cubic inches
D. 698 cubic inches
E. None of these

A B C D E

6. A certain triangle has sides, which are, respectively, 6 inches, 8 inches, and 10
inches long. A rectangle equal in area to that of the triangle has a width of 3 inches. The
Perimeter of the rectangle, expressed in inches, is

A. 11
B. 16
C. 22
D. 23
E. 24

A B C D E

7. The cube of 1/3 is

A. 3/9
B. 3/27
C. 1/81
D. 1/27
E. 1/9

A B C D E

8. In general, the sum of the squares of two numbers is greater than twice the product
of the numbers. The pair of numbers for which this generalization is not valid is

A. 8,9
B. 9,9
C. 9,10
D. 9,8
E. 8,10

A B C D E

9. A piece of wire 132 inches long is bent successively in the shape of an equilateral
triangle, a square, a regular hexagon, a circle. The plane surface of the largest area is
included when the wire is bent into the shape of a

A. Circle
B. Square
C. Hexagon
D. Triangle
E. Line

A B C D E

10. If pencils are bought at 35 cents per dozen and sold at 3 for 10 cents the total
profit on 5 1/2 dozen is
A. 25 cents
B. 35 cents
C. 27 1/2 cents
D. 28 1/2 cents
E. 31 1/2 cents
A B C D E
0
II Choose the correct answer for
each question and shade the
No. Of
ANALYTICAL 20 corresponding CIRCLE in the
Questions
Section answer sheet

For question 1 to 4

Three adults—R, S, and V—will be traveling in a van with five children—F, H, J, L, and M.
The van has a driver's seat and one passenger seat in the front, and two benches behind
the front seats, one bench behind the first. Each bench has room for exactly three
people. Everyone must sit in a seat or on a bench, and seating is subject to the
following restrictions: An adult must sit on each bench. Either R or S must sit in the
driver's seat. J must sit immediately beside M.

1. Which of the following can sit in the front passenger seat?

A: J
B: L
C: R
D: S
E: V
A B C D E

2. Which of the following groups of three can sit together on a bench?

A: F, J, and M
B: F, J, and V
C: F, S, and V
D: H, L, and S
E: L, M, and R

A B C D E

3. If F sits immediately beside V, which of the following CANNOT be true?

A: J sits immediately beside S.


B: L sits immediately beside V.
C: H sits in the front passenger seat.
D: F sits on the same bench as H.
E: H sits on the same bench as R.
A B C D E

4. If S sits on a bench that is behind where J is sitting, which of the following must be
true?

A: H sits in a seat or on a bench that is in front of where M is sitting.


B: L sits in a seat or on a bench that is in front of where F is sitting.
C: F sits on the same bench as H.
D: L sits on the same bench as S
E: M sits on the same bench as V.

A B C D E

For question 5 to 9
The principal of a school is forming a committee. There are to be five members: three
teachers, chosen from Mr. J, Ms. K, Ms. L, Mr. M, and Mr. N; and two students, chosen
from O, P, Q, and R. The composition of the committee must conform to the following
conditions: Ms. J will serve only if R is also on the committee. Ms. L will not serve
unless Ms. K and O also serve. Neither Mr. M nor Mr. N will serve without the other. If
P serves, neither Q nor R can serve.

5. Which of the following is an acceptable committee?

A: J, L, M, N, O
B: K, L, N, O, P
C: K, M, N, O, R
D: L, M, N, O, R
E: M, N, O, P, Q

A B C D E

6. How many different committees could include Mr. J and Q?

A: 1
B: 2
C: 3
D: 4
E: 5

A B C D E

7. If Q and R are both on the committee, who else must be on the committee?

A: J
B: K
C: L
D: M
E: O

A B C D E

8. If M is not on the committee, each of the following must be on the committee EXCEPT

A: J
B: L
C: O
D: Q
E: R

A B C D E
9. In how many different ways can the principal select an acceptable committee?

A: Fewer than 3
B: 3
C: 5
D: 7
E: More than 7

A B C D E

For question 10 to 13

A contractor will build five houses in a certain town on a street that currently has no
houses on it. The contractor will select from seven different models of houses—T, U, V,
W, X, Y, and Z. The town's planning board has placed the following restrictions on the
contractor: No model can, be selected for more than one house. Either model W must
be selected or model Z must be selected, but both cannot be selected. If model Y is
selected, then model V must also be selected. If model U is selected, then model W
cannot be selected.

10. If model U is one of the models selected for the street, then which of the
following models must also be selected?

A: T
B: W
C: X
D: Y
E: Z

A B C D E

11. If T, U, and X are three of the models selected for the street, then which of the
following must be the other two models selected?

A: V and W
B: V and Y
C: V and Z
D: W and Y
E: Y and Z

A B C D E
12. Which of the following is an acceptable combination of models that can be
selected for the street?

A: T, U, V, X, Y
B: T, U, X, Y, Z
C: T, V, X, Y, Z
D: U, V, W. X, Y
E: V, W, X, Y, Z

A B C D E

13. If model Z is one model not selected for the street, then the other model NOT
selected must be which of the following?

A: T
B: U
C: V
D: W
E: X

A B C D E

For question 14 to 16

Seven children—F, J, K, M, R, S, and T—are eligible to enter a spelling contest. From


these seven, two teams must be formed, a red team and a green team, each team
consisting of exactly three of the children. No child can be selected for more than one
team. Team selection is subject to the following restrictions: If M is on the red team, K
must be selected for the green team. If F is on the red team, R, if selected, must be on
the green team. R cannot be on the same team as S. J cannot be on the same team as
K.

14. Which of the following can be the three members of the Red team?

A: F, J, and K
B: F, R, and T
C: J, K, and T
D: K, M, and R
E: M, R, and T

A B C D E

15. If M and F are both on the red team, the green team can consist of which of the
following?

A: J, K, and R
B: J, S, and T
C: K, R, and S
D: K, R, and T
E: R, S, and T

A B C D E
16. If M is on the red team, which of the following, if selected, must also be on the
red team?

A: F
B: J
C: R
D: S
E: T

A B C D E

For question 17 to 20

A mail carrier must deliver mail by making a stop at each of six buildings: K, L, M, O, P,
and S. Mail to be delivered are of two types, ordinary mail and priority mail. The
delivery of both types of mail is subject to the following conditions: Regardless of the
type of mail to be delivered, mail to P and mail to S must be delivered before mail to M
is delivered. Regardless of the type of mail to be delivered, mail to L and mail to K must
be delivered before mail to S is delivered. Mail to buildings receiving some priority mail
must be delivered, as far as the above conditions permit, before mail to buildings
receiving only ordinary mail.

17. If K is the only building receiving priority mail, which of the following lists the
buildings in an order, from first through sixth, in which they can receive their mail?

A: L, K, P, S, O, M
B: L, K, S, P, M, O
C: K, L, P, M, S, O
D: K, P, L, S, O, M
E: O, K, L, P, S, M

A B C D E

18. If L, M, and S are each receiving priority mail, which of the following lists the
buildings in an order, from first to sixth, in which they must receive their mail?

A: K, L, P, S, O, M
B: L, K, O, P, S, M
C: L, K, S, P, M, O
D: M, L, S, P, K, O
E: S, L, M, P, K, O
A B C D E
19. If the sequence of buildings to which mail is delivered is O, P, L, K, S, M and if S
is receiving priority mail, which of the following is a complete and accurate list of
buildings that must also be receiving priority mail?

A: O, L
B: O, P
C: P, L
D: P, M
E: O, P, L, K
A B C D E

20. If only one building is to receive priority mail, and, as a result, O can be no
earlier than fourth in the order of buildings, which of the following must be the building
receiving priority mail that day?

A: K
B: L
C: M
D: P
E: S
A B C D E
III Choose the correct answer for each
question and shade the
VERBAL No. Of corresponding CIRCLE in the
20
Section Questions answer sheet

Each sentence below has one or two blanks, each blank indicating that
something has been omitted. Beneath the sentence are five lettered words or
sets of words. Choose the word or set of words that, when inserted in the
sentence, best fits the meaning of the sentence as a whole.

1. Florence Nightingale was ___ in the development of modern medicine, ___ such
practices as sanitization of hospital wards and isolation of actively infected
patients.

A. a collaborator…rejecting
B. a maverick…protesting
C. an innovator…initiating
D. a pioneer…criticizing
E. an individualist…standardizing

A B C D E

2. As a journalist who works to overturn erroneous convictions, Griffin Nicholson


was opposed to the court ruling ___ appeals for inmates who might be ___ .

A. barring…culpable
B. curbing…exonerated
C. encouraging…innocent
D. scrutinizing…eligible
E. shielding…esteemed
A B C D E

3. Linda Greenhouse's articles for the New York Times are an outstanding example
of ___, capsulizing prose into a necessarily limited space.

A. Callousness
B. Brevity
C. Intuition
D. Propriety
E. Fortitude
A B C D E
4. Roberto Clement was seen as ___ during his life because of both his selflessness
on the baseball field and his humanitarian work in his native Nicaragua.

A. An individualist
B. a grandstander
C. a sybarite
D. an altruist
E. an opportunist

A B C D E

5. His habit of spending more than he earned left him in a state of perpetual------
but he------------hoping to see a more affluent day.

A. indigence: persevered in
B. confusion: compromised by
C. enervation: retaliated by
D. motion: responded

A B C D E

6. Known for his commitment to numerous worthy causes, the philanthropist


deserved------ for his-----------.
A. recognition: folly
B. blame: hypocrisy
C. reward: modesty
D. credit: altruism

A B C D E

7. You should --------this paragraph in order to make your essay more-----.

A. Delete, succinct
B. Enlarge, redundant
C. Remove, discursive
D. Revise, abstruse

A B C D E

8. A------glance pays-----attention to details.

A. Furtive: meticulous
B. Cursory: little
C. Cryptic: close
D. Keen: scanty

A B C D E

Each question below consists of a related pair of words or phrases, followed by


five lettered pairs of words or phrases. Select the lettered pair that best
expresses a relationship similar to that expressed in the original pair.
9. TIRADE: ABUSIVE

A. Diatribe: familial
B. Satire: pungent
C. Panegyric: laudatory
D. Eulogy: regretful

A B C D E

10. SOLDIER: REGIMENT

A. Colonel: martinet
B. Dancer: balletomane
C. Singer: chorus
D. Trooper: rifle

A B C D E

11. ASYLUM: SHELTER

A. Harbor: concealment
B. Palisade: display
C. Stronghold: defense
D. Cloister: storage
A B C D E

12. STATIC: MOVEMENT

A. Humdrum: excitement
B. Chronic: timeliness
C. Ecstatic: decay
D. Diligent: industry

A B C D E

13. INTEREST: FASCINATE

A. Vex: enrage
B. Vindicate: condemn
C. Regret: rue
D. Appall: bother

A B C D E
14. LAUREL: VICTOR

A. Chevrons: army
B. Oscar: movie star
C. Power: glory
D. blue ribbon: cooking

A B C D E

15. PRECEDENT: JUSTIFICATION

A. Kindness: obedience
B. Authority: sanction
C. Usage: submission
D. Tradition: novelty
A B C D E

16. IMPLICATE: COMPLICATE

A. Vitality: inevitable
B. Empathy: sympathy
C. Importune: construct
D. Imply: simplify

A B C D E

17. PERMEATE: RUEFUL

A. Truculent: merciful
B. Sadden: pitiful
C. Evaporate: mournful
D. Penetrate: sorrowful
A B C D E

18. ENERVATE: STRENGTHEN

A. Aver: attribute
B. Divert: turn
C. Apprise: appraise
D. Stultify: enliven

A B C D E

Read the passages and answer the questions given at the end:

Recent technological advances in manned undersea vehicles have overcome some of the
limitations of divers and diving equipment. Without vehicles, divers often become
sluggish and their mental concentration was limited. Because of undersea pressure that
affected their speech organs, communication among divers was difficult or impossible.
But today, most oceanographers make observations by the means of instruments that
are lowered into the ocean or from samples taken from the water direct observations of
the ocean floor are made not only by divers of more than seven miles and cruise at the
depth of fifteen thousand feet. Radio equipment buoys can be operated by remote
control in order to transmit information back to land based laboratories, including data
about water temperature, current and weather.
Some of mankind’s most serous problems, especially those concerning energy and food,
may be solved with the help of observations made by these undersea vehicles.

19. With what topic is the passage primarily concerned?

A. Recent technological advances.


B. Communication among divers.
C. Direct observation of the ocean floor
D. Undersea vehicles

A B C D E

20. Divers have problems in communicating underwater because?

A. The pressure affected their speech organs


B. The vehicles they used have not been perfected.
C. They did not pronounce clearly
D. The water destroyed their speech organs.
A B C D E
Drill Test II
Choose the correct answer for
I
No of each question and shade the
Quantitative 10
Questions corresponding CIRCLE in the
Section
answer sheet

1. A piece of wood 35 feet, 6 inches long was used to make 4 shelves of equal length. The
length of each shelf was
A. 9 feet, 1 1/2 inches
B. 8 feet, 10 1/2 inches
C. 7 feet, 10 1/2 inches
D. 7 feet, 1 1/2 inches
E. 6 feet, 8 1/2 inches
A B C D E

2. The tiles in the floor of a bathroom are 15/16 inch squares. The cement between the tiles
is 1/16 inch. There are 3240 individual tiles in this floor. The area of the floor is
A. 225 sq. yds.
B. 2.5 sq. yds.
C. 250 sq. ft.
D. 22.5 sq. yds
E. 225 sq. ft.

A B C D E

3. A man bought a TV set that was listed at $160. He was given successive discounts of
20% and 10%. The price he paid was
A. $129.60
B. $119.60
C. $118.20
D. $115.20
E. $112.00

A B C D E

4. Mr. Jones' income for a year is $15,000. He pays 15% of this in federal taxes and 10%
of the remainder in state taxes. How much is left?

A. $12,750
B. $9,750
C. $14,125
D. $13,500
E. $11,475

A B C D E

5. The radius of a circle which has a circumference equal to the perimeter of a hexagon
whose sides are each 22 inches long is closest in length to which one of the following?
A. 7
B. 21
C. 14
D. 28
E. 24

A B C D E
6. If a, is a multiple of 5 and b = 5a, which of the following could be the value of a + b?
I. 60 II. 100 III. 150
A. I only
B. III only
C. I and III only
D. II and III only
E. None of these
A B C D E

7. Which of the following expressions has the greatest value?

A. 4×4÷4+4
B. 4÷ 4×4+4
C. 4×4–4×4
D. 4÷4+4×4
E. 2÷2+2×2

A B C D E

8. If (a + 3) / 5 is an integer, what is remainder when ‘a’ is divided by 5?


A. 1
B. 2
C. 3
D. 4
E. 5
A B C D E
9. The integral part of logarithm is called
A. Characteristic
B. Mantissa
C. Solution
D. Root
E. None of these

A B C D E
10. On the y-axis, the x-coordinate is
A. 1
Β. ∞
C. zero
D. -∞
E. –1

A B C D E
II Choose the correct answer for
each question and shade the
ANALYTICAL No. Of corresponding CIRCLE in the
20
Section Questions answer sheet

For question 1 to 3

A volunteer uses a truck to pick up donations of unsold food and clothing from stores
and to deliver them to locations where they can be distributed. He drives only along
a certain network of roads. In the network there are two-way roads connecting each
of the following pairs of points: 1 with 2, 1 with 3, 1 with 5, 2 with 6, 3 with 7, 5
with 6, and 6 with 7. There are also one-way roads going from 2 to 4, from 3 to 2,
and from 4 to 3. There are no other roads in the network, and the roads in the
network do not intersect. To make a trip involving pickups and deliveries, the
volunteer always takes a route that for the whole trip passes through the fewest of
the points 1 through 7, counting a point twice if the volunteer passes through it
twice. The volunteer's home is at point 3. Donations can be picked up at a
supermarket at point 1, a clothing store at point 5, and a bakery at point 4.
Deliveries can be made as needed to a tutoring center at point 2, a distribution
center at point'6, and a shelter at point 7.

1. If the volunteer starts at the supermarket and next goes to the shelter, the first
intermediate point his route passes through must be

A: 2
B: 3
C: 5
D: 6
E: 7

A B C D E

2. If, starting from home, the volunteer is then to make pickups for the shelter at
the supermarket and the bakery (in either order), the first two intermediate
points on his route, beginning with the first, must be

A: 1 and 2
B: 1 and 3
C: 2 and 1
D: 2 and 4
E: 4 and 2

A B C D E
3. If, starting from the clothing store, the volunteer next is to pick up bread at
either the supermarket or the bakery (whichever stop makes his route go
through the fewest of the points) and then is to go to the shelter, the first two
points he reaches after the clothing store, beginning with the first, must be

A: 1 and 2
B: 1 and 3
C: 4 and 2
D: 6 and 2
E: 6 and 4

A B C D E

For question 4 to 5

There are seven cages next to each other in a zoo. The following is known about the
cages. Each cage has only one animal, which is either a monkey or a bear. There is
a monkey in each of the first and last cages. The cage in the middle has a bear. No
two adjacent cages have bears in them. The bear’s cage in the middle has two
monkey cages on either side. Each of the two other bear cages are between and
next to two monkey cages

4. How many cages have monkeys in them?

A: 2
B: 3
C: 4
D: 5
E: 6

A B C D E

5. The bear cage in the middle must have

A: No other bear cage to its left


B: No monkey cage on its right.
C: A bear cage to its left and to its right
D: Other bear cages next to it.
E: No monkey cage to its left.

A B C D E

For question 6 to 8

A nursery class in a school has a circular table with eleven seats around it. Five girls
(Kiran, Lado, Maryam, Omera and Parveen) and five boys (Farhan, Ghaus, Haris,
Imdad and Jahangir) are seated around the table. None of the girls are seated in a
seat adjacent to another girl. Kiran sits between Farhan and Ghaus, and next to
each of them. Jahangir does not sit next to Imdad.

6. Which of the following is a possible seating order around the table?

A: Empty seat, Farhan, Kiran, Ghaus, Lado, Omera, Haris, Imdad, Parveen, Jahangir,
and Maryam.
B: Empty seat, Farhan, Kiran, Ghaus, Lado, Jahangir, Parveen, Omera, Imdad,
Maryam, Haris.
C: Empty seat, Farhan, kiran, Ghaus, Omera, Jahangir, Parveen, Imdad, Maryam,
Haris, Lado.
D: Empty seat, Omera, Farhan, Kiran, Ghaus, Lado, Jahangir, Imdad, Parveen,
Haris, Maryam.
E: Empty seat, Maryam, Farhan, Kiran, Ghaus, Lado, Jahangir, Perveen, Imdad,
Omera, Haris.

A B C D E

7. If Lado, Haris, Maryam, Jahangir, and Ghaus are seated in that order, which of
the following is a correct completion of the seating order after Ghaus?

A : Kiran, Farhan, Omera, Imdad, Parveen, empty seat.


B : Kiran, Farhan, Imdad, Omera, empty seat, Parveen
C : Farhan, Parveen, Kiran, Imdad, Omera, empty seat.
D : Kiran, Farhan, Parveen, Imdad, empty seat, Omera.
E : Kiran, Farhan, Omera, empty seats, Parveen, Imdad.

A B C D E

8. If Jahangir leaves his seat and occupies the empty seat, his new seating position
would be between:

A : Farhan and Kiran


B : Maryam and Ghaus
C : Kiran and Ghaus
D : Imdad and Lado
E : Parveen and Lado

A B C D E

For question 9 to 11
Four telephone operators (Abid, Baqir, Chauhan, and Daud) each have to perform
duties at the telephone exchange on four different days, Thursday through Sunday.
The following is known about their duty schedule: Chauhan has his duty day before
Abid. Daud has his duty day later than Baqir.

9. Which of the following is a possible order of duty days for the four operators?

A: Chauhan, Daud, Abid and Baqir.


B: Daud, Chauhan, Abid, and Baqir.
C: Baqir, Chauhan, Daud and Abid.
D: Abid, Chauhan, Daud and Baqir.
E: Abid, Baqir, Daud and Chauhan.
A B C D E

10. If Chauhan has his duty day on Saturday, who must have his duty day on
Thursday?

A: Either Abid or Daud.


B: Daud
C: Abid
D: Either Baqir or Daud.
E: Baqir.

A B C D E

11. Each of the following is possible EXCEPT:

A: Chauhan has his duty on Thursday.


B: Baqir has his duty on Thursday.
C: Daud has his duty on Saturday.
D: Baqir has his duty on Sunday
E: Abid has his duty on Sunday.

A B C D E

For question 12 to 13
There are 12 seats facing the blackboard in a classroom, four seats
(A1, A2, A3 & A4) in that order are in row A, the first row from the blackboard.
Immediately behind row A is row B with four seats (B1, B2, B3 & B4) in that order.
Immediately behind row B, is the last row C with four seats (C1, C2, C3 & C4) in that
order. Six students attend the class the following is known about there seating
arrangement: Ejaz sits exactly in front of Comil, Seat A2 is always unoccupied Daud
does not sit next to Farhat, Gharuy sits in seat A4 Hamid does not sit in seat B4 All
the seats in row C always remain empty

12. If Daud sits in seat B3, then Farhat must sit in seat:

A: A3
B: A1
C: B4
D: B2
E: C2
A B C D E

13. Suppose that Hamid and Ejaz are sitting in seats A1 and A3 respectively, then it
CANNOT be true that seat:

A: B1 is occupied by Daud.
B: B2 is empty
C: B1 is empty
D: B3 is OCCUPIED BY Comil
E: B4 is empty

A B C D E

For question 14 to 17

The principal of a school is forming a committee. There are to be five members:


three teachers, chosen from Mr. J, Ms. K, Ms. L, Mr. M, and Mr. N; and two students,
chosen from O, P, Q, and R. The composition of the committee must conform to the
following conditions: Ms. J will serve only if R is also on the committee. Ms. L will
not serve unless Ms. K and O also serve. Neither Mr. M nor Mr. N will serve without
the other. If P serves, neither Q nor R can serve.

14. Which of the following is an acceptable committee?

A: J, L, M, N, O
B: K, L, N, O, P
C: K, M, N, O, R
D: L, M, N, O, R
E: M, N, O, P, Q

A B C D E

15. How many different committees could include Mr. J and Q?


A: 1
B: 2
C: 3
D: 4
E: 5

A B C D E

16. If Q and R are both on the committee, who else must be on the committee?
A: J
B: K
C: L
D: M
E: O

A B C D E

17. In how many different ways can the principal select an acceptable committee?
A: Fewer than 3
B: 3
C: 5
D: 7
E: More than 7

A B C D E
For question 18 to 20

Three adults—R, S, and V—will be traveling in a van with five children—F, H, J, L,


and M. The van has a driver's seat and one passenger seat in the front, and two
benches behind the front seats, one bench behind the other. Each bench has room
for exactly three people. Everyone must at in a seat or on a bench, and seating is
subject to the following restrictions: An adult must sit on each bench. Either R or S
must sit in the driver's seat. J must sit immediately beside M.

18. Which of the following can sit in the front passenger seat?
A: J
B: L
C: R
D: S
E: V

A B C D E

19. Which of the following groups of three can sit together on a bench?
A: F, J, and M
B: F, J, and V
C: F, S, and V
D: H, L, and S
E: L, M, and R

A B C D E

20. If F sits immediately beside V, which of the following CANNOT be true?


A: J sits immediately beside S.
B: L sits immediately beside V.
C: H sits in the front passenger seat.
D: F sits on the same bench as H.
E: H sits on the same bench as R.

A B C D E
III
Choose the correct answer for each
question and shade the corresponding
VERBAL No. Of
20 CIRCLE in the answer sheet
Section Questions

Each sentence below has one or two blanks; each blank indicates that
something has been omitted. Beneath the sentence are five lettered words or
sets of words. Choose the word or set of words that, when inserted in the
sentence, best fits the meaning of the sentence as a whole.

1. Surprisingly enough, it is more difficult to write about the--------than about


the---and strange.

A. specific, foreign
B. abstract, prosaic
C. commonplace, exotic
D. simple, routine

A B C D E

2. A-----response is one that is made with----------.

A. stupid, fear
B. speedy, alacrity
C. sure, slowness
D. harmful, grimaces

A B C D E

3. A----is a-------.

A. norm, standard
B. criterion, mistake
C. discipline, school
D. doctrine, follower

A B C D E

4. It is widely believed that a nuclear war could ____ enough smoke and dust to
block out the sun and freeze the earth.

A. billow
B. extinguish
C. generate
D. duplicate
E. decimate
A B C D E
5. Consumption of red meat has ____ because its fat content has become a
worrisome and ____ matter.

A. abated ... dubious


B. skyrocketed ... stressful
C. abounded ... divisive
D. stabilized ... newsworthy
E. declined ... controversial
A B C D E

6. It takes ____ character to ____ the extremities of the arctic region.

A. an unflappable ... sustain


B. a dictatorial ... brook
C. a Spartan ... negotiate
D. an inimitable ... resist
E. a nomadic ... espouse

A B C D E

7. Consumers refused to buy meat products from the company because of rumors
that the water supply at the meat processing plant was ______; the rumors,
however, were quite ______, with no hard evidence to back them up.

A. uninspected .. reckless
B. contaminated .. unsubstantiated
C. impure .. damaging
D. misdirected .. scandalous
E. unscrupulous .. vicious
A B C D E

8. Many kinds of harmful viruses are unhindered when passing through different
parts of the host organism; indeed, there are few organic substances which such
viruses’ cannot______.

A. undermine
B. disseminate
C. aerate
D. exterminate
E. perforate
A B C D E

9. Their conversation was unsettling, for the gravity of their topic contrasted so
oddly with the ______ of their tone

A. uniqueness
B. rapidity
C. lightness
D. precision
E. reverence

A B C D E
10. Throughout the animal kingdom, ____ bigger than the elephant.

A. whale is only the


B. only the whale is
C. is the whale only
D. only whale is the
E. whale is only

A B C D E

Each question below consists of a related pair of words or phrases, followed


by five lettered pairs of words or phrases. Select the lettered pair that best
expresses a relationship similar to that expressed in the original pair.

11. YAWN: BOREDOM ::

A. dream : sleep
B. anger : madness
C. smile : amusement
D. face : expression
E. impatience : rebellion

A B C D E

12. OBSTRUCTION : BUOY ::

A. construction : building
B. boy : girl
C. danger : red light
D. iceberg : titanic
E. arise : lay down

A B C D E

13. CONCERT : MUSIC ::

A. performance : artist
B. exhibition : art
C. play : actor
D. operetta : singer
E. rock : role
A B C D E

14. TEAMMATE : ADVERSARY ::

A. felon : criminal
B. enemy : associate
C. pacifier : agitator
D. winner : loser
E. friend : foe
A B C D E
For Question 15-20 read the following passage:

A popular theory explaining the evolution of the universe is known as the Big Bang
Model. According to the model at some time between twenty billion years ago, all
present matter and energy were compressed into a small ball only a few kilometers
in diameter. It was, in effect, an atom that contained in the form of pure energy all
of the components of the entire universe. Then, at a moment in time that
astronomers refer to as T = 0, the ball exploded, hurling the energy into space.
Expansion occurred. As the energy cooled most of it became matter in the form of
protons, neutrons and electrons. These original particles combined to form
hydrogen and helium and continued to expand. Matter formed into galaxies with
stars and planets.

15. Which sentence best summarizes this passage?

A. The big band theory does not account for the evolution of the universe
B. According to the Big Bang Model, an explosion caused the formation of the
universe
C. The universe is made of hydrogen and helium
D. The universe is more than ten billion years old
A B C D E
16. According to this passage when were the galaxies formed?

A. Ten Billion Years ago


B. Fifteen billion Years ago
C. At T = 0
D. Twenty billion years ago
A B C D E
17. The word “compressed” in the passage could best be replaced by

A. Excited
B. Balanced
C. Reduced
D. Controlled
A B C D E
18. It may be inferred that

A. Energy and matter are the same


B. Protons, neutrons, and electrons are not matter
C. Energy may be converted into matter
D. The galaxies stopped expanding as energy cooled

A B C D E

19. The word “it” in the passage refers to

A. Energy
B. Space
C. Expansion
D. Matter

A B C D E

20. The environment before the Big Bang is described as all the following EXCEPT

A. Compressed matter
B. Energy
C. All the components of the universe
D. Protons, electrons and neutrons

A B C D E
Drill Test III
I Choose the correct answer for each
question and shade the
Quantitative No. Of corresponding OVAL in the answer
20
Section Questions sheet

1. If the pattern of dots shown above is continued so that each row after Row One
contains 1 dot more than the row immediately above it, which row will contain 12
dots?

A. Seven
B. Eight
C. Nine
D. Ten
E. Twelve

A B C D E

2. Each of Steve's buckets has a capacity of 11 gallons, while each of Mark's


buckets can hold 8 gallons. How much more water in gallons can 7 of Steve's
bucket's hold compared to 7 of Mark's buckets?

A. 3
B. 7
C. 21
D. 24
E. 56
A B C D E

3. Two integers have a sum of 42 and a difference of 22. The greater of the two
integers is

A. 22
B. 25
C. 28
D. 31
E. 32

A B C D E

4. The average of five numbers is 34. If three of the numbers are 28, 30 and 32,
what is the average of the other two?

A. 40
B. 50
C. 60
D. 70
E. 80

A B C D E

5. In the figure above, what is the value of x?

A. 30
B. 40
C. 50
D. 80
E. 100

A B C D E

6. In a certain cake, two straight cuts (made along two different radii) succeed in
removing 4/15 of the total cake. What is the central angle in degrees of the
piece cut?

A. 26
B. 60
C. 85
D. 92
E. 96

A B C D E

7. If an equilateral triangle and a square have the same perimeter, what is the ratio
of the length of the sides of the equilateral triangle to the lengths of the sides of
the square?

A. 3:4
B. 4:3
C. 1:4
D. 1:3
E. 3:1
A B C D E
8. If 2 and 4 each divide q without remainder, which of the following must q divide
without remainder.

A. 1
B. 2
C. 4
D. 8
E. It cannot be determined from the information given.

A B C D E

9. The ratio of boys to girls in a certain classroom was 2 : 3. If boys represented


five more than one third of the class, how many people were there in the
classroom?

A. 15
B. 25
C. 30
D. 45
E. 75

A B C D E

10. Let xy = z, where x,y,z and nonzero numbers. If x is multiplied by 3 and z is


divided by 3, this is equivalent to multiplying y by

A. 1/9
B. 1/3
C. 1
D. 3
E. 9

A B C D E

11. If 5x = 3, then (5x + 3)² =

A. 0
B. 9
C. 18
D. 36
E. 81

A B C D E
12. If the postal charges for a package are 62 cents for the first five ounces and 8
cents for each additional ounce, what is the weight of a package for which the
charges are $1.66? (Assume there are 16 ounces in one pound)

A. 1.05 pounds
B. 1.1 pounds
C. 1.125 pounds
D. 1.25 pounds
E. 1.5 pounds
A B C D E

13. If m/n = .75, then what is 3m + 2n?

A. 0
B. 8
C. 14
D. 17
E. 24

A B C D E

14. Which is greater?

Column A Column B
(10/4) / (3/2) * (3/7) (3/4) * (10/7) / (3/2)

A. if the quantity in Column A is greater


B. if the quantity in Column B is greater
C. if the two quantities are equal
D. if there is no relationship between these two quantities
E. if the relationship cannot be determined from the information given

A B C D E

15. Which of the following has a graph that is symmetric to the x-axis

A. y = x
B. y = x2 + 3
C. y2 = x
D. y = x3 – x
E. y != x

A B C D E
16. The prime factors of 96 are:

A. 2 and 3
B. 6 and 8
C. 2, 3 and 4
D. 8 and 12
E. 3 and 9

A B C D E

17. 8 are what percent of the 6?

A. 1.25
B. 75
C. 125
D. 133.333
E. 150
A B C D E

18. 2- 3 =

A. -8
B. 8
C. -6
D. -1/8
E. 1/8

A B C D E

19. If x + 1 < 3x + 5, then

A. X > -2
B. x < -2
C. x = 0√√√√
D. x<2
E. x>2

A B C D E

20. Which of the numbers cannot be represented by a repeating decimal?

A. 11/9
B. 23/7
C. √3
D. 4 1/3
E. 2

A B C D E
Choose the correct answer for each
II
question and shade the
Analytical
No. of corresponding CIRCLE in the answer
Reasoning 15
Questions sheet
The office staff of the XYZ Corporation presently consists of three bookkeepers (L, M and
N ) and five secretaries (O, P, Q, R and S). Management is planning to open a new office
in another city sending three secretaries and two bookkeepers from the present staff. To
do so they plan to separate certain individuals who do not function well together. The
following guidelines were established to set up the new office:

(a) Bookkeepers L and N are constantly finding faults with one another therefore should
not be sent together to the new office.
(b) N and P function well alone but not as a team. They should be separated.
(c) O and R have not been on speaking terms for many months. They should not go
together.
(d) Since O and Q have been competing for a promotion, they should not be in one team.
Based on the information given above find the correct answers to the following Questions:
1. If M insists on staying back then how many combinations are possible?

A. 1
B. 2
C. 3
D. None

A B C D E

2. If L is to be moved as one of the bookkeepers, which of the following CANNOT be a


possible working unit?

A. LMOPS
B. LMPQS
C. LMORS
D. LMPRS

A B C D E

3. If N is sent to the new Office which member of the staff CANNOT be sent?

A. O
B. M
C. Q
D. R
A B C D E

4. If O is sent to the new office then which of the following is a possible team?

A. LMOPR
B. MNOQS
C. MNOPS
D. LMOPS
A B C D E

5. If both N and Q are moved to the new office, how many combinations are possible?

A. 2
B. 3
C. 4
D. 1

A B C D E

6. A map representing countries R, S, W, X, Y and Z is to be drawn. Adjacent countries


cannot have the same color in the map.
The countries adjacent to each other are as follows:

Each of R, S, X and Y is adjacent to W.


X is adjacent to Y.
Each of R and S is adjacent to Z.

If X is the same color as Z then it must be true that

A. W is a different color from any other country.


B. S is a different color from any other country.
C. X is the same color as Y.
D. S is the same color as X.

A B C D E

Two statements, labeled I. & II, follow each of the following questions. The
statements contain certain information. In the questions you do not actually
have to compute an answer, rather you have to decide whether the information
given in the statements I. and II. is sufficient to find a correct answer by using
basic mathematics and every day facts?

7. A long distance runner has just completed running 28 miles. How long did it take him to
finish the journey?

I. His record speed is 8.25 miles per hour.


II. His average speed through the journey was 8 miles per hour.

A. Statement I. ALONE is sufficient but II. ALONE is not sufficient to answer this
question.
B. Statement II. ALONE is sufficient but I. ALONE is not sufficient to answer this
question.
C. Statements I. and II. TOGETHER are sufficient to answer the question but
NEITHER of them is sufficient ALONE.
D. Statements I. and II. COMBINED are NOT sufficient to answer the question and
additional information is needed to find the correct answer.

A B C D E

8. Captain of the national cricket team has to be the most popular member of the team.
Who is the captain of Pakistan’s national cricket team?
I. Waqar is the best player on the team.
II. Waseem is the senior-most member.

A. Statement I. ALONE is sufficient but II. ALONE is not sufficient to answer this
question.
B. Statement II. ALONE is sufficient but I. ALONE is not sufficient to answer this
question.
C. Statements I. and II. TOGETHER are sufficient to answer the question but
NEITHER of them is sufficient ALONE.
D. Statements I. and II. COMBINED are NOT sufficient to answer the question and
additional information is needed to find the correct
answer.

A B C D E

9. In a BCE class at CIIT, 30 boys and 10 girls registered Calculus II. How many boys
passed the course?
I. 5 students could not pass.
II. There were 2 girls who obtained A grade.

A. Statement I. ALONE is sufficient but II. ALONE is not sufficient to answer this
question.
B. Statement II. ALONE is sufficient but I. ALONE is not sufficient to answer this
question.
C. Statements I. and II. TOGETHER are sufficient to answer the question but
NEITHER of them is sufficient ALONE.
D. Statements I. and II. COMBINED are NOT sufficient to answer the question and
additional information is needed to find the correct answer.

A B C D E

10. A horse ran 100 miles without stopping. What was its average speed in miles per hour?
I. The journey started at 8 PM and ended at 4 AM the following day.
II. The horse ran 20 miles per hour for the first 50 miles.

A. Statement I. ALONE is sufficient but II. ALONE is not sufficient to answer this
question.
B. Statement II. ALONE is sufficient but I. ALONE is not sufficient to answer this
question.
C. Statements I. and II. TOGETHER are sufficient to answer the question but
NEITHER of them is sufficient ALONE.
D. Statements I. and II. COMBINED are NOT sufficient to answer the question and
additional information is needed to find the correct answer.

A B C D E
11. How much time will computer a need to solve 150 problems?
I. The computer needs 50 seconds to solve one problem.
II. Computer never takes more than 60 seconds to solve a problem.

A. Statement I. ALONE is sufficient but II. ALONE is not sufficient to answer this
question.
B. Statement II. ALONE is sufficient but I. ALONE is not sufficient to answer this
question.
C. Statements I. and II. TOGETHER are sufficient to answer the question but
NEITHER of them is sufficient ALONE.
D. Statements I. and II. COMBINED are NOT sufficient to answer the question and
additional information is needed to find the correct answer.

A B C D E

12. How many pencils does Raheel have?

I. He bought two boxes each containing 10 pencils.


II. He lent two pencils to Khaleel

A. Statement I. ALONE is sufficient but II. ALONE is not sufficient to answer this
question.
B. Statement II. ALONE is sufficient but I. ALONE is not sufficient to answer this
question.
C. Statements I. and II. TOGETHER are sufficient to answer the question but
NEITHER of them is sufficient ALONE.
D. Statements I. and II. COMBINED are NOT sufficient to answer the question and
additional information is needed to find the correct answer.

A B C D E

13. In a certain farm there are 47 goats. How many large brown goats are there?

I. 16 of the goats are large.


II. There are 18 brown goats in the farm.

A. Statement I. ALONE is sufficient but II. ALONE is not sufficient to answer this
question.
B. Statement II. ALONE is sufficient but I. ALONE is not sufficient to answer this
question.
C. Statements I. and II. TOGETHER are sufficient to answer the question but
NEITHER of them is sufficient ALONE.
D. Statements I. and II. COMBINED are NOT sufficient to answer the question and
additional information is needed to find the correct answer.
A B C D E
14. Can there be more than 200 pictures in a 60-page book?

I. There is at least one picture in each page.


II. There are no more than 3 pictures in any page.

A. Statement I. ALONE is sufficient but II. ALONE is not sufficient to answer this
question.
B. Statement II. ALONE is sufficient but I. ALONE is not sufficient to answer this
question.
C. Statements I. and II. TOGETHER are sufficient to answer the question but
NEITHER of them is sufficient ALONE.
D. Statements I. and II. COMBINED are NOT sufficient to answer the question and
additional information is needed to find the correct answer.
A B C D E
15. If P > Q and R > S, then, P + R > Q + S. Is X > Y?
I. X + A > Y + B
II. A > B

A. Statement I. ALONE is sufficient but II. ALONE is not sufficient to answer this
question.
B. Statement II. ALONE is sufficient but I. ALONE is not sufficient to answer this
question.
C. Statements I. and II. TOGETHER are sufficient to answer the question but
NEITHER of them is sufficient ALONE.
D. Statements I. and II. COMBINED are NOT sufficient to answer the question and
additional information is needed to find the correct answer.

A B C D E
III Choose the correct answer for each
question and shade the corresponding
VERBAL CIRCLE in the answer sheet
Section No of
10
Questions

Each sentence below has one or two blanks, each blank indicates
that something has been omitted. Beneath the sentence are five
lettered words or sets of words. Choose the word or set of words
that, when inserted in the sentence, best fits the meaning of the
sentence as a whole.

1. Although its publicity has been ___, the film itself is intelligent, well-acted,
handsomely produced and altogether ___

A. tasteless … respectable
B. extensive … moderate
C. sophisticated … moderate
D. risqué … crude
E. perfect … spectacular
A B C D E

2. The Inuit natives of Alaska's North Slope worry that ___ oil exploration might
___their sensitive natural environment.

A. additional…assist
B. current…bolster
C. curtailed…shatter
D. unregulated…damage
E. controlled…reassess
A B C D E

3. Ants live in colonies based on ___; each member contributes to the good of all by
actively working with others in performing necessary tasks.

A. Heredity
B. Individualism
C. Cooperation
D. Reasoning
E. Instinct
A B C D E

Each question below consists of a related pair of words or phrases, followed


by five lettered pairs of words or phrases. Select the lettered pair that best
expresses a relationship similar to that expressed in the original pair.

4. STUDYING: LEARNING::
A. running : jumping
B. investigating : discovering
C. reading : writing
D. dancing : swimming
E. talking : listening
A B C D E

5. AFTERNOON : DUSK ::

A. breakfast : dinner
B. yesterday : tomorrow
C. Sunday : Saturday
D. night : dawn
E. arise : lay down

A B C D E

6. VIBRATION: SOUND :

A. gravity : pull
B. watercolor : paint
C. accident : death
D. worm : reptile
E. arrive : home
A B C D E

7. RUN : RACE ::

A. walk : pogo stick


B. swim : boat
C. fly : kite
D. sink : bottle
E. repair : automobile

A B C D E

Read the passages and answer the questions asked at its end.

Almost a century ago Alfred Binet, a gifted psychologist, was asked by the French
Ministry of Education to help determine who would experience difficulty in school. Given
the influx of provincials to the capital, along with immigrants of uncertain stock, Parisian
officials believed they needed to know who might not advance smoothly through the
system. Proceeding in an empirical manner, Binet posed many questions to youngsters
of different ages. He ascertained which questions when answered correctly predicted
success in school, and which questions when answered incorrectly foretold school
difficulties. The items that discriminated most clearly between the two groups became,
in effect, the first test of intelligence.

Binet is a hero to many psychologists. He was a keen observer, a careful scholar, an


inventive technologist. Perhaps even more important for his followers, he devised the
instrument that is often considered psychology's greatest success story. Millions of
people who have never heard Binet's name have had aspects of their fate influenced by
instrumentation that the French psychologist inspired. And thousands of
psychometricians — specialists in the measurement of psychological variables — earn
their living courtesy of Binet's invention.
Although it has prevailed over the long run, the psychologist's version of intelligence is
now facing its biggest threat. Many scholars and observers — and even some
iconoclastic psychologists — feel that intelligence is too important to be left to the
psychometricians. Experts are extending the breadth of the concept — proposing much
intelligence, including emotional intelligence and moral intelligence. They are
experimenting with new methods of ascertaining intelligence, including some that avoid
tests altogether in favor of direct measures of brain activity. They are forcing citizens
everywhere to confront a number of questions: What is intelligence? How ought it to be
assessed? And how do our notions of intelligence fit with what we value about human
beings? In short, experts are competing for the "ownership" of intelligence in the next
century.

8. According to the passage, which of the following is most similar to the "barometer"
developed by Binet?

A. The S.A.T. or other standardized college admission test.


B. The written portion of a driver's license test.
C. Open tryouts for a varsity athletic team
D. An electronic scan of brain-wave activity.
E. The trivia questions of a game show.
A B C D E

9. The author suggests which of the following about "citizens everywhere"?

A. They do not have a sufficiently accurate definition of intelligence to evaluate


recent scientific developments.
B. They stand to benefit from recent progress in the scientific assessment of
intelligence.
C. The experiments they are performing with new methods of intelligence
measurement are valuable and interesting.
D. They are at odds with the experts over who should have the right to define
"intelligence."
E. Traditionally they have not given careful consideration to some important
issues concerning intelligence.

A B C D E
10. As used in line # 8, "discriminated" most nearly means

A. equalized
B. predetermined
C. showed favoritism
D. displayed intolerance
E. distinguished

A B C D E
Drill Test IV
I Choose the correct answer for each
question and shade the
Quantitative No. Of corresponding CIRCLE in the
20
Section Questions answer sheet

1. If the length of BC is twice the length of AC, what are the coordinates of B where
A=(x,y)?

A. (x,2y)
B. (-x,2y)
C. (2x,y)
D. (-2x,y)
E. (-2x,2y)

A B C D E

2. The average of five numbers is 34. If three of the numbers are 28, 30 and 32, what
is the average of the other two?
A. 40
B. 50
C. 60
D. 70
E. 80
A B C D E
G

3. In the figure above, rectangle AEJL has been divided into 8 congruent squares with
each of the 8 squares having an area of 16. What is the length of
AE + MF + LG+ AL + BK + CJ + DH + EG?

A. 32
B. 44
C. 88
D. 128
E. 176

A B C D E

4. For any positive integer x, #x = x²/3 and &x = 9 /x. which of the following is an
expression for the product of #x and &x?

A. 3x
B. x
C. 1
D. x3/64
E. 27 x3
A B C D E

5. In a certain town, p gallons of gasoline are needed per month for each car. How
long will q gallons last at this rate given that there are r cars in town?

A. pr/q
B. qr/p
C. r/pq
D. q/pr
E. pqr

A B C D E
6. Let xy = z, where x,y,z are nonzero numbers. If x is multiplied by 3 and z is divided
by 3, this is equivalent to multiplying y by

A. 1/9
B. 1/3
C. 1
D. 3
E. 9

A B C D E

7. If x, y, and z are different positive odd integers and x + y + z = 11, what is the
greatest possible value of z?

A. 10
B. 9
C. 8
D. 7
E. 6

A B C D E

8. If the postal charges for a package are 62 cents for the first five ounces and 8 cents
for each additional ounce, what is the weight of a package for which the charges are
$1.66? (Assume there are 16 ounces in one pound)

A. 1.05 pounds
B. 1.1 pounds
C. 1.125 pounds
D. 1.25 pounds
E. 1.5 pounds

A B C D E

9. What fraction of two weeks is 24 minutes?

A. 1/120
B. 1/336
C. 1/840
D. 1/2880
E. 1/20160

A B C D E
10. If the vertices of a triangle are at (0,0), (-3, 4) and (3, 4), what is the area of the
triangle?

A. 4
B. 6
C. 12
D. 14
E. 18

A B C D E

11. A water-tank has a base with dimensions 2 feet by 6 feet. If a cube with each side 1
foot is totally immersed in the water, how many inches will the water rise? (12inches
= 1 foot)

A. 1
B. 2
C. 4
D. 8
E. It cannot be determined from the information given

A B C D E

12. In the figure above, the quadrilateral ABCD is a trapezoid with x = 2. The diameter
of each semicircle is a side of the trapezoid. What is the sum of the lengths of the
four drawn semicircles? (Round to the nearest whole number.)

A. 13
B. 16
C. 19
D. 22
E. 31

A B C D E
13. If n + 3 = n x 3, then n =

A. 0.5
B. 1.5
C. 2
D. 2.5
E. 3
A B C D E

14. If an equilateral triangle and a square have the same perimeter, what is the ratio of
the length of the sides of the equilateral triangle to the lengths of the sides of the
square?

A. 3:4
B. 4:3
C. 1:4
D. 1:3
E. 3:1

A B C D E

15. A restaurant has a special whereby both parents can eat for $20 and each child can
eat for $5. Assuming a family group consists of both parents and at least one child,
what is the maximum number of family groups that could have attended given that
the restaurant took $115?

A. 6
B. 5
C. 4
D. 3
E. 2

A B C D E

16. Which of the following points lays in the interior of the circle whose radius is 10 and
whose center is at the origin?

A. (-9, 4)
B. (5, -19)
C. (0, -10)
D. (10, -1)
E. (0,15)

A B C D E
II Choose the correct answer for each
question and shade the
Analytical No. Of corresponding CIRCLE in the answer
15
Reasoning Questions sheet
17. If the perimeter of the rectangle ABCD is 14, what is the perimeter of ∆BCD?

7
A B
12
7 + √29
86 D C
It cannot be determined from the
information given.

A B C D E

18. The roots of ax2 + bx + c = 0 are real only if

A. b2 – 4ac ≥ 0
B. b2 – 4ac = 0
C. b2 + 4ac = 0
D. b2 – 4ac < 0
E. b2 – 4bc < 0

A B C D E

19. The two numbers, whose sum is -13 and product -30, are

A. 2, 15
B. 2, -15
C. -3, 10
D. 3, 10
E. -3, -13
A B C D E

20. Let A = total area of five circles of radius r and let B = total area of three circles of
radius s. If A = B, then r / s =

A. 3/5
B. √3 / √5
C. 3π / 5
D. √(3π) / √5
E. 3π

A B C D E
Two statements labeled I & II, follow each of the following questions. The statements contain
certain information. In the questions you do not actually have to compute an answer, rather you
have to decide whether the information given in the statements I. and II. is sufficient to find a
correct answer by using basic mathematics and every day facts?

1. What day of the week is today?

I. Today is March 25.


II. Akram left Pakistan on Wednesday.

A. Statement I. ALONE is sufficient but II. ALONE is not sufficient to answer this
question.
B. Statement II. ALONE is sufficient but I. ALONE is not sufficient to answer this
question.
C. Statements I. and II. TOGETHER are sufficient to answer the question but
NEITHER of them is sufficient ALONE.
D. Statements I. and II. COMBINED are NOT sufficient to answer the question and
additional information is needed to find the correct answer.

A B C D E

2. Can any of the four rivers be more than 300 meters wide?

I. The narrowest of the four rivers is 240 meters wide.


II. Average width of the four rivers is 300 meters.

A. Statement I. ALONE is sufficient but II. ALONE is not sufficient to answer this
question.
B. Statement II. ALONE is sufficient but I. ALONE is not sufficient to answer this
question.
C. Statements I. and II. TOGETHER are sufficient to answer the question but
NEITHER of them is sufficient ALONE.
D. Statements I. and II. COMBINED are NOT sufficient to answer the question and
additional information is needed to find the correct answer

A B C D E

3. If it is raining then there must be clouds. Are there clouds?

I. It is not raining.
II. It rained yesterday.

A. Statement I. ALONE is sufficient but II. ALONE is not sufficient to answer this
question.
B. Statement II. ALONE is sufficient but I. ALONE is not sufficient to answer this
question.
C. Statements I. and II. TOGETHER are sufficient to answer the question but
NEITHER of them is sufficient ALONE.
D. Statements I. and II. COMBINED are NOT sufficient to answer the question and
additional information is needed to find the correct answer.

A B C D E

Read the passage to answer the question 4-5


A map representing countries R, S, W, X, Y and Z is to be drawn. Adjacent countries
cannot have the same color in the map. The countries adjacent to each other are as
follows:
Each of R, S, X and Y is adjacent to W.
X is adjacent to Y.
Each of R and S is adjacent to Z.

4. Which of the following countries can be the same color as W?

A. S
B. X
C. Y
D. Z

A B C D E

5. Which of the following is a pair of countries that can be the same color?

A. R and S
B. S and W
C. W and X
D. X and Y
A B C D E
Questions 6 to 11 depends on the following passage

A college president wishes to select four members of a faculty-student committee as special


representatives to meet with the college's board of trustees.
The faculty-student committee consists of eight members four of which (F, G, H and I) are faculty
members whereas the other four (R, S, T and U) are students.
The president can select any four of the eight committee members as long as the following rules
are observed:
The four representatives must consist of exactly two faculty members and two students.
Either F or G must be one of the representatives but F and G both cannot be the representatives.
If R is a representative then H must also be a representative.
If T is a representative then G cannot be a representative.
6. If T is a representative but H is not a representative then the whole group can be
determined if it were also true that:

A. F is a representative.
B. I is a representative.
C. R is not a representative.
D. U is not a representative.

A B C D E
7. If R is a representative then which of the following CANNOT be a representative?

A. H
B. I
C. S
D. T

A B C D E
8. If G is a representative then which of the following can be the other three representatives?

A. F, S and U
B. H, I and R
C. H, R and S
D. I, R and U
A B C D E
9. If neither S nor U is a representative then which of the following is the pair of faculty-
member representatives?

A. F and G
B. F and H
C. F and I
D. G and H
A B C D E

10. If G, I and S are representatives then which of the following must also be a
representative?

A. H
B. R
C. T
D. U

A B C D E

11. If F and I are representatives then which of the following is not a representative?

A. I
B. S
C. U
D. R

A B C D E

Questions 12 to 14 depends on the following passage

At a congress of the Ruling Party, the seven top party leaders, who are all cabinet
ministers, are seated on a platform in order of rank the Prime Minister being in the
center. The closer a person is to the Prime Minister; the higher is his/her rank.
Moreover, a person sitting on the right of the PM outranks the one sitting equidistant on
the left of the PM. The seven leaders are T, U, V, W, X, Y, and Z.
Y is four places to the left of the Minister of Agriculture, who is two places to the right of
V.
U’s neighbors are T and the Minister of Agriculture.
Z is two places to the left of W.
The Ministers of Education, Mining and Culture are seated together, in order, from left to
right.
The remaining Ministers are those of Social Welfare and Defense.

12. The fifth ranking person in the party hierarchy is:


A. Z, the Minister of Mining
B. Y, the Minister of Culture
C. W, the Prime Minister.
D. X, the minister of Defense.

A B C D E
13. How many of the seven party leaders outrank the Minister of Education?
A. 3
B. 4
C. 5
D. 6

A B C D E

14. The lowest ranking Minister is


A. Minister of Social Welfare.
B. Minister of Defense.
C. Minister of Education.
D. Minister of Mining.

A B C D E

15. “A meadow in springtime is beautiful, even if no one is there to appreciate it.”


This statement would be a logical opposite to which of the following claims?
A. People will see only what they want to see.
B. Beauty exits only in the eyes of the beholder.
C. Beauty does not depend on seasons.
D. The greatest pleasure available to mankind is the contemplation of
beauty.

A B C D E
III Choose the correct answer for each
question and shade the
VERBAL No. Of corresponding CIRCLE in the answer
10
Section Questions sheet

Each sentence below has one or two blanks, each blank indicates that
something has been omitted. Beneath the sentence are five lettered words or
sets of words. Choose the word or set of words that, when inserted in the
sentence, best fits the meaning of the sentence as a whole.
1. Some illnesses such as smallpox, which have been almost eliminated in the United
States are still ____ in many places abroad.
A. discussed
B. prevalent
C. scarce
D. unknown
E. hospitalized

A B C D E

2. A recent study indicates that the crime rate in the United States remains ____ and
that one in three households ____ some form of major crime in any year
A. incredible ... witnesses
B. astronomical ... experiences
C. simultaneous ... perpetrates
D. unsuccessful ... initiates
E. defeated ... prosecutes

A B C D E

Each question below consists of a related pair of words or phrases, followed by


five lettered pairs of words or phrases. Select the lettered pair that best
expresses a relationship similar to that expressed in the original pair.

3. SALVAGE : TREASURE
A. settle : argument
B. incorporate : company
C. send : correspondence
D. rescue : victim
E. recycle : newspaper
A B C D E

4. CONTROVERSY : ARBITRATOR
A. peacemaker : conflict
B. artifact : anthropologist
C. game : referee
D. dispute : mediator
E. disease : pathologist

A B C D E

Read the passages and answer the questions given at its end:
We are profoundly ignorant about the origins of language and have to content ourselves
with more or less plausible speculations. We do not even know for certain when
language arose, but it seems likely that it goes back to the earliest history of man,
perhaps half a million years. We have no direct evidence, but it seems probable that
speech arose at the same time as tool making and the earliest forms of specifically
human cooperation. In the great Ice Ages of the Pleistocene period, our earliest human
ancestors established the Old Stone Age culture; they made flint tools and later tools of
bone, ivory, and antler; they made fire and cooked their food; they hunted big game,
often by methods that called for considerable cooperation and coordination. As their
material culture gradually improved, they became artists and made carvings and
engravings on bones and pebbles, and wonderful paintings of animals on the walls of
caves. It is difficult to believe that the makers of these Paleolithic cultures lacked the
power of speech. It is a long step Admittedly, from the earliest flint weapons to the
splendid art of the late Old Stone Age: the first crude flints date back perhaps to
500,000 B.C., while the finest achievements of Old Stone Age man are later than
100,000 B.C.; and, in this period, we can envisage a corresponding development of
language, from the most primitive and limited language of the earliest human groups to
a fully developed language in the flowering time of Old Stone Age culture.

How did language arise in the first place? There are many theories about this, based on
various types of indirect evidence, such as the language of children, the language of
primitive societies, the kinds of changes that have taken place in languages in the
course of recorded history, the behavior of higher animals like chimpanzees, and the
behavior of people suffering from speech defects. These types of evidence may provide
us with useful pointers, but they all suffer from limitations, and must be treated with
caution. When we consider the language of children, we have to remember that their
situations are quite different from that of our earliest human ancestors, because the
child is growing up in an environment where there is already a fully developed language,
and is surrounded by adults who use that language and are teaching it to him. For
example, it has been shown that the earliest words used by children are mainly the
names of things and people (“Doll,” “Spoon,” “Mummy”): but, this does not prove that
the earliest words of primitive man were also the names of things and people. When
the child learns the name of an object, he may then use it to express his wishes or
demands: “Doll!: often means “Give me my doll!” Or “I’ve dropped my doll: pick it up
for me!”; the child is using language to get things done, and it is almost an accident of
adult teaching that the words used to formulate the child’s demands are mainly nouns,
instead of words like “Bring!”’ “Pick up!”; and so on.

5. The main idea of this excerpt is


(A) to provide evidence of the origin of language.
(B) to present the need for language.
(C) to discuss how early man communicated.
(D) to present the culture of early man.
(E) to narrate the story of English.

A B C D E

6. Theories of the origin of language include all of the following EXCEPT


(A) changes occurring through the years.
(B) the need to communicate.
(C) language of children.
(D) the first man’s extensive vocabulary.
(E) communication among primitive men.
A B C D E

7. The purpose of the discussion of the word, “Doll,” is intended to


(A) Trace the evolution of a noun.
(B) Support the fact that naming things is most important.
(C) Indicate how adults teach language to children.
(D) Show the evolution of many meanings for one word.
(E) Evince man’s multiple uses of single words
A B C D E

8. The implication of the author regarding the early elements of language is that
(A) There were specific real steps followed to develop our language.
(B) Care must be exercised when exhuming what we consider the roots of
language.
(C) We owe a debt of gratitude to the chimpanzee contribution.
(D) Adults created language in order to instruct their children.
(E) Language was fully developed by primitive man.

A B C D E

9. If we accept that primitive man existed for a very long period of time without
language, then we may assume that
(A) language is not necessary to man’s existence.
(B) language developed with the developing culture of primitives.
(C) primitives existed in total isolation from one another.
(D) children brought about a need for language.
(E) mankind was not intended to communicate.

A B C D E

10. After a reading of this article, one might infer that


(A) society creates problems with language.
(B) language is for adults to instruct children.
(C) society uses language to improve itself.
(D) with the evolution of language came wisdom.
(E) language brings power.

A B C D E
Answer Keys to
Drill Tests

A
AA BB
B C D
DD EE
EEE
DRILL TEST I - ANSWER KEY

Section-I Quantitative

1. B C D E
15. A B C E
2. A B C D E
16. A C D E
3. A C D E
17. A B C E
4. A B C D
18. A B D E
5. A B D E
19. A C D E
6. A B D E
20. A B C D
7. A B C E
8. A C D E
Section-III Verbal
9. B C D E
1. A B D E
10. A B D E
2. A B D E
3. A C D E
Section-II Analytical Reasoning
4. A C D E
5. B C D E
1. A C D E
6. A B C E
2. A B C E
7. B C D E
3. A B C D
B C 8. A C D E
4. A D
9. A B C E
5. A B D E
10. A B D E
6. B C D E
11. A B D E
7. A C D E
12. B C D E
8. A B C E
13. B C D E
9. A B C E
14. A C D E
10. A B C D
15. A C D E
11. A B D E
16. A C D E
12. A B D E
E 17. A B C E
13. A C D
18. A B C E
14. A B C D
19. A B C E
20. B C D E
DRILL TEST II - ANSWER KEY

Section-I Quantitative

1. A C D E
15. B C D E
2. A C D E
16. A B D E
3. A B C E
17. A B C E
4. A B C D
18. A C D E
5. A C D E
19. A B C E
6. A B D E
20. A B C D
7. A B C E
8. A C D E
Section-III Verbal
9. A B D E
1. A B D E
10. A B C E
2. A C D E
3. B C D E
Section-II Analytical Reasoning
4. A B D E
5. A B C D
1. A C D E
6. A B D E
2. B C D E
7. A C D E
3. A C D E
A B E 8. A B C E
4. D
9. A B D E
5. A B D E
10. A C D E
6. A B C D
11. A B C D E
7. B C D E
12. A B D E
8. A B C D
13. A C D E
9. A B D E
14. A B C D
10. A B C D
15. A C D E
11. A B C E
16. A B D E
12. B C D E
B 17. A B D E
13. A C D
18. A B D E
14. A B D E
19. B C D E
20. A B C E
DRILL TEST III - ANSWER KEY

Section-I Quantitative

11. A B C D E
1. A B C D E
12. A B C D E
2. A B C D E
13. A B C D E
3. A B C D E
14. A B C D E
4. A B C D E
15. A B C D E
5. A B C D E

6. A B C D E
Section-III Verbal
7. A B C D E
1. A B C D E
8. A B C D E
2. A B C D E
9. A B C D E
3. A B C D E
10. A B C D E
4. A B C D E
11. A B C D E
5. A B C D E
12. A B C D E
6. A B C D E
13. A B C D E
7. A B C D E
14. A B C D E

A B C D E
8. A B C D E
15.
A B C D E 9. A B C D E
16.
A B C D E 10. A B C D E
17.
18. A B C D E

19. A B C D E

20. A B C D E

Section-II Analytical Reasoning


1. A B C D E
2. A B C D E
3. A B C D E
4. A B C D E
5. A B C D E

6. A B C D E

7. A B C D E

8. A B C D E

9. A B C D E

10. A B C D E
DRILL TEST IV - ANSWER KEY
Section-I Quantitative

11. A B C D E
1. A B C D E
12. A B C D E
2. A B C D E
13. A B C D E
3. A B C D E
14. A B C D E
4. A B C D E
15. A B C D E
5. A B C D E

6. A B C D E
Section-III Verbal
7. A B C D E
1. A B C D E
8. A B C D E
2. A B C D E
9. A B C D E
3. A B C D E
10. A B C D E
4. A B C D E
11. A B C D E
5. A B C D E
12. A B C D E
6. A B C D E
13. A B C D E
7. A B C D E
14. A B C D E

A B C D E
8. A B C D E
15.
A B C D E 9. A B C D E
16.
A B C D E 10. A B C D E
17.
18. A B C D E

19. A B C D E

20. A B C D E

Section-II Analytical Reasoning


1. A B C D E
2. A B C D E
3. A B C D E
4. A B C D E
5. A B C D E

6. A B C D E

7. A B C D E

8. A B C D E

9. A B C D E

10. A B C D E
SAMPLE TEST
General

Note: The Sample Test does not include quantitatively the same number
of questions as there would be in the actual papers. They are merely
meant to provide conceptual guidance to the users or prospective
candidates.
I Choose the correct answer for each
question and shade the
VERBAL No. Of corresponding CIRCLE in the answer
15
Section Questions sheet

Each sentence below has one or two blanks, each blank indicates
that something has been omitted. Beneath the sentence are five
lettered words or sets of words. Choose the word or set of words
that, when inserted in the sentence, best fits the meaning of the
sentence as a whole.

1. Despite the millions of dollars spent on improvements, the telephone system in India
remains ________ and continues to ___________ the citizens who depend upon it.

A. Primitive…inconvenience
B. Bombastic...upset
C. Suspicious...connect
D. Outdated...elate
E. Impartial...vex

2. Unlike the images in symbolist poetry which are often vague and _______ , the
images of surrealist poetry are startlingly ________ and bold.

A. extraneous...furtive
B. trivial...inadvertent
C. obscure...concrete
D. spectacular...pallid
E. symmetrical…virulent

3. A good trial lawyer will argue only what is central to an issue, eliminating
___________ information or anything else that might __________ the client.

A. Seminal...amuse
B. Extraneous...jeopardize
C. Erratic...enhance
D. Prodigious...extol
E. Reprehensible…initiate

4. Pollen grains and spores that are 200 millions old are now being extracted from
shale and are ____________ the theory that the breakup of the continents occurred
in stages; in fact, it seems that the breakups occurred almost __________ .

A. refining...blatantly
B. reshaping...simultaneously
C. countermanding...imperceptibly
D. forging...vicariously
E. supporting...haphazardly
Each question below consists of a related pair of words or phrases, followed by
five lettered pairs of words or phrases. Select the lettered pair that best
expresses a relationship similar to that expressed in the original pair.

5. DETENTION : RELEASE ::

A. viciousness : attack
B. calamity : repair
C. qualification : employ
D. induction : discharge
E. therapy : confuse

6. PONDEROUS : WEIGHT ::

A. eternal : temporality
B. convincing : decision
C. gargantuan : size
D. ancient : value
E. prototypical : affection

7. FEBRILE : ILLNESS ::

A. tenacious : astonishment
B. juvenile : maturity
C. classic : cultivation
D. eccentric : discrimination
E. delusional : insanity

8. EQUIVOCATION : MEANING ::

A. feint : intention
B. secrecy : stealth
C. geniality : amiability
D. travesty : insight
E. refinement : innovation

Choose the lettered word or phrase that is most nearly opposite in


meaning to the word in capital letters.

9. WHIMSICAL :

A. chivalrous
B. perfect
C. predictable
D. hidden
E. backward

10.REVERE :

A. collide
B. succumb
C. threaten
D. divide
E. despise
11.INURED :

A. authoritative
B. dissolute
C. bereft
D. sensitive
E. taxing

12.ALACRITY :

A. skullduggery
B. reluctance
C. interment
D. bellicosity
E. specificity

Read the passages and answer the questions asked at its end.

Art, like words, is a form of communication. Words, spoken and written, render
accessible to humans of the latest generations all the knowledge discovered by the
experience and reflection, both of preceding generations and of the best and foremost
minds of their own times. Art renders accessible to people of the latest generations all
the feelings experienced by their predecessors, and those already felt by their best and
foremost contemporaries. Just as the evolution of knowledge proceeds by dislodging and
replacing that which is mistaken, so too the evolution of feeling proceeds through art.
Feelings less kind and less necessary for the well-being of humankind are replaced by
others kinder and more essential to that end. This is the purpose of art, and the more
art fulfills that purpose the better the art; the less it fulfills it, the worse the art.

13.The author develops the passage primarily by

A. theory and refutation


B. example and generalization
C. comparison and contrast
D. question and answer
E. inference and deduction

14.According to the author, knowledge is

A. evolutionary and emotional


B. cumulative and progressive
C. static and unmoving
D. dynamic and cyclical
E. practical and directionless

15.According to the passage, all of the following are true EXCEPT:

A. Art is a form of communication.


B. Art helps to refine sensibilities.
C. Art is a repository of experience.
D. Real art can never be bad.
E. Art is a progressive human endeavor.
II
Choose the correct answer for
each question and shade the
Analytical No. Of
20 corresponding CIRCLE in the
Reasoning Questions
answer sheet
Section

Questions 16-19 are based on the following.

The Western Derby is a race held annually at Bayshore Racetrack. There are eight
gates at the racetrack, but only seven horses are entered in this race—Julius
Caesar, King's Bounty, Longshot, Man Among Boys, Nocturnal, Odyssey, and
Phantom. One of the gates is left empty. The horses are at the gate, waiting for the
race to begin.

Gate 1, on the inside of the racetrack, is occupied.


Phantom is at a gate inside of Nocturnal.
The number of gates separating Julius Caesar and King's Bounty equals the number
of gates separating Longshot and Man among Boys.
Nocturnal and Odyssey are next to each other.

16. If Odyssey is at Gate 2, which of the following must be true?

A. Nocturnal is at the innermost gate.


B. King's Bounty is at the outermost gate.
C. A horse occupies the outermost gate.
D. Phantom is at the innermost gate.
E. The outermost gate is not empty.

17. Which of the following is a possible assignment for the horses, from the inside to
the outside?

A. Phantom, King's Bounty, Julius Caesar, Odyssey, Nocturnal, Man Among


Boys, Longshot, vacant
B. vacant, Phantom, Julius Caesar, Longshot, King's Bounty, Man Among Boys,
Nocturnal, Odyssey
C. Longshot, Man Among Boys, Nocturnal, vacant, Phantom, Odyssey, King's
Bounty, Julius Caesar
D. Julius Caesar, King's Bounty, Longshot, Phantom, vacant, Man Among Boys,
Nocturnal, Odyssey
E. Phantom, Julius Caesar, Nocturnal, vacant, Odyssey, King's Bounty,
Longshot, Man Among Boys

18.If Julius Caesar is at Gate 6, King's Bounty is at Gate 7, and Odyssey is at Gate
4, which of the following must be true?
I. Longshot is at Gate 1.
II. Nocturnal is at Gate 5.
III. Man Among Boys is at Gate 2.
IV. Gate 8 is vacant.

Building Standards in Education and Professional Testing 165


A. I and II only
B. II and III only
C. II and IV only
D. I, II, and III only
E. I, II, III, and IV

19.If Julius Caesar and King's Bounty are at the second and fourth gates,
respectively, all of the following can be true EXCEPT

A. Phantom is at Gate 1
B. Man Among Boys is at Gate 3
C. Longshot is at Gate 6
D. Odyssey is at Gate 7
E. Nocturnal is at Gate 7

20. Studies have shown that families who install smoke detectors and own fire
extinguishers have a reduced risk of losing a child in a house fire. Therefore, no
family who installs smoke detectors and owns a fire extinguisher will lose a child
in a house fire.

Of the following, the best criticism of the argument above is that the argument
does not

A. take into account the possibility of losing a child in a house fire despite all
precautionary measures
B. indicate that fire extinguishers are effective during early stages of a fire
C. cite the fact that smoke detectors have proven to be effective in waking
sleeping children during a house fire
D. differentiate between the two major causes of house fires: cooking and
heating
E. take into account that families who buy smoke detectors are also more likely
to purchase fire insurance

21.LSD is a drug known to cause synesthesia, a phenomenon in which sensory input


somehow becomes interchanged in the brain: a person with synesthesia might
smell a symphony, hear sun light, or taste a pinprick. While most cases are drug
induced, some people suffer from synesthesia in various forms since birth.

Which of the following can be most safely inferred from the information above?

A. Synesthesia is not always a drug-induced phenomenon.


B. Some great artists of this century have been known for their synesthetic
proclivities.
C. LSD is an addictive drug.
D. Synesthesia is rarely bothersome to those who experience it.
E. Synesthesia at birth is a result of mothers who have tried LSD.

Building Standards in Education and Professional Testing 166


22. Palindromes are easier to solve than acrostics, but acrostics are more difficult to
create than palindromes. Rebuses are more difficult to solve than acrostics, yet
rebuses are easier to create than palindromes.
If the above information is true, then it must also be true that

A. acrostics are more difficult to create than rebuses


B. palindromes are more difficult to solve than rebuses
C. rebuses are easier to solve than acrostics
D. acrostics are easier to create than rebuses
E. rebuses are easier to solve than palindromes

Questions 23-25 are based on the following.

A university has a procedure for registering and recording complaints. Due to


strict bureaucratic regulations, the following system of passing complaints must be
observed:

A is the first registrar to receive all incoming complaints.


F is the recorder and final administrator to handle a complaint.
Personnel B, C, D, and E may pass complaints only as follows:
A to B
B to either C or D
C to either B or E
D to C
E to either D or F

23.Which is an acceptable path for a complaint to follow, passing from A?

A. B to C to D to F
B. B to D to C to F
C. B to C to E to F
D. B to E to F
E. D to C to F

24.If a complaint is received and is handled by each personnel member only one
time, which of the following could be one of the passes?

A. A to C
B. C to B
C. C to F
D. D to C
E. E to D

25.Between which two personnel may a complaint pass by means of two different
paths without any duplication of passes?

A. B to E
B. C to D
C. C to E
D. D to B

Building Standards in Education and Professional Testing 167


E. E to B

Building Standards in Education and Professional Testing 168


Questions 26-31 are based on the following.

In a baseball field, one team can practice at a time. There are seven teams—the
Aces, the Bears, the Cubs, the Ducks, the Eagles, the Falcons, and the Giants.
The baseball field is open seven evenings a week from Monday to Sunday
(Sunday being considered the last day of the week), and the allocation of
practice times is governed by the following rules:

On any evening, only one team can play.


The Aces must practice on Monday.
The Ducks practice exactly one day before the Falcons practice.
The Falcons practice exactly one day before the Giants practice.
The Cubs and the Bears must practice earlier in the week than the Eagles.

26.The latest day in the week that the Bears can practice is

A. Tuesday
B. Wednesday
C. Thursday
D. Friday
E. Saturday

27.If a person went to the baseball field on three consecutive evenings, he or she
could see which of the following teams in the order listed?

A. the Falcons, the Giants, the Cubs


B. the Falcons, the Giants, the Ducks
C. the Aces, the Ducks, the Cubs
D. the Bears, the Cubs, the Falcons
E. the Ducks, the Eagles, the Falcons

28.One week, the Cubs practiced on Wednesday and the Ducks practiced the next
day. That week, the Bears must have practiced on

A. Monday
B. Tuesday
C. Friday
D. Saturday
E. Sunday

29.If the Giants practice on Thursday, the Eagles and the Ducks must practice on
which days, respectively?

A. Sunday and Tuesday


B. Saturday and Tuesday
C. Friday and Wednesday
D. Wednesday and Thursday
E. Tuesday and Monday

30.If the Falcons practice on Saturday, the Eagles must practice on what day?

A. Tuesday

Building Standards in Education and Professional Testing 169


B. Wednesday
C. Thursday
D. Friday
E. Sunday

31.The practice schedule has to adhere to which of the following?

A. The Ducks practice earlier in the week than the Eagles.


B. The Falcons practice on a later day than the Eagles.
C. The Falcons practice earlier in the week than the Giants.
D. The Cubs practice earlier in the week than the Ducks.
E. The Bears practice earlier in the week than the Cubs.

32.Wine, cheese, butter, and raisins are all examples of early techniques to preserve
food. In modern times, food scientists have developed other techniques such as
dehydration, hermetic sealing, and radiation. Of these, radiation is the most con-
troversial because preliminary studies have shown that radiation alters the
natural chemical bonds in fruits and vegetables. Instead of providing salutary
effects, eating radiated produce may well introduce irritating chemicals into the
body, creating a possible health hazard.

Which of the following, if true, supports the conclusion that eating radiated
produce poses a possible health hazard?
A. Radiation affects only those chemical bonds associated with water, that is,
hydrogen and oxygen.
B. Radiation kills microorganisms that hasten food decay.
C. The radiation-induced bonds are unlike any of those found in non-radiated
produce.
D. Certain microorganisms, namely those found in yogurt cultures, are essential
for proper digestion.
E. Radiation has no effect on foods preserved by drying.

33. Blue Blood, Inc., is a private blood products company that buys blood only from
qualified donors. To qualify, a person must weigh at least 105 pounds, must not
have taken malaria medication in the last three years, must never have had
hepatitis, and must never have used intravenous drugs. Blue Blood nurses know
that traveling has an effect on the possibilities for blood donation: Everyone who
travels to Malaysia is required to take malaria medication; no one who enters
Singapore can have ever used intravenous drugs; everyone traveling to Gorisimi
gets hepatitis.

Which of the following situations would not automatically disqualify a person


from selling blood to Blue Blood?

A. traveling to Malaysia two years ago


B. having once weighed 110 pounds and now weighing 95 pounds
C. being denied admission to Singapore
D. traveling to Gorisimi five years ago
E. using intravenous drugs that were legal at the time

Building Standards in Education and Professional Testing 170


34.Before marriage, couples should be tested for AIDS and any other sexually
communicable diseases. Negative results will guarantee the health and safe-ness
of their marriage.

Which of the following is an assumption of the argument in the passage


above?

A. Current state laws require couples who are planning to get married to be
tested for infectious disease in order to prevent possible health problems in
the future.
B. There are many infectious diseases that can be sexually transmitted from
one individual to another.
C. Fortunately even if a test proves positive for a communicable disease,
couples can still lead healthy marriages by taking the proper precautions.
D. Due to advances in medical research over the years, infectious diseases that
used to be fatal can now be effectively treated.
E. All the diseases detectable through testing have no incubation period and the
results of these tests can immediately indicate whether or not the individual
has the disease.

Question 35 is based on the following.

Nine athletes attend a sports banquet. Three of the athletes—}, K, and L—are varsity
football players; two of the athletes—M and N—are varsity basketball players. The
other four athletes— O, P, Q, and R—belong to the hockey club. All nine athletes will
be seated at three small tables, each seating three athletes. The athletes must be
seated according to the following rules: O and J do not sit at the same table.

P sits together with at least one of K or M.

There can be at most only one football player at a table.

There can be at most only one basketball player at a table.

35.Suppose just one varsity athlete sits at a certain table, and that athlete happens
to be J. If so, who else sits with J?

A. P, Q
B. P, R
C. Q, R
D. O, Q
E. O, P

Building Standards in Education and Professional Testing 171


Choose the correct answer for
III
No of each question and shade the
Quantitative 15
Questions corresponding CIRCLE in the
Section
answer sheet

36. If √y = 9, then y2 - √y =

A. √3-9
B. 0
C. 9-√3
D. 6552
E. 6561

37.If (x+3)/6 = 12/(x+4), what is the positive value of x?

A. 2
B. 3
C. 5
D. √60
E. 12

38. Cindy wants to paint her office. She can buy three cans of the same-priced paint
and three identical brushes for $21, or she can buy four cans of the same paint
and one brush for $22. How much does a can of paint cost?

A. $2
B. $3
C. $4
D. $5
E. $6

39.Which of the following must be true?

I. (25 - 81) = (5 - 9) (5 + 9)
II. 7(9 + 6) = 7(9) + 7(6)
III. 6 ÷ (3 - 1) = (6 ÷ 3) - (6÷1)

A. I only
B. II only
C. III only
D. I and II only
E. I, II, and III

40.The sum of a and 9 - 2a is less than 8. Which of the following is (are) the
value(s) of a?

I. a<-1
II. a< 1

Building Standards in Education and Professional Testing 172


III. a>1

A. I only
B. II only
C. III only
D. I and II only
E. I and III only

41.Susan is having a party. At 7:00 P.M., guests begin arriving at a uniform rate of
8 people every 15 minutes. If this pattern continues, how many guests will have
arrived by 9:30 P.M.?

F. 10
G. 20
H. 40
I. 64
J. 80

42. For positive integers p and q, if p2 + 2q2 = 41, and 2p2 +q2 = 34, then p2 =

A. 2.5
B. 7
C. 3
D. 9
E. 16

43.If a:b is 7:6 and 3b:2c is 2:3, what is c/a ?

A. 14/27
B. 7/9
C. 6/7
D. 9/7
E. 27/14

44.In the figure above, if the radius of the


circle is 8, and triangle TRS is inscribed in
the circle, then the length of arc TRS is

A. 16π/3
B. (32 π)/3
C. 16 π
D. (128 π)/3
E. 64 π

45. For developing pictures, XYZ Photo Lab


charges a service fee of $3 for every order
it receives in addition to a printing fee. If
the order consists of 12 pictures or less, the printing fee per picture is $0.36. If

Building Standards in Education and Professional Testing 173


the order consists of more than 12 pictures, the printing fee per picture is
$0.24. What is the total cost per picture for an order consisting of 30 pictures?

A. $0.11
B. $0.24
C. $0.34
D. $0.46
E. $3.24

46.Lisa found an easy way to add up a sequence of positive even integers with an
even number of terms. She formed pairs of equal sums by adding the first
integer to the last, the second integer to the next-to-last, and so on. She then
computed the total by adding these equal sums. If the total Lisa obtained was
930, how many terms were there in the sequence of positive even integers if
the sequence started with the number 2?

A. 30
B. 39
C. 40
D. 60
E. 465

47. December is the busiest month at Lamont's Gift Shoppe, where sales in
December are 40 percent higher than average. If sales in February are typically
20 percent lower than average, what is the ratio of February sales to December
sales?

A. 1:2
B. 4:2
C. 4:5
D. 4:7
E. 6:7

48.How many 4-digit numbers are there that consist of only odd digits?

A. 20
B. 625
C. 1,024
D. 4,500
E. 5,000

49. For some integer m, let [m] be defined by the equation {m} = m (1- m). If n +
1 = {n + 1}, then n =

A. -2
B. -1
C. 0
D. 1
E. 2

Building Standards in Education and Professional Testing 174


50.Box A and box B have 6 cards each. Each card is marked with one integer, 1
through 6. Both boxes can have more than one card with the same integer, but
the sum of all the integers in each box must be 18. Two of the cards in box/1
are 6's and two of the cards in box B are 5's. If one card is drawn from box A
and one from box B, but neither a 6 nor a 5 is drawn, what is the largest pos-
sible sum of the integers on the cards drawn from the two boxes?

A. 3
B. 4
C. 7
D. 8
E. 12

Building Standards in Education and Professional Testing 175

You might also like